1Dr - Mamoun Jamous

You might also like

Download as pdf or txt
Download as pdf or txt
You are on page 1of 258

‫الزمالء األفاضل‬

‫ سؤال‬0222 ‫أضع بين أيديكم مذكرتي‬


‫آمل أن يُكلل هذا العمل بالتوفيق و النجاح لكم‬
‫ مأمون جاموس‬.‫د‬
. ‫ عبدالمعطي الذي ساعدني بكتابتها على نظام وورد‬.‫ونشكر د‬

1. Female with ABD Mass history of caesarean section planed for laparoscopy repair hernia, most
common complication?

a. Pain
b. Infection
c. Adhesion Formation
d. Cosmetic
e.
2. Barium patient with gastric band obesity 2 years ago present for vomiting and distension?

a. Endoscopic
b. Laparoscopic
c. Laparotomy

3. Old patient with complete rectal prolapsed is?


a. Altemier

4. Longitudinal tear in CBD during Laparoscopic Cholecystectomy management.

a. Primary closure with suture


b. ERCP and stent via the rent
c. Clip of the rent and drain
d. Closure over T-Tube

5. Myloprolifirative huge spleen with severe left hypochondrial pain for diagnosis.

a. Laparoscopy
b. MRI
c. CT with contrast

6. Hip metastasis after of breast cancer management of pain.

a. Radiotherapy
b. Opiod
c. Bisphosphonates Infusion

7. Bleeding per rectum, colonoscopy and barium re negative except for hyper vascular area in
transverse colon most common:
a. Colon Cancer
b. Dieulafoy lesion
c. Aortoenteric fistula

8. 73 years old male with an alcoholic liver disease previous appendectomy, cachectic, vomiting
positive succession splash?

a. Intestinal Tumor
b. Adhesion
c. Ascites
d. Gastric outlet obstruction

9. Severe diarrhoea, patient disturbed conscious Na 162, K 4.9 best fluid replacement.

a. Saline
b. Ringer
c. D5
d. Dextrose-saline
10. Female 20 years old, history of negative abdominal exploration for acute abdominal 2 months ago,
come with vomiting distension, positive trill fluid, aspiration positive amylase management?

a. ERCP
b. Laparoscopy
c. CT

11. Rectal Cancer 4cm from anal margin for nodal staging.

a. CT abdominal and pelvis


b. Endoanal Ultrasound
c. MRI
d. PET

12. Traumas -> exploration -> infected ->septic shock. The cause of poor peripheral tissue perfusion.
a. Low cardiac output
b. Resistance
c. Ineffective perfusion in peripheral tissue
d. Glomerular filtration rate

13. Risk for breast cancer.

a. Menarche at 14
b. Contraceptive pills
c. Leave born after 30
d. History of mother with breast cancer

14. Most suitable surgery for wt. Reduction.

a. Gastric by pass

15. Hard Thyroid nodule woodly with compression manifestation with history of episodes of
hypothyroidism and jaundice, best diagnosis?

a. FNA
b. Radionuclide
c. Thyroid Function

16. Patient with hypertension, hypernatremia, hypokalemia and mass above kidney.

a. Cushing
b Adrenal tumor

17. Best method for localization of parathyroid nodule before surgery?

a. Subtraction scan
b. Sestamibi scan
18. Spherocytosis planned for splenectomy investigation.

a. Fragility test
b. U.S to rule out gall bladder stone

19. Melanoma 2mm thickness safety margin

a. 0.5
b. 1
c. 1.5
d. 2cm

20. 24 year explosive injury, the II?

a. Burn
b. Shrapnels

21. Explosion with neck injury, exploration esophageal & proximal sca injury treatment

a. Repair of both
b. Repair artery and esophagostomy
c. Ligation artery and repair esophagus
d. Ligation artery and esophagostomy

22. Buttock infection with crepitus with gram positive spore forming bacilli treatment?

a. Penicillin G

23. Hemorrhagic shock Heart Rate: 130, Blood Pressure: 85, Pulse Rate: 35. What is percentage of
blood loss?

a. <15%
b. 15-30%
c. 30-40%
d. >40%

24. Hemorrhagic shock Heart Rate --, Blood Pressure ---, Respiratory Rate—what is the replaced
fluid?
a. Starch
b. Blood
c. Isotonic

25. Best Mesh for hernia repair?

a. Non absorbable
b. Delayed absorbable
c. Iinelastic
26. Case dark urine
a. Toxic shock

27. Hand infection most associated with osteomyelitis

a. Felon

28. Patient with history lumpectomy for phyllodes tumor after recurrence

a. Lumpectomy + radio
b. Lumpectomy + chemo
c. Simple mastectomy

29. Anatomic liver division depends on?

a. Hepatic Vein
b. Portal vein
c. Hepatic Artery

30. In achalasia there is absence of

a. Complete of relaxation of low esoph


b. Incomplete of relaxation of low esoph
c. Esophageal peristalsis

31. Most common anomaly with gastroschisis

a. Malrotation
b. Intestinal atresia

32. Exploration for appendectomy, injury of cecal diverticula, soiling, cecum is massively involved?

a. Ileocecectomy
b. Right Hemicolectomy
33. Diverticulitis with pensegmoid abscess 6x6.

a. Antibiotics
b. Percut aspiration

34. Pain in L.L.Q CT. -> abscess contagions confluent with sigmoid cause

a. Rupture diverticulum
b. Bleeding in abscess

35. 32 years old female who is 24 weeks pregnant present to the emergency department with an
acute onset of abdominal pain, fever and vomiting she states the pain wake her up in the middle the
night with sudden onset of epigastric pain that is now diffuse. She has no vaginal bleeding and fetal
monitoring demonstrates normal vitals for the fetus. Upon physical exam, the patient has diffuse
tenderness with guarding throughout the abdomen, worse in the epigstric region, pelvic examination
is normal, she has a leukocytosis of 15000 cells, abdominal x-ray series shows some dilated bowel
loops but no other findings, what is your next step in management of this patient?
a. Abdominal ultrasound
b. CT of abdominal pelvis and contrast
c. Admit and observe with serial abdominal exam
d. Explanatory laparotomy
e. Diagnostic laparoscopy

36. 30 years old patient presented with stool mixed with blood since two weeks, upper
Gastrointestinal and lower spleen, diagnosis is?

a. Typhoid ulcer
b. Diverticulosis
c. Dividenal Ulcer
d. Meckel diverticulosis

37. The risk of regional node metastasis in 0.70mm thick melanoma is?

a. < 5%
b. 10%
c. 20%
d. 30%

38. Case scenario about patient on Anzac for simple gastric, but you shifted him to omeprazole,
patient came back and asking about complication of omeprazole especially those after 10 years of use.
The options were:

a. Nothing at all
b. Diarrhea
c. Gastric Mucosa Atrophy
d. Gastric Tumor

39. Most significant study to check for esophageal tumors invasion to nearby bronchus.

a. Bronchoscop
b. MRI
c. Esophagoscopy
d. CT Chest

40. Post neck femur surgery adult. What medication will be given as VTE prophylaxis?

a. Aspirin
b. Low dose of Heparin
c. Full dose of Heparin
d. Clopidogrel

41. Patient to do elective inguinal hernia repair recurrent, best operation

a. TAPP
b. TEP
c. Open
42. Patient with inguinal scrotal hernia presented with Abominal. Distention, vomiting, and severe
pain most likely about the hernia (not tender).

a. Obstructed
b. Stragulated
c. Incarcerated

43. Patient came from Mexico with RUQ pain

a. Amebic liver abscess

44. 30 years old patient with total extraperitonial repair of hernia, the pressure most be?

a. > 10 mmhg
b. 11-15 mmhg
c. 16-20 mmhg

45. 56 years old patient have history of acute pancreatitis showing splenic aneurysm with
calcification is?
a. Observation and follow up
b. Ligation of aneurysm
c. Splenectomy with removal of the affected artery

46. Hypokalemic change in ECG is?

a. Prolonged QT segment
b. Prominent U wave
c. Widened QRS complex

47. Scenario of TIA investigation best by

a. CT angio
b. Deplear Ultrasound
c. CT brain

48. 63 years old admitted for elective colon resection fro recurrent attacks of sigmoid diverticulitis,
you want to administer prophylactic antibiotics in choosing a regimen you should be aware that the
most common organism found in the colon of normal individuals is?

a. Escheric cdi
b. Clostridium difficile
c. Pseudomonas species
d. Bacteroides species

49. On the second day after AAA repair, patient passes grossly bloody stool, the next step is?

a. Immediate exploratory laparotomy


b. Sigmoidoscopy
c. C.T with Abdominal with contrast
d. Aortogram

50. Patient with burns to the entire back, scalp (50% of the head and neck) and the posterior
thighs has what percentage of his or her total body surface area burned

a. 40%
b. 28%
c. 20%
d. 32%

51. Most common strong anticoagulant secreted by medical leeches is

a. Heparin
b. Warrfarin
c. Hirudin

52. Sathy margin needed for ecision of malignant melanoma it thickness more than 4mm?

a. 10 mm
b. 10-20 mm
c. 20-30 mm
d. 40 mm

53. Burn degree of electrical high voltage on arm boy 15 years old

a. 1st
b. 2nd
c. 3rd
d. 4th

54. CBD structure and management and post CBD injury

a. Choledocduodenostomy with stent


b. Choledocduodenostomy without stent
c. ERCP with stent
d. End to end anastomosis

55. Patient with anal pain with no soaking cloth with colonoscopy free, pain with DRE?

a. Intersphincteric abscess
b. Pile
c. Fissure

56. Which can compromise respiratory during delivery to baby?

a. Cystic hygroma
b. Torticollis

57. Potassium level more in?


a. Saliva
b. Bleeding

58. Most common complication after haemorrhoidectomy?

a. Urine Retention

59. Output from ileostomy most be kept under colostomy


a. 500-700
b. 2500
c. 1000-1500

60. Patient with septic shock which HB value considered for blood transfusion

a. 6–7
b. 7–8
c. 8–9
d. 10

61. Unconjugated Billirubin = indirect = water insoluble conjugated billirubin = direct = water
soluble = fat insoluble

62. Type of gastritis cause by H. Pylori

a. Type B

63. Treatment of annular pancreas?

a. Duodenojejunostomy

64. Pancreatic Tumour 2cm with hypokalemia?

a. Vipoma

65. Patient underwent Lapchole with posterative day one bile leak. What is the best treatment?

a. ERCP and stent

66. Best investigation to diagnosis CHPS?

a. Endoscopy -> adult


b. U.S -> paediatrics

67. Patient complaining of pruritus at night around anus, no history of bleeding or pain or fever,
diagnosis is?

a. Anal fissure
b. Perineal abscess
c. Piles
d. Perineal fistula

68. Patient complaining of mid sigmoid colon cancer, the best management is ?

a. Anterior resection
b. Resection and anastomosis
c. Abdomino perineal
d. Hartman Procedure

69. Patient suffering from perforated diverticulutis drainage was done with colostomy, patient
received 3rd generation cephalosporin before operation and post operation for 1 week. Patient
develops fever and by investigation ->intraperitonial abscess at day 6 post operation, this most usually
due to.

a. Patient should received anti anaerobic


b. Should receive 1st generation cephalosporin
c. Multiple pre operative doses of antibiotic

70. Underweight in front presented with dipping of saliva with abdominal distension and cyanosis
and dyspmea(Ryle tube fill with air) x-ray shows Ryle in chest. What is the initial management?

a. Continuous cleaning and suction of oesophagus


b. Repair of esophagotracheal fistula
c. Gastrostomy to decompress distended stomach
d. Intubation

71. The most common anomaly with infant complaining of gastroschisis?

a. Intestinal atresia
b. Falot
c. Esophagotracheal Fistula

72. Endorectal ultrasound. How many layers detected?

a. 2
b. 3
c. 4
d. 5

73. Percentage of malignant potential in ulcerative colitis disease diagnosis?

a. 2% after 10 years
b. 8% after 20 years
c. 18% after 30 years

74. Patient with inflammatory colitis, which suggest crohn disease diagnosis?
a. Ileitis
b. Rectal Sparing
c. Crypt abscess
d. Proctitis
75.
76. Treatment of desmoids tumour in patient with FAP?

a. Tamexifen
b. Cortison
c. Radiotherapy
d. Methotrexat

77. Treatment of infected bullous dermatitis with crepitus under skin (clostridium perfringens)?

a. Clindamycine
b. Penicillin G

78. Patient with BP 120/80 createnin and urea up, diagnosis?

a. Siris
b. Sepsis
c. Sever Sepsis
d. Septic Shock

79. Patient had right heneicolictomyfod FPC 10 years ago, now presenting with a 14 cm parietal
abdominal mass at the operation site, what is the proper management?

a. Triple Chemotherapy
b. Conservative Management
c. Wide local excision
d. Combined chemo-radiotherapy

80. Patient with gastric ulcer by endoscope due to H. Pylori the priority to?

a. Eradicate H. Pylori
b. Reduce Acid Reflux
c. Endoscopic biopsy of ulcer

81. Patient with peptic ulcer not respect treatment, the best management?

a. Vagotomy
b. Highly selected Vagotomy
c. Billroth operation

82. Scenario of TIA investigation best by?

a. C.T angio
b. Duplex Ultrasound
c. C.T brain
83. Boy 10 years old play pain at hip, pain relieved by rest diagnosis?

a. Perthes disease

84. Patient complaining of bone pain, depression, loss of memory, fatigability, most likely causes?

a. High serum calcium and intact PTH


b. Decrease calcium urine level
c. Decrease serum phospat level

85. 30 years old female with longitudinal tender tear on the 9 oçlock lithotomy position with bright
bleeding per reaction. What finding merit to do biopsy?

a. Site
b. Size
c. Bleeding
d. Recurrence

86. 56 years old mail with constipation bleeding perrectumcachexy has cercumferences and verge
adenocarcinoma management?

a. Defunctioning Colostomy
b. Abdomeno perineal resection with permanent colostomy
c. Chemotherapy
d. Radiotherapy

87. Most common cause of urine tract infection with E-coli colovesical fistula?

a. Diverticulosis
b. Crhon’s disease
c. Ulcerative Colitis

88. Patient with history of diverticulosis came with high grad fever, lower abdominal pain, U.S
show EXE a hemogenesis collection on pelvis. What will you do?

a. Percutaneous drainage
b. Antibiotic
c. Fluid
d. Follow up

89. Patient take antibiotic for treatment of celulitis for more than 13 weeks, develop coliky
abdominal pain, diarrhea, fever, most important test to detected the cause?

a. Stool cytotoxin test


b. ELISA
c. Blood Culture
90. Patient with diffuse painless thyroid gland enlargement with T3, T4 normal, TSH up, Thyroid
antibodies positive?

a. Thyrodectomy
b. Radioiodine
c. Radiotherapy
d. Anti thyroid drug

91. Patient complaining abdominal mass on the anal canal biopsy slow show human papilloma
virus
a. Paget’s disease
b. Bowen’s disease
c. Adenocarcenoma
d. Abdominal 3abed disease

92. Type of shock needs vasopressin?

a. Hypovolemic
b. Cardiogenic
c. Distributive

93. What type of shock has the following characteristics? COP low, SVR high, BP low, Oxygen low.

a. Cardiogenic
b. Hypovolemic
c. Distributive

94. In ICU patient developed hypotension, hyponatremic, hypoglycaemic, and hyperkalemia. What
is the most important drug given to save the patient?

a. Hydrocortison
b. Fludrocortion
c. Calcium
d. Sodium chloride

95. Patient develop hypernatrimic, hypokalemic, hypertension, CT show mass on kidney most likely
diagnosis?

a. Adrenocortical carcinoma
b. Renal Tumour
c. Pheochromocytoma

96. Patient complaining of right side chest pain, chest x-ray show right side lung opacity cyst,
breathing diminished on the right side, diagnosis is hydrated cyst, what the most appropriate
treatment?

a. Albendazol
b. Pnemenectomy
c. PAIR
d. Excision

97. Pregnant women 2nd trimester 13 week has tremor, palpitation right neck swelling, most likely
treatment?

a. Thyrodectomy
b. Anti Thyroid drug
c. Radio iodine
d. Radiation
98. Paget disease pain of the bone management?

a. NSAID

b. Bisphosphonates

99. Extend jugdar vein in hypotension patient with normal respiratory air entry?

a. Hemotorax
b. Pneumotorax
c. Cardiac Tamponade

100. Pylovic Adenocarcinoma

a. Radical I, BI
b. Radical II, BII
c. Radical III
d. Radical IV

101. Middle age 1 month epigastic pain + diarrhea gastrin 120, next?

a. Somatostatin receptor scan


b. Endo tumour Ultrasound

102. Post liver abscess exploration 3 days abdominal detention + Oligurea, fever + tachycardia +
mild hypotension, intravesical pressure 28?

a. Colostomy
b. Exploration and evacuation
c. Colectomy
d. C.T Abdominal

103. Scenario, patient child 4 years old presented with clear vomiting, what is the best investigation?

a. Ultrasound

104. Scenario, patient old age vomiting 3 times per day. Laboratory date: K 1,8.What is ECG change
will suspected?

a. Prominent U wave with flat T wave


105. Scenario, patient do cycstectomy with ureto-segmoid diversion ECG show prominent U wave.
What is problem in patient?

a. Hypokalemia

106. Male patient diarrhea 1m, abd colicky and fistula opening above dental line. What is the best
investigation?

a. C.T colone
b. MRI
c. Colonscopy
d. Br. Enema

107. Patient present to E.R, RUQ pain + jaundice, lab show above direct billirubin, above indirect
billirubin, above alkaline phosphatise, temp 38.6. What is next step?

a. MRCP
b. ERCP
c. C.T abdominal

108. What is associated to coin indicate malignancy?

a. Calcification
b. Stella the like figure

109. Female patient age 65 years old presented with constipation distension and abdominal cramps.
Show give history that she is constipated 2 months ago but she believed that the cause was changing
diet. She also do hysterectomy 10 years ago on examination: ++ intestinal sound with tenderness right
iliae fossa. What is the disgnosis?

a. Acute Appendicitis
b. Adhesive I.O
c. Segmoid Volvulous

110. Female patient 78 years old midline swelling, not move with deglution but move upward with
swallowing. Patient refuses any surgery. What is the fate of this swelling?

a. Spontaneous Disappear
b. Malignancy Transformation
c. Laryngeal compration
d. Nothing

111. Origin of B.C.C is?

a. Pluripotent cells

112. Patient of AF gotin embolectomy it femoral artery start pain, numbness, swelling of the foot,
examination show tenderness, calf mass, popleteal artery is felt, what is the diagnosis?
a. D.V.T
b. Compartment Syndrome

113. Scenario, patient sudden epigastic pain become generalized abdominal pain and tenderness x-
ray abdominal show air under diaphragm, Diagnosis?

a. Gastric Perforation
b. Appendicular Rupture
c. Ileal Perforation

114. Scenario, patient acute right iliac fossa pain 2 days associated with vomiting. Pain started
around umbilicus P.R: tender, Diagnosis?

a. Acute Appendicitis

115. Female patient no history of jaundice. During lap cholectomy you find wide cystic duct and wide
C.B.D. what will do?

a. Cholecystectomy the Ultra Sound


b. Per –Operative Cholangiography
c. Open cholecystectomy and palpate CBD
d. Intraoperative ultra sound

116. Factors affecting recurrence of colon cancer is?

a. Staging
b. Lymph nodes

117. Scenario, patient does upper endoscopy show grade II varices. What is the best management?

a. Sclerotheraphy
b. Devascularization
c. Total Shunt

118. Old age male patient alcoholic present with molena 2 days. What is the cause?

a. Bleeding duodenal ulcer


b. Bleeding gastric ulcer
c. Oesophageal

119. Scenario of jaundice newborn, ultra sound show no evidence of any extra hepatic biliary ducts.
What is the age beyond operation will have poor results?

a. 8 weeks
b. 12 weeks
c. 14 weeks
d. 18 weeks
120. Scenario, old age male patient suffering from upper abdominal pain. C.T show dilated distal 2/3
of pancreas, MRCP show stone in junction between distal 2/3 and 1/3 of pancreatic duct. What is best
management?

a. Distal pancreatectomy
b. Duodenopancreatectomy
c. ERCP & stent

121. Scenario 2 year’s old baby with bilateral in descended testis, what is most diagnostic test?

a. Laparoscopy
b. Karyotyping

122. Patient very malnourished, what is element should be normal if patient is for surgery?

a. Vitamin C
b. Folic Acid
c. Copper
d. Iron

123. Patient he had papillary thyroidectomy yesterday on normal saline and diet present with
generalized seizure, what is the cause?

a. Cerebral Metastasis
b. Low Calcemia

124. Scenario of patient vomiting many times. ABG parameters written PH: 7.50, PCOZ: 48, HCO3
:35. What is the management?

a. Ringer Solution
b. Saline
c. ... formula
d. ...formula

125. Patient did total thyroidectomy, presented on 6 thdaywith tingling hand calcium: 6, what is
management?

a. Vitamin D
b. Calcium
c. Vitamin + Calcium
d. Complete without treatment until 1m

126. Scenario, female patient swelling 3cm with pean de orange with supra clavicular L.N, without
no evidence of distant metastasis?

a. T3 N0 M0
b. T3 N1 M0
c. T4 Nb1 M0
d. T4 Ne1M1
127. Scenario, female patient trauma right hypochondriac pain, and yesterday she missing herself
until next day presented with severe right hypochondriac pain, ultra sound. What is possible organ to
be injured?

a. Spleen
b. Liver
c. Stomach
d. Intestine

128. Scenario, patient with sudden lumbar back, pain radiated to flanks, he give history of back
lumbar pain not radiated 2 days ago, patient vital signs: pulse 80, BP 110/70 temp: 37, after 1 hour BP:
80/40, pulse: 106, temp: 37, What is management?

a. C.T abdominal
b. Laparotomy
c. Repair of endoscopy for aneurysm

129. Patient trauma hypochondrium pain radiated to shoulders, vital signs BP: 100/70, pulse: 108.
What is initial management?

a. Ultra sound
b. C.T
c. Laparotomy

130. Scenario of patient hypovolemic shock, mild anxious, BP 120/80, P: 106. What is stage of shock?

a. I
b. II
c. III
d. IV

131. Scenario of patient has anal swelling and bleeding, swelling is protruded not reduced except
manually. What is the management?

a. Haemorrhoidectomy
b. Band
c. Ligation
d. Laxative

132. Scenario patient suffering from bleeding per rectum P-R show ulcerative mass 4cm from anal
verge. What is management?

a. Colonoscopy with biopsy


b. C.T
c. MRI

133. Scenario, patient mass is 2.5cm at base of appendix. What is management?


a. Right hemicolectomy
b. Appendectomy

134. Scenario, patient show ulcer in greater curvature. What is the management?

a. Upper endoscopy with biopsy


b. Gastrectomy

135. Scenario, patient suffer from swelling at mid 1/3 oesophagus. What is best management?

a. Endoscopy with biopsy


b. Esophagostomy

136. The commonest histological malignancy in GIT?

a. Adenocarcinoma
b. Squamous cell carcinoma

137. The best prognosis of breast cancer?

a. Mucinous
b. Paget
c. Tubular

138. Commensal that discovered cause pathogenesis staph?

a. Epidermic

139. Scenario, patient recurrent inguinal scrotal hernia, irreducible only, not obstructed, not
strangulated, history of prostatectomy. What is management?

a. Open and repair with mesh


b. Laparoscopic repair

140. Boy with piece of wood in foot, red streaking. What is the causative organism?

a. Streptococcus pyogenes

141. Patient develop systemic inflammable response syndrome SIRS, what is parameter?

a. Temp > 38.6

142. Inguinal scrotal hernia, what is sign?

a. Can’t palpate the vas

143. Gun shot in leg with paresthesia and weak preferal pulsation, management?

a. Exploration and repair


b. Expectant treatment

144. Which tumour not producing gynecomastie?

a. Lung
b. Bone
c. Prostate
d. Testis

145. Prognosis of cancer bladder based?

a. Age
b. Hematuria
c. Invasion of wall
d. Size of mass

146. Neonate 16 day old with abdominal distension and WBC= 8900, diagnosis?

a. Enterocolitis
b. Volvulus
c. Intussception

147. 2mm thickness melanoma, margin?

a. 1 cm
b. 2 cm

148. Case with non-bilious vomiting and hypokalemia, ECG changes?

a. Depressed ST segment
b. Elevated T wave

149. Daily need protein for adult person

a. 0.8mg/kg/day

150. I.V fluid resuscitation, 70kg per person?

a. 20ml / kg
b. 50ml / kg
c. 100ml / kg

151. Best investigation for GERD?

a. Esophagoscopy
b. 24hr PH monitoring
c. Manometry
d. Barium Swallow
152. Post trauma x-ray shows air in mediastium best next best next investigation?

a. CT chest
b. Esophagoscopy
c. Angiography

153. Patient with upper GIT bleeding all investigation and coagulation are normal diagnosis?

a. Dieulafoy Lesion
b. Peptic Ulcer

154. Gall Bladder Polyp 4mm management?

a. Expectant treatment
b. Lap cholecystectomy
c. Extended cholecystectomy

155. 6 months after cholecystectomy MRCP image show defect in CBD, diagnosis?

a. CBD stone
b. Hepatic duet stricture
c. CBD injury
d. Cholangiocarcinoma

156. Bile found in perotonium and cystic duct injury occur post cholecystectomy, management?

a. T-tube
b. ERCP
c. Primary repair

157. Aspiration of pancreatic sac reveals fluid rich on carbohydrate, diagnosis?

a. Pseudocyst
b. Mucinous cystadenocarcinoma
c. Serous cystadenocarcinoma

158. Hepatic mass with increase platelet consumption?

a. Hemangiome
b. HCC (heptocelularcarcinoma)
c. FNH (focal nodular hyperplasia)

159. Swimmer female underwent modified mastectomy and breast reconstruction, which type of
flap will interfere with swimming?

a. Latdorsi flap

160 Breast mass in upper inner quadrent aspiration reveals, malignant cells, management?
a. Radical mastectomy
b. Modified Mastectomy only
c. Radical mastectomy + chemotherapy
d. Radical mastectomy + chemotherapy + radiotherapy

161. Gallbladder with adenocarcinoma in mucosa and submucosa, management?

a. Cholecystectomy
b. Wedge resection of liver

162. Gallblader with squamous cell carcinoma not reaches serosa, management?

a. Celiac L.N biopsy


b. Extended cholecystectomy
c. Liver resection

163. Liver bright mass in MRI, management?

a. Biopsy
b. Resection
c. C.T

164. Stab neck in midline in the level of cricoids with hematoma, management?

a. C.T
b. Angiography
c. Exploration

165. Stab neck below angle of mandible with hematoma and weak carotid pulse and drowsiness,
management?

a. Exploration
b. C.T
c. Angiography

166. Most pre operative risk factor?

a. Aortic Stenosis
b. Bronchitis

167. Most common cause of recurrent epigastric hernia?

a. Usage of absorbable sutures in repair


b. Sport
c. Missed defect
d. Increase intra abdominal pressure

168. 45 years old male patient with late vomiting and abdominal distention, diagnosis?
a. Crohn disease
b. Sigmoid carcinoma
c. Diverticulitis

169. After Chest Trauma weak femoral pulse and x-ray reveals air in mediastinum, ehat is diagnosis?

a. Aortic rupture
b. Flail Chest
c. Tension Pneumotorax

170. What’s true about fibrocystic change of breast?

a. Disappears after menopause


b. Premalignant
c. Need surgery

171. Case of Crohon disease with perforated distal ilieum, magement?

a. Primary repair
b. Resection of perforated segment
c. Ilieocolectomy

172. Long case of alcoholic old man patient with liver cirrhosis and ascitis, management?
a. TIPS
b. TIPS & liver transplantation

173. Volckmann’s ischemic contracture occurs mostly with?

a. Fracture Femure
b. Supracondylar fracture of humerus
c. Colle’s fracture

174. Pain in flank with urine output 500 / 24h investigation?

a. C.T
b. Cystoscopy
c. IVP

175. Patient weight 105 kg and length 170cm went to endocrinologist, barium swallow show
elevated gastroesophageal junction diaphragm, management?

a. Nutritional and hormonal treatment


b. Lap gastric bypass
c. Intragastricballon
d. Lap. Banding

176. Image shows right breast erythematous mass in inner lower quadrent 3cm x 3cm in size, no L.N
detected, what is the management?
a. Antibiotic

177. Patient with history of gastric ulcer come to ER, with bleeding duodenal ulcer, management?

a. Truncalvagatomy
b. Truncal + pyloroplasty
c. Truncal + distal gastrectomy

178. Post operative hypotension and empty drains and no urine output, management?

a. Diuretics
b. Catheter flush
c. Dopamine
d. I.V fluids

179. RTA post laparotomy dark brownish urine management?

a. Urine analysis
b. Na bicarbonate

180. RTA and patient thrown aqay, he cant move his lower limbs which are worm, what’s type of
shock?
a. Hypovelemic
b. Cardiogenic
c. Neurogenic

181. Interlope infection after abdominal exploration management?

a. Antibiotic and drainage

182. Most radio sensitive tumour?

a. Neuroblastoma
b. Nephroblastoma
c. Lymphoma

183. Most common malignancy in age 30-40 years old?

a. Osteosarcoma
b. Wilm’stumor
c. Seminoma

184. Trigeminal nerve injury is caused by?

a. Fracture temporal bolk


b. Fracture maxilla
c. Fracture mandible

185. Ulcer in right check, no LN palpalable?


a. Melanoma
b. BCC
c. SCC

186. Case of exploration and although pre-operative usage of first generation cephalosporin
infection occurred, what’s the cause?

a. 3rd generation should be used


b. Operation not exceed 2hr
c. Antibiotic used for 7 days post-operative

187. New born with persistent vomiting, x-rays shows no air in GIT?

a. Oesophageal atresia

188. Patient with massive trunkal burn the way for emergent resuscitation?

a. Unburned preferal limb


b. Venous cut down
c. Central venous line

189. What is the lest tumour cause gynecomastia?

a. Colon
b. Lung
c. Prostate
d. Gallbladder
e. Testis
f. Renal
g. Bone

190. Patient come to ER with stab wound to the neck c/o S.O.B and bubbling from wound, x-ray chest
showed air in mediastinum. What is diagnosis?

a. Rupture trachea
b. Rupture Oesophagus
c. Traumatic Pnemotorax
d. Entrapped external air

191. Old patient with COPD and HD, ASA grade III, comes with rectal prolabse. O/E 7cm rectal from
anus, what is the best operation?

a. Altemeier operation – ASA 3-4

192. 3 years old patient presented with stool mixed with blood since 2 weeks, upper GI and lower
colonoscopy are negative, by examination – palpable spleen, diagnosis is?

a. Typhoid ulcer
b. Duodenal ulcer
c. Diverticulitis
d. Meckel Diverticulum

193. 56 years old patient have history of acute pancreatitis show splenic aneurysm with calcification
treatment is?

a. Observation and follow up


b. Ligation of aneurysm
c. Splenectomy with removal of the effected artery

194. Hypokalemia changes in ECG are?

a. Prolonged QT with prominent U wave


b. Widened QRS complex
C. Arythemia

195. Scenario of TIA, investigation best by?

a. C.T angio
b. C.T brain
c. Duplex U.S

196. The hernia that never completely reducible?


a. Epigastric
b. Litters
c. Sliding
d. Pantalon

197. Indirect inguinal herniareach to scrotum characterized by?

a. Cant Pelpate vas


b. Positive translumination testis
c. Irreducible

198. Bob at calf muscle appears since 1 day in patient 25 years old, spread via S.C struck to groin, the
most causative is?

a. E coli
b. S. pyogenes
c. S. Difficil
d. C.W (Clostridium Welchii) or perfringe

199. In with patient incisional hernia, during adhisiolysis resulted – perforation of transverse colon
with gross fical contamination primary closer was done, what is after?

a. Repair and mesh


b. Primary later repair
c. Closer in mass with tension
200. Patient suffering from perforated diverticulitis drainage was done with colostomy, patient
received 3rd generation cephalosporine before and post operation for 1 week, patient develops fever, -
by investigation – intraperitoneal abscess at day 6 post operation, this is most commonly due to?

a. Patient should received anti anaerobic


b. Should receive 1st generation cephalosporine
c. Multiple pre operative doses of antibiotic

201. Underweight infant presented with dripping of saliva with abdominal distension and cyanosis
and dyspnearyle tube fill with air, x-ray shows ryle in chest. What is the initial management?

a. Continuous cleaning and suction of oesophagus


b. Repair of esophagiotrachial fistula
c. Gsstrestomy to decompress distinted stomach
d. Intubation

202. Infant presented by vomiting many times, patients develops which ECG the following?

a. Hypochloremichypokalemic metabolic alkalosis


b. Hyperchloremichypokalemic metabolic alkalosis
c. Metabolic alkalosis
d. Metabolic Aciodosis

203. Patient with pain at right hypochondrium and fever by ultrasound – left lobeabscess, what is
the best management?

a. Open drainage
b. Give metronidazole
c. Aspiration under ultrasound guided
d. Aspiration and insertion of pigtail drainage

204. Scenario of septic shock, patient complaining of sudden onset of fever, desquamation (palm and
soles) due to staphylococcus A, pyogenes?

a. Distributive shock
b. TSS (toxic shock syndrome)

205. Patient complaining ofpruritus at right around anus, no history of bleeding or pain or fever,
diagnosis is?

a. Anal fissure
b. Perianal abscess
c. Piles
d. Perianal fistula

206. Patient complaining of mid sigmoid colon cancer, the best management is?

a. Anterior resection
b. Resection and anastomosis as colorectal anastomosis
c. Abdominal perineal
d. Hartman procedure

207. The most common anomaly with infant complaining of gastroschisis is?

a. Intestinal atresia
b. Fallot triad
c. Esophageal tracheal fistula

208. Patient complaining of upper GI bleeding by upper GI endoscopy – mass after 35cm of insertion
of endoscope, what is the next for staging this mass?

a. Endoluminal ultra sound


b. Barium swallows
c. C.T abdominal
d. Small intestinal barium follows through

209. Patient complaining of gastric ulcer since 15 years and he develops upper GI bleeding last
month, what is the best management?

a. Endoscope and biopsy


b. Upper dose of Omeprazole
c. Selective vagatomy
d. Barium meal

210. Patient complaining of dyspepsia since 10 years he develops upper abdominal pain at lar 2
months by ultra sound – SCC, patient above the dose of omeprazole to overcome pain, patient also lost
weight due to fear to eat, what is the most important sign in those history for this patient?

a. Loss of weight
b. Development of upper abdominal pain
c. Upper the dose of omeprazole

211. By barium Swallow – feeling defect in the lower third oesophagus by upper GI – semilunar mass
with intact mucosa, diagnosis is?

a. Lieomyoma
b. Adenocarcinoma
c. Esophageal adenoma
d. Squamous cell carcinoma

212. 1st layer appear in endoluminal ultra sound of oesophagus is?

a. Mucosa
b. Submucosa
c. Muscler
d. Serosa
213. 50 years old female patient complaining of bilateral breast pain, what you do?

a. Mammography
b. Mastectomy
c. Follow up
d. Fine needle biopsy

214. Patient showing intrabiliaryradical dilatation by ultra sound with normal CBD, what is next?

a. ERCP
b. MRCP
c. C.T abdominal
d. PTC

215. Perineal opening with purulent discharge and cord like structure relating to anus, diagnosis is?

a. Anal abscess
b. Fissure
c. Fistula

216. Patient with stab left side chest between 9, 10, ribs anterior auxiliary line, patient is vitally
stable, what is next?

a. Observation and follow up


b. Laparotomy
c. Thoracotomy
d. Insertion of chyst tube

217. C.T, RTA with blunt trauma to right hypochondrium with tenderness, no air under diaphragm,
the most common injured is?

a. Liver
b. Stomach
c. Gallbladder
d. Duodenum

218. Patient 22 years old presented with pain at right iliac fossa with rebound tenderness WBC 12.5,
temp 38.2, diagnosis is?

a. Salpingitis
b. Appendicitis
c. UTI

219. RTA with no. 9, 10, 11, left ribs with tenderness at left hypochondrium and pain radiating to
back, the indication for surgical intervention if this patient?

a. Hypotension
b. Splenic hematoma
c. Other abdominal injury
220. Wound healing is affected by?

a. Monocytes
b. Collagen present at fibres

221. Origin of basal cell carcinoma is?

a. Pluripotential stem cell

222. Patient 70 years old with history of M.I last 2 weeks he develops abdominal pain since 1 day,
mostly due to?

a. Acute appendicitis
b. Mesenteric ischemia

223. 72 years old patient with ASA III and rectal prolaps. What is the best management?

a. Altemeier Procedure
b.
224. Most common case of recurrent epigastric hernia after repair is?

a. Missed defect

225. Patient with multiple areas of dilatation of pancreatic duct with pain, best management is?

a. Distal pancreatectomy
b. Lateral pancreaticojejunostomy
c. Whipple operation

226. 2 years old child bilateral undescended testis, you will search by?

a. Ultra sound
b. C.T
c. Laparoscopy

227. Commonest cause of haemophilia?

a. G.B stone
b. Liver trauma
c. Vascular disorders

228. Right upper limb on exercise and pallor on rising and edema on downward position treatment
is?
a. Cervical rib resection
b. Arterial grafting
c. Sympathectomy

229. Bilateral undescended testis, not inguinal or scrotal, best investigation for localization is?
a. MRI
b. CT
c. Ultra sound
d. Laparoscopic

230. 18 years old girl with right breast cyst aspiration revealed greenish fluid, what is the best
immediate step?

a. Cytologic examination of the fluid


b. Excision of the cyst
c. Antibiotics

231. Neck swelling in child positive transillumination ultra sound – multiple fluid filled cyst?

a. Thyroglosal
b. Bronchiole
c. Cystic hygroma

232. Neonate with round epigastric mass and projective vomiting, will treated by?

a. Ramstead procedure
b. Sistrunk
c. Ladd’s

233. Gastroschisis associated with?

a. Malrotation

234. Neonate with round epigastric mass, Projective vomiting resuscitated by?

a. Saline
b. R.L
c. Dextrouse

235. Patient with abscess in the gluteal region (perineal) transmitted to other side?

a. Perineal
b. Supralevator
c. Ischiorectal
d. Intersphicteric

236. Patient with acute pain, adult no other disease, pain with defecation and blood on towel?

a. Anal fissure

237. Female patient with 4 children with history of anterior fissure, most probably due to?

a. Vaginal Delivery
b. Constipation
c. Crohn’s

238. Pseudo membranous Colitis?

a. Vancomycin

239. Coloscopic view of Pseudo membranous?

a. Fibropurulem exudates

240. History of antibiotics + diarrhea?

a. Flagyl

241. Patient with colonic resection received ceffriaxone as prophylactic, developed abdominal
abscess?

a. No adequate coverage of anaerobe

242. Necrotizing facilities diagnosis?

a. From history
b. Blood culture positive
c. X-ray (air in soft tissue)
d. All of the above

243. Necrotizing fasciitis treatment?

a. Penicillin G
b. Vancomycin
c. Clindamycin 600-900 IV/8h
d. Imipenam + clindamycin + vancomycin + surgical debridement + oxygen therapy

244. Necrotizing fasciitis causative organism?

a. Staphylococcus aureus
b. Group Astreptpyogenes
c. Clostridalmyonerosis
d. Escherichia coli

245. Child with foreign body sole of foot with streaks of cellulites up to medial thigh, organism strep?

a. Lymphagitis

246. Infection like clostridium perfringens?

a. Clindamycine
247. 10 units of blood-die, patient with no. Pelvis

a. K above

248. Spleen – from picture CT?

a. 20 year tender left UQ +

249. Picture CT liver laceration

250. Picture CT. Spleen laceration

251. Degree of abdominal compartment syndrome that need surgical intervention?

a. 12-15 mmhg
b. 16-20 mmhg
c. 21-25 mmhg
d. >25 mmhg

252. Give GCS about 6-7, how to protect air way?

a. Intubation

253. No. Spine Lh, cutting of spinal cord, can be associated with?

a. Hypovelemic shock
b. Septic shock
c. Neurogenic

254. Mediastinal Emphysema, S.C emphysema?

a. Bronchoscopy
b. Bronchogram
c. Chest tube
d. C.T

255. Gun shot in chest, patient hemodynamic is stable, but cold clammy skin fever?

a. Hypoxie
b. Anxiety

256. Euthyroid patient after surgery hypotension fever, above HR?

a. Storm thyroid
b. Adrenal insufficiency

257. Moon face, buffalo hump, steroid hormone above?


a. Cortisol
b. ADH
c. Insulin

258. SIRS which is most indicator?

a. Temp > 38.5


b. WBC < 4000
c. Pacoz< 32

259. SIRS + infection + above creatinine + BUN BP 120/80?

a. Sepsis
b. Septic shock
c. Sever Sepsis

260. Most indicative to give more fluid?

a. SBP < 65
b. CVP < 15

261. Post operative – edema which hormone?

a. ADH

262. Flam Burn, die from?

a. Inhalation
b. Infection
c. Dehydration
d. Hypoalbuminemia

263. History of D.V.T 10 years back + melanoma, 2 years image like lipodermatosclerosis, cause?

a. Melatonic nodule
b. Hemosiderin

264. Graft in (48) after operation depends on?

a. Imbibition

265. Adrenocortical Insuffiency?

a. Conn syndrome
b. Cushing syndrome
c. Addison syndrome

266. Dysphogialusoria, caused by aberrant an artery, which of the following?


a. Left subclavian artery
b. Right subclavian artery
c. Right common carotid artery

267. FNA – follicular – hemithyroidectomy or lobectomy, during operation, frozen section was
carcinoma, what you do?

a. Total thyroidectomy
b. Total thyroidectomy + modified neck dissection

268. Thyroid, all investigation are normal except blood glucose was high?

a. Go for HR + morning insulin


b. Stand for glycosylated haemoglobin
c. Change in their diet and lifestyle

269. Spliegian hernia?

a. More in female
b. More in old age
c. Arise near acute line
d. Less likely strangulate

270. Patient come from meico with fever, pain RUQ, what you will find in investigation?

a. Positive amoebic serology


b. Positive echinochoal serology
c. high billirubin level

271. Female with retroareolar mass and bloody nipple discharge?

a. Duct ectasia
b. Fibrocystic disease
c. Duct Papiloma
d. Duct carcinoma

272. 20 years old male, father have a C A colon, what the most indicate risk for him?

a. CEA level
b. Colonoscopy
c. C.T abdominal

273. Patient with valvular heart disease have thromboembolism in leg popletial artery with good
distal run off, what to do?

a. Ultra sound
b. Excision
274. Patient with valvular heart disease have thromboembolism in leg in potential artery with good
distal run off, what to do?

a. Embolectomy with fogarty


b. Heparin
c. Aspirin

275. Scenario patient had laparotomy for I.O, you find terminal ileum firm rubbery with gray to
white serosa and adherent loops?

a. TB ileitis
b. Actinomycosis ileitis

276. Child, nonbilioius vomiting, which fluid used for resuscitation?

a. Dextrose 5%
b. Isonic saline
c. Ringer

277. Child with ITP, platelet 50,000. What to do?

a. Giving steroids
b. Splenectomy

278. Origin of BCC?

a. Keratiinising cell layer


b. Multipotent cell
c. Follicular structures

279. Origin of SCC?

a. Pluripotent cell
b. Totipotent cell
c. Keratinising cell

280. Dieulafoy lesion, the most common site in?

a. Close to pylorus
b. Close to cardia
c. Major curvature

281. Dieulafoy lesion, the best diagnosis?

a. Ultra sound
b. C.T
c. Endoscopy
d. Angiogram
282. Subselective embolization a dieylafoy lesion with?

a. Endoscopy
b. Angiogram
c. Guide ultrasound

283. Cause of painful anal condition?

a. Contaminated zone
b. Rich vazcularization
c. Many nerve endings in the region

284. Myeloproliferative disorder, causes?

a. Viral
b. Bacterial
c. Weak immunity
d. Genetic

285. Gas gangrene, prevention?

a. Vaccine anti clostridium


b. Remove foreign objects and dead tissue from wound

286. Gas gangrene, cause?

a. E. coli
b. Clostridium perfringens
c. Streptococcus

287. Incisional hernia, the best management?

a. Laparotomy with mesh repair


b. Autologous tissue stem cell
c. Laparoscopic with mesh repair

288. Hypertrophic pyloric stenosis treatment?

a. Kasai procedure
b. Ramstedt procedure
c. Puestow procedure
d. Ladd procedure

289. Patient with hyperthyroid, controlled on anti-thyroid drugs, but seek advice for fear of
recurrence and disfigurement, what to do?

a. Thyroidectomy
b. Radioactive iodine
290. Tisticular torsion is most frequent between?

a. 1-3 years of age


b. 12-18 years of age
c. 30-40 years of age

291. Scenario, 62 years old female patient, left iliac foss a pain, normal colonoscopy, few pus cells in
stool?

a. Crohn’s
b. Typhoid
c. Cancer
d. Bacillary

292. RTA with liver laceration but patient is stable, what to do?

a. Observed vital signs and HCT

293. Mediastinal mass with enlarged mediastinal LN and mass in upper left lobe of left lung, what to
do?

a. Chest x-ray mediastinoscopy


b. MRI
c. Lung biopsy
d. C.T

294. Most risk factor of breast cancer in young female her mother has breast cancer before?

a. Positive family history

295. Male retrosternal pain, relieved by food otherwise positive

a. Boerhaave syndrome
b. M.I
c. G.U

296. Female patient has infection on gentamycin end metronidazole, culture negative but CRP is
high, WBC is still high, what to do?

a. Change to imipenem
b. Stop the treatment
c. Continue on some drugs

297. Felon, the most common cause?


a. Streptococcus
b. Staphelococusaureus

298. Patient pregnant, 24 weeks came with acute onset of abdominal pain, fever and vomiting. She
states that pain woke her up at night, now pain is diffuse no vaginal bleeding, fetus exam okay,
guarding whole abdominal pelvic exam normal, WBC 15000, x-ray abdominal dilated loops. What is
the next step?

a. Ultra sound
b. C.T abdominal pelvic contrast
c. Exp. Laparotomy
d. Diagnostic laparoscopy

299. Parathyroid – psychiatric patient treatment?

a. I.V fluid (hyper Ca++)

300. Bilateral recurrent nephorolitiasis

a. Parathyriodectomy

301. Breast with free mammogram, history of sister with ovary cancer, mother with breast cancer
BRCA I, what the next?

a. Follow up
b. Genetic study BRCA I

302. Characinoid 4cm in tip of appendix?

a. Right hemicolectomy

303. Small pneumothorax, visible only in CT need?

a. Chest tube
b. Observation

304. Pneumothorax, visible with chest x-ray need?

a. Chest tube
b. Observation

305. Deviation mediastinal or trachea in?

a. Small hemothorax
b. Moderathemothorax
c. Massive hemothorax

306. RTA patient with open pneumothorax

a. Close the wound


b. Apply bandage taped on 3 sides
c. Leave it open
307. Patient 60 years old with occlusion of the profundafemoris artery, acute thrombosis of a
popliteal aneurysm, what you will do?

a. Amputation above the knee


b. Amputation below the knee

308. Patient with popliteal artery pressure <50 mmhg?

a. Above knee amputation


b. below knee amputation

309. Patient RTA trauma vascular injury, neurologic injury, open tibia # with posterior tibial nerve
disruption ischemia > 6 hours, what to do?

a. Above knee amputation


b. Below knee amputation

310. Ideal length stump in BKA (below knee amputation)?

a. 5 cm
b. 9 cm
c. 12-17 cm

311. Patient 78 years old, difficulty walking, hand weakness complaining of dysphagia and weight
loss endoscopy oesophagus was normal?

a. Achalasia
b. Squamous cell carcinoma
c. Amyotrophic Lateral Sclerosis (ALS)

312. Patient with dysphagia, gurgling, food particles?

a. Achalasia
b. Zenker’s diverticulum
c. Tumour

313. Diffuse oesophageal spasm?

a. Corksrew Oesophagus

314. GERD?

a. PH monitoring

315. Patient male with hematemesis?

a. Dieulafoy lesion

316. Gas under diaphgram (x-ray)?


a. Perforted D.U

317. Gastric Ulcer 15 years back investigation?

a. Esophagogastroduodenoscopy + multiple biopsy

318. Liver Laceration, first investigation?

a. C.T

319. Liver abscess, treatment?

a. Percuataneous drainage
b. Antibiotics

320. Patient 71 years old, feeling tired, weakness, abdominal pain, unconjugated bilirubin is high,
jaundice, diagnosis?

a. Porphyria
b. Gilbert Syndrome
c. Gauncher disease

321. Post Splenectomy, diagnosis for splenosis is?

a. Howell – jolly bodies is present


b. Howell – jolly bodies is absent

322. In blast injury, the most common organ cause of death?

a. Ears
b. Gastrointestinal Tract
c. Lung

323. In encephalopathy which is better?

a. Distal splenorenal shunt (DSRS)


b. Transjugular IntrahepaticPortosystemic Shunt (TIPS)

324. Obstructive jaundice?

a. Ultrasound
b. ERCP

325. Female patient typical of acute cholecystitis, investigation?

a. Ultrasound

326. Gallstone ileus, diagnosis?


a. X-ray

327. Bilateral of IHBD and normal CBD level of obstruction? (Intrahepatic biliary dilatation)?

a. Hepatic ducts

328. Pseudo cyst?

a. Gastrojejunostomy

329. Pseudo cyst <4 cm?

a. Wait and watch

330. ERCP – dilated pancreatic duct, in chronic pancreatitis (steatorrhea and DM), treatment?

a. Pancreaticojejonostomy (puestow)

331. Glucagonoma associated with?

a. Necrolytic migratory erythema

332. Spherocytosis for splenectomy, investigation?

a. Ultrasound GB – to be removes with spleen

333. Hodgkin, diagnosis?

a. L.N biopsy

334. Crohn disease all of the following are differential diagnosis except one?

a. Intestinal tuberculosis
b. Behcet disease
c. Familial adenomatous polyposis
d. Ulcerative colitis

335. Crohn disease after surgery(segmental resection), all of the following complication subsided
except one?

a. Erythema nodosum
b. Pyodermagangrenosum
c. Ankylosing spondylitis
d. Uveitis

336. Appendical, location most common is?

a. Retrocecal
b. Dorsomedial
c. Retroperitoneal

337. In appendicitis with score for C.T?

a. 1-3
b. 4-6
c 7 or above

338. Patient with Ogilvie syndrome, colonoscopy for decompression done but failed he went to
operation, during laparotomy the bowel found viable, what do you do?

a. Resection of the segment


b. Right Hemicolectomy
c. Place decompressivececostomy tube
d. Nothing to do

339. Patient was vaccinated but observed contamination and there is evaluation disease?

a. HbsAb -, Hbs Ag+, HbcAb-


b. HbsAb+, Hbs Ag+, HbcAb+
c. HbsAb-, Hbs Ag-, HbcAb+
d. HbsAb+, Hbs Ag-, Hbc Ag-

340. Colon polyps t2 – endoscopic, what is the next?

a. Laparoscopic resection
b. Endoscopic removed

341. Diverticulitis most common site in?

a. Ileum
b. Transverse colon
c. Sigmoid colon

342. Diverticulitis most common pain in?

a. RIF
b. LIF
c. Hypogastric

343. Diverticulitis the diagnosis confirmed with?

a. Barium enema
b. Colonoscopy
c. C. T

344. Diverticulitis stage III & IV management?


a. Primary anastomosis
b. Right hemicolectomy
c. Hartman Procedure

345. Prolapsed piles, treatment?

a. Surgery

346. Femoral Hernia?

a. Above and medial the public tubercle


b. Below and lateral the public tubercle
c. Above and lateral the patient
d. Below and medial the patient

347. Inguinal hernia?

a. Above and medial the patient


b. Below and lateral the patient
c. Above and lateral the patient
d. Below and medial the patient

348. What is the correct about Richter hernia?

a. It cause to intestinal obstruction


b. Contains of appendicitis
c. Not to affect bowels movements
d. There is rare mortality

349. Patient with inguinal hernia 2 years ago, now he came in ER with pain since last night, OIE :
irreducible hernia, he declare since last night no pass stool or flatus, diagnosis?

a. Richter hernia
b. Incarcerated hernia

350. Hernia medial and inferior to public tubercle and ring occlusion test negative?

a. Complete indirect

351. Metabolic derangement of gastric outlet obstruction

a. Metabolic Alkalosis

352. Patient underwent henioplasty a day after diagnosed to have gas gargrene the most important
treatment?

a. Surgical debridement and large dose of penicillin


353. Patient diagnosed to have pseudo membranous eolitis on IV Cephalosporin for 14 days,
colonoscopy showed yellowish flower best treatment by?

a. Pancolectomy with ileostomy


b. Shift to vancomycin

354. 70 years old patient with recurrent inguinal hernia with history of retropubic prostatectomy,
best treatment by?

a. Open hernioplasty
b. Open Herniorrhaphy
c. Lao TEP
d. Lap. TAPP

355. Which can compromise respiration during delivery to baby?

a. Cystic Higroma
b. Torticollis

356. Potassium level more in?

a. Blood
b. Saliva
c. Urine

357. Most common complication after hemorrhoidectomy?

a. Bleeding
b. Urine retention

358. Output from ileostomy must be kept under?

a. <1500
b. 500

359. Output from colostomy must be kept?

a. 500 - 700
b. 1500

360. Patient with septic shock, which HB volume considered for blood transfusion?

a. 6–7
b. 7–8
c. 8-9
d. 10

361. Unconjugated Billirubin?


a. Water insoluble
b. Water soluble

362. Conjugated billirubin?

a. Soluble in the fat


b. Water insoluble
c. Water soluble

363. Type of gastritis caused by H. Pylori?

a. Type A
b. Type B

364. Type of gastritis caused by pernicious anaemia?

a. Type A
b. Type B

365. Treatment of annular pancreas?

a. Duodenojejunostomy

366. Pancreatic Tumour 2cm with hypokalemia?

a. Vipoma

367. Patient underwent Lap. Chole with post-operative day are bile leak, best treatment?

a. ERCP and stenting

368. Best investigation to diagnosis CHPS?

a. Endoscopy in adult
b. Ultrasound in paediatrics

369. Ultrasound dilated intrahepatic duct with normal extrahepatic duct level of obstruction?

a. Hepatic duct junction

370. On the second day after abdominal aortic aneurysm repair, the patient passes grossly bloody
stool, the next step is?

a. Immediate exploratory laparotomy


b. Sigmoidoscopy
c. C.T with contrast

371. Female has mass in lower abdomen increase with standing and decrease with supin position,
best investigation?
a. C.T

372. Most common cause of haemophilia?

a. Iatrogenic
b. Trauma

373. Patient came from mexico with fever and abdominal pain and diarrhea for 10 days, C.T showed
hypoechoic mass subphrenic the investigation will be?

a. Echinococcal test
b. Amebic Test

374. Patient with previous cancer 5 years, ultrasound revealed mass at the upper pole of kidney next
step?

a. Guided biopsy
b. Lap investigation

375. Dirty wound and vaccinated 6 years back against tetanus?

a. Anti-tetanus serom
b. Anti-tetanus globulin

376. Patient with HCS, what’s findings by ultrasound, the most important and likely for surgery?

a. Size
b. Site
c. Content
d. Fatty liver

377. Picture of thyroglossal cyst 52 years, patient didn’t surgery, what would likely happen?

a. Secondary infection
b. No change

378. By endorectal ultrasound, how many layers detected?

a. 2
b. 3
c. 4
d. 5

379. Percentage of malignant potential in ulcerative colitis after 20 years?

a. 2%
b. 8%
c. 18%
380. AZATHIOPRINE in treatment of IBD?

a. Decrease leucocytes
b. Used instead corticosteroid
c. It is onset of action 6-12 weeks
d. Can be give IV

381. Treatment of desmoid tumour in patient with FAP?

a. Tamoxifen
b. Cortisone
c. Radiotheraphy
d. Methotrexate

382. Treatment of infected bullous dermatitis with crepitus under skin (Clostridium Perfringens)?

a. Clindamycin
b. Penicillin G

383. Patient with BP 120/30, creatinine and urea is high diagnosis?

a. SIRS
b. Sepsis
c. Severe Sepsis
d. Septic Shock

384. Liver divided anatomically based on?

a. Hepatic artery’s
b. Hepatic veins
c. Buliary ducts

385. Cause of cold extremities while oxygen is good?

a. Leirch syndrome (aortoiliac)

386. Gastroschisis associated with anomaly?

a. Intestinal Atresia

387. Dermatitis and Erythema occur in?

a. Glucagonoma
b. Vipoma

388. The most common site of Liomyoma?

a. Submucosal
389. The best indicator of adequate resuscitation in shock is?

a. Normal blood pressure


b. Normal pulse
c. Adequate urine output
d. Decrease lactate level

390. Basal cell carcinoma arise from pluripotent basal cell layer?

391. Initial maintenance fluid therapy for a child 10kg?

a. 1 cc/kg/h
b. 2 cc/kg/h
c. 3 cc/kg/h

392. Acute pancreatitis, CT revealed pancreatic tail cystic 3cm dilatation in pancreatic duct with
filling defect and papillary projection inside the cystic dilatation, what is the treatment?

a. Total pancreatectomy
b. Duodenopancreatectomy
c. Lateral pncreaticojejubostomy
d. ERCP and stenting

393. Female patient 30 years old with acute abdominal pain WBC 35000?

a. Acute pancreatitis

394. Male patient 30 years old with right hypochondrial swelling, tender, ultrasound revealed cyst
contain daughter cyst?

a. Mebendazol
b. Ultrasound guided aspiration
c. Tinidazol
d. PAIR and mebendazol

395. Child 15 years old underwent splenectomy after 15 days, paltelets was 50000, what to do?

a. Observation only
b. Re-exploration and search of accussor spleen
c. Radiotheraphy
d. Give corticosteroids

396. The most important initial management of suspected blunt myocardial injury is?

a. Electroencephalographic monitoring
b. Chest C.T
c. Assessment of cardiac enzymes
d. Insertion of pulmonary artery catrheter
e. Echocarfiogram

397. Acute abdominal pain and distension and the liver is not dull, what to do for him?

a. X-ray abdominal erect


b. X-ray abdominal supin
c. Chest x-ray erect
d. Chest x-ray supin

398. Most common cause of extrahepatic biliary dilatation is?

a. Pancreatic Ca
b. Choledocholitiasis

399. 23 years old female lactating and stopped lactate due to breast trauma,now she has 2 x 3 cm
swelling beside areas, when press give whitish discharge?

a. Cancer of the breast


b. Galactocel
c. Fibroadenoma
d. Fibrocystic disease

400. Mesenteric cyst arising from?

a. Chylomucosa
b. Chyloseroasa
c. Chylolymphatic

401. Myeofibrosis is?

a. Epiphysis cancer
b. Diaphysis cancer
c. Metaphysis cancer
d. Bone marrow cancer

402. I have a rogue ulcer at the side of my nose that seems to be getting larger diagnosis?

a. Rodent ulcer

403. Patient with rodent ulcer in the nose removed by elctrocautery, whats most common
complication?

a. Infection

404. The mortality rate of acute appendicitis is?

a. 4 / 100000
b. 4 / 10000
c. 4 / 10000000
405. 2 units of packed RBC, given in CVC, after 2 hours the patient become unconscious due to

a. lot complication of blood transfusion

406. The most commonest cause of liver transplantation?

a. Hepatitis
b. HCC
c. Biliary

407. Diabetic Patient presented to ER with confusion, sweeting, abdominal pain, clinically
dehydrated with hypotension, treatment?

a. Glucose
b. Insulin
c. Keton
d. Keto acid

408. 6 months boy was discovered by her mother to have perineal lump during the changing of
pampers right scrotum is empty, best investigation?

a. Ultrasound
b. Pelvic MRI
c. Aspiration biopsy

409. Which of the following normally not palpable?

a. Cervical node

410. Origin of cell basal cell carcinoma?

a. Pluripotential

411. Rodent ulcer safety margin?

a. 0.5 mm
b. 1 mm
c. 2 mm
d. 10 mm

412. Scenario of case with hematochezia colonoscopy is free, best investigation is?

a. Barium meal follows through


b. Capsule endoscopy
c. C.T
d. MRI

413. Cause of prolonged QT interval, with U wave?


a. Hypokalemia
b. Hyperkalemia
c. Hypernatremia
d. Hypocalcaemia

414.

415. (Photo with swelling) beside the lateral abed of eye brow, how can you differentiate the
swelling from sebaceous cyst?

a. Punction
b. Consistency
c. Translucency

416. Scenario with photo of pendulus abdomen for multipara lady 4 c/s in the last 8 years she is
complaining of lower abdomen swelling disappear when lay down and on standing it is with pain, best
investigation for diagnosis?

a. Ultrasound
b. C.T
c. Laparoscopy

417. Baby with gastroschisis is liable to have?

a. Situs Inversus total is


b. Gut malrotation
c. Intestinal Atresia
d. Cardiac anomalies

418. Patient has Esophgeal cancer invadethe trachea, best investigation?

a. Upper GI
b. Bronchoscopy
c. Chest

419. Regarding GI secretion which one have more potassium content?

a. Bile
b. Saliva
c. Pancreatic Juice
d. Intestinal

420. Scenario with photo patient have black foot and leg pulslessness femoral artery were occluded
endovascular done and the superficial femoral only become patent and they decide to amputation,
what will be the level of amputation?

a. Below the knee


b. Above the knee
c. Through the knee
d. Disarticulation

421. Scenario of patient had alcohol induced pancreatic and came back after two weeks with mild
epigastric pain and ultrasound revealed 5.7cm per pancreatic fluid collection, what is the best
management?

a. IV anti-biotic + analgesics
b. Sonar guided evacuation of the collection
c. No surgical intervention needed

422. Scenario for old man underwent cholectestomy 15 years old and no jaunded and clay stool,
ultrasound revealed dilated intrahepatic radicals and the CBD is normal in caliper. What is the best
investigation to evaluate the ob:

a. ERCP
b. MRI
c. PTC
d. CT

423. Healing failure of a wound in the medial aspect of the lower one third of right leg producing
ulcer measuring 4.5cm with sloping edges and floor shows granulation tissue?

a. Arterial ulcer
b. Venous ulcer
c. Tropic ulcer

424. Most fatal in old age?

a. Intussusception
b. Dieualafoy lesion
c. Colon cancer
d. Aorto-enteric fistula

425. Scenario Scenrio of patient underwent removal of infected cyst and dressing 2 times per day
with saline after few days exfoliative rash in the hands fever 40c, hypotension, the wound become red
nd hot surrounded by inflamed skin and dark urine, this is due to?

a. NecrotiziingFascitis
b. Cellulitis
c. Drug eruption
d. Septic shock

426. Male patient 46 years old, undergoes a distal gastrectomy for a small tumour in the gastric
antrum that is biopsy proven to be adenocarcinoma, the specimen is sent for pathology reveals
microscopic evidence of tumour at the margins, which of the following most accurately describes this
resection?

a. R0
b. R1
c. R2
d. R3

427. Capacitive coupling in lap chole is?

a. Transfer of electric energy between matalicinstrumement


b. Insultion
c. Arabic

428. Photo for case with left arm high tension wire injury what to be done to save arm forearm?

a. Fasciotomy
b. Escharotomy
c. Wound Debridgement

429. Photo x-ray for old man with 4 days no stool or flatus?

a. Sigmoid volvulus
b. Colitis
c. Mesenetric Ischemia

430. Olds man with pneumaturia and recurrent UTI commonest cause for ciloversical fistula is?

a. Cancer Colon
b. Crohn Disease
c. Ulcerative Colitis
d. Diverticulitis

431. Patient had laparotomy for IQ, you find terminal ileum firm , rubbery with garish white serosa
and adherent loops?

a. TB ileitis
b. Actinomyces ileitis
c. Crohn’s disease

432. Child, non bilioius vomiting, which fluid used for resuscitation?

a. Dextrose 5%
b. Isotonic Saline
c. Ringer

433. Scenario 62 years old female patient, left iliac fossa pain, normal colonoscopy, few puss cell in
stool?

a. Crohn
b. Typhoid
c. Cancer
d. Bacillary dysentery
434. Mediastinal mass with enlarged mediastinal LN and mass in upper left lobe of left lung, what to
do?

a. CX medistinoscope
b. MRI
c. Lung biopsy
d. CT

435. Most risk factor of breast cancer in young female her mother has breast cancer before?

a. Positive FH

436. Primary biliary cirrhosis associated with?

a. Vipoma
b. Insulinoma
c. CREST
d. Glaucoma

437. Occur in chronic ulcerative colitis they are significant indicators of carcinoma elsewhere, which
one from the following?

a. Angiodysplasia
b. Dieulafoy lesion
c. DALM
d. Gardner syndrome

438. Patient with spherocytosis referred to surgery because of splenomegaly and became candidate
for splenectomy, what investigation the surgeon order before going to surgery?

a. Osmotic fragility test


b. Scintigraphy
c. Serum billirubin

439. A 56 years old man suffered from intermittent claudication for 5 years, he has recently
developed cramping abdominal pain that is made worse by eating, he has history of a 15-16 weights
less, what is the most likely diagnosis?

a. Chronic intestinal ischemia (intestinal anguinal)


b. Chronic Cholecystitis
c. Peptic Ulcer
d. Abdominal aortic aneurysm

440. Patient with previous c/s, for laparoscopic H repair, what is the common complication?

a. Recurrence
b. Adhesion
c. Filed defect
441. 35 years old known case gall stone came with mild jaundice and right upper quadrant mass,
what you will do with her?

a. Ultrasound
b. C.T
c. MRCP
d. ERCP

442. How to differentiate between direct and indirect inguinal hernia?

a. Internal ring
b. External ring
c. Relation to inferior epigastric artery
d. Relation to vas

443. %0 years old post M.I 5 days ago presented with calf swelling which is tender and femoral pulse
is palpable and popliteal is not palpable, what is diagnosis?

a. Embolus
b. Thrombus
c. Rupture popliteal aneurysm

444. Mallory Weiss syndrome diagnosis by?

a. Endoscopy
b. Reach greater curvature

445. Patient with mild third oesophagus carcinoma 3cm reach muscles and there is celiac lymph nod,
what is the treatment?

a. Esophagostomy
b. Chemoradiotheraphy
c. Stent

446. Trauma with fracture to shaft of femur treatment?

a. Closed interloking nail


b. Opened interloking nail

447. Old age marasmus feature of marasmus feature of hypovolemia?

a. Sunken eyes
b. Dilated vein
c. Prominent bone of case
d. Shrunken eyes

448. Patient on TPN, develop acrodermatitis?


a. Zinc deficiency
b. Linolenic acid deficiency

449. Patient on TPN, on blood transfusion develop comatose, cause?

a. Electrolyte imbalance

450. Post splenectomy platelet is 52000, splenectomy done 2 weeks ago, what you will do?

a. Cortizone
b. Observation
c. Reoperation to remove splenunculi

451. 6 years old had wire injury, you will give?

a. Toxoid
b. Immunoglobulin
c. Both

452. GERD – esophageal healing lead to ?

a. Short esophagus
b. Barettesophagus

453. Wound stay in its primary inflammation until?

a. Eschar formation

454. Big toe has fury whitish lesion tender scripting showed human papilloma virus diagnosis?

a. Verruca Vulgaris

455. Patient with cervical L.N enlarged, during thyroidectomy no palpable lesion in the thyroid, what
is your plan?

a. Total thyroidectomy and modified neck dissection

456. Patient underwent sigmoid colectomy for characinoid tumour 2 years after he found to have
2cm lesion in the liver, treatment will be?

a. Surgical resection
b. Radiofrequency
c. Neoadjovent with imatinib

457. Diagnosis of Insulinoma

a. Elevated C peptid and pro-insulin


b. Fasting insulin to glucose < 50 and fasting insulin > 24
458. Patient with T1 lesion 3cm from and anal verge and no LNs, best treatment?

a. Transanal excision
b. Abdomino perineal

459. Diagnosis of Vipoma?

a. Fasting VIP plasma level and CT and somatostatinreceptor


460. Diagnosis of glucagoma

a. Fasting glucagon level 500 to 1000

461. Patient underwent UGI endoscopy and felt sever retosternal chest pain, CXR was normal, what
confirmtory test to be done?

a. Water soluble contrast study

462. Most common complication of gastroschisis?

a. Sepsis

463. Anomalies associated with gastroschisis?

a. Intestinal atresia

464. Anomalies associated with amphalocel?

a. Cardiac anomalies 50% and gut rotation

465. Complication with omphalocele?

a. Sepsis

466. Patient fracture femor developed dyspnea and petechial haemorrhage?

a. Fat embolism

467. Patient with blast injury has Pneumotorax, burn, fracture tibia and fibula and injury by
shrapnels, which injury of them is a quaternary blast injury?

a. Shrapnels
b. Burn
c. Pneumotorax
d. Fractures

468. Treatment of muscle cramps due to hypocalcaemia

a. IV calcium gluconate 10%


469. ECG show T wave flat in?

a. Hypokalemia

470. ECG show T wave peaked in?

a. Hyperkalemia

471. Mobile mass in young female breast 17 years?

a. Fibroadenoma

472. Bloody nipple discharge?

a. Duct papilloma

473. Treatment of mobile mass with tongue protrusion (thyroglosal cyst)?

a. Sistrunk procedure

474. Long case dark urine?

a. Toxic shock syndrome

475. Equation of osmolarity

a. 2Na + glucose / 18 + BUN / 2.8

476. Long case colon disease characteristic features (continues from rectum to ascendant colon)?

a. Ulcerative colitis

477. Long case diagnosis gall stone ileus, common site of impaction?

a. Terminal ileum

478. Case abdominal pain + watery diarrhea?

a. Vipoma

479. Long case of pseudo pancreatic cyst and is chronic TH?

a. Cyst Gastrostomy

480. Diagnosis of Hodgkin lymphoma?

a. Lymph node biopsy

481. D. pl positive if?


a. 10cc blood, 100000 RBC/mm3, 500 WBC/mm3, bile, bacteria of food particle, amylase
>175, bilirubin>0.01

482. Pancreatic injury imaging for diagnosis?

a. C.T

483. Most common cause of haemophilia

a. Iatrogenic

484. Treatment of pseudo membranous colitis

a. Metronidazol or Vancomycine

485. Diagnosis of pseudomembranous colitis

a. Colonoscopy and elisa

486. Post operative major surgery unilateral leg swelling and dyspnoea?

a. Pulmonary embolism

487. Diverticulosis if a symptomatic?

a. No treatment just conservative

488. Common cause of pyogenic liver abscess?

a. E-coli

489. Main cause of inguinal hernia in old patient?

a. BPH

490. Best investigation of anal fistula?

a. MRI
b. Colonoscopy

491. Post eating 15 minutes abdominal pain?

a. GU
b. Ischimic colitis

492. Post splenectomy immunization every?

a. 1 year
b. 3 years
c. 5 years
d. 10 years

493. Healing by fibrosis cause?

a. Short oesophagus
b. Barett oesophagus

494. TIA, what do you do?

a. Carotid duplex
b. Angio
c. MRI
d. CT

495. Transvers colon cancer, which artery should be liagated?

a. Middle colic artery


b. Middle colic artery + right colic

496. Femoral embolectomy?

a. Popliteal pulse is felt


b. Rec symptoms
c. Re. Embolus
d. DVT

497. Characteristic of somatostatinomaare?

a. Mild diabetic, skin rash, glossitis


b. Ulcer diathesis, diarrhea
c. Mild diabetics, diarrhea gall stone
d. Diarrhea, hypokalemia, hypochloremia

498. Mild diabetes, skin rash and glossitis are features of?

a. Somatostatinoma
b. Gastrinoma
c. Glucagonoma
d. Insulinoma

499. Male patient with inguinal scrotal swelling suspecting indirect inguinal hernia?

a. Vas deferens cannot be felt


b. Transluminated
c. Need for emergency surgery
500. Female patient complaining of pus discharge from perineal sinus at 11 o’clock, she was
complaining of the same since 3 months?

a. Colon cancer
b. Infection
c. Perineal crypts
d. Ulcerative colitis
e. Crohn’s disease

501. One of the patient safety requirement, identify by two identifiers?

a. Age and his history


b. Kind of his work and sex
c. Full name and date of birth or medical identification
d. Level his culture and country

502. One of the patient privacy ethics which of the following is correct?

a. Allow to discuss his medical data with others


b. Not to discuss his medical data with others
c. Exceptional circumstances where there is considerable harm to other should disclose
partially patient information

503. There are circumstances where a health professional disclosed without consent about principal
of confidentiality. Which one of the following is correct?

a. Disclosure required by law


b. Disclosure in the interest of the patient or other people
c. Disclosure in the public interest
d. All of the above

504. In principle of confidence there are two methods?

A - Disclosure with consent


B. - Disclosure without consent

In which one where confidential information is sheared between team health professionals
a. in A
b. in B
c. in both
d. neither one
e.
505. Patient 35 years old, he has asymptomatic popliteal aneurysm, require?

a. Treatment
b. Observation
c. Excision

506. Glioplastoma it arises from the malignant of?


a. Ewing’s sarcoma
b. Astrocytoma
c. Osteoid osteoma

507. In von will brand disease which one is normal?

a. VIII
b. Bleeding time
c. PTT
d. PT

508. In haemophilia, which one is normal?

a. Bleeding time
b. PTT
c. VIII
d. PT

509. If a health professional decides to disclose confidential information to a third party either. What
should they tell to them?

a. Disclose pertinent information to protect patients or others from harm


b. For protection of patients
c. For protection of health professional

510. Patient during anaesthesia he needs transfusion, after blood transfusion happened hypotension
and sudden haemorrhage, attributable to?

a. He was given other blood group


b. Patient suffered polycythemiacera
c. Hymollytic transfusion reaction
d. Loss of VIII factor

511. Patient RTA alert, conscious, he had haemorrhage he needs blood transfusion, after blood
transfusion, happened hypotension, fever and chills, attributable to?

a. He was given other blood group


b. Patient suffered of polycythemiavera
c. Hymollytic transfusion reaction
d. Loss of VIIII factor

512. The test specific for transfusion reaction?

a. Free plasma haemoglobin


b. INR
c. DIC
d. Troponin
513. Stored plasma is deficient in:

a. Factor VII and VIII


b. Factor II and VI
c. Factor VII and IX
d. Factor V and VIII

514. Meningocele and meningomyelocele can be distinguished by?

a. Ruptive of lesion
b. Appearance
c. Activity in lower limb
d. Gestational age

515. Sever liver cirrhosis he has hematemesis due to?

a. Deficiency factor VIII and Ca++


b. Deficiency factor II and Ca++
c. Deficiency factor III and Ca++
d. Deficiency factor VII and Ca++

516. Exclusive in the extrinsic coagulation due to?

a. Deficiency factor VIII


b. Deficiency factor V
c. Deficiency factor II and XII
d. Deficiency factor VII

517. Patient was given a few days ago TPN the nurse stopped TPN and she start to give him blood
transfusion but the patient entered a coma due to?

a. Hyperglycaemia
b. Electrolyte imbalance
c. Hypopotasimia
d. Hypoglycaemia

518. Patient was given a few days ago TPN he is with high output fistula, suddenly patient entered
coma, due to?

a. Hypoglycaemia
b. Electrolyte imbalance
c. Hyperglycaemia
d. All of the above

519. Patient with massive trunkal burn the way for emergent resuscitation?

a. Unburned peripheral limb


b. Venus cut down
c. Central venus line
520. What is the least tumour cause gynecomastia
a. Colon
b. Lung
c. Prostate
d. Gall bladder
e. Festis
f. Renal
g. Bone
521. What is least likely acute surgical abdomen?

a. Thalassemia
b. DKA
c. Henochscholeinpurpura
d. Lead poisoning

522. Hernia contain Micheal diverticulum?

a. Spigelian
b. Litters
c. Rechter
d. Pantaloon

523. Patient came to ER with stab wound in the neck, c/o S.O.B and bublling from wound, chest x-ray
showed mediastinum air. What is diagnosis?

a. Rupture trachea
b. Rupture oesophagus
c. Traumatic pneumothorax
d. Entrapped external air

524. Old patient with COPD and IHD, ASA grad 3 comes with rectal prolabse, O/E 7 cm rectal from
anus, what is the best operation?

a. Altemers Op.-ASA 3-4


b. Open Method
c. Closed method
d. closed anal circle – ASA 4-5

525. Patient with 10 years ago of laparotomy for perforated ulcer, now planned for lab. Choly, the
way for 1st trocar insertion?

a. Optical trocar
b. Closed method with needle
c. Ultrasound during operation
d. Hassan method

526. Patient came with perforated Michael diverticulum that involved the cecum, during operation
1.5 litters of puss in peritoneum, what do you do?
a. Right hemicolectomy
b. Resection and anastomosis
c. Iliotransvers with ileostomy

527. Paget’s disease of bone (false)?

a. All patients treated with calcitonine


b. Much more in younger than adult
c. Cause bone pain
d. Diagnosis with alkaline phosphatise with bisphosphonates

528. Patient with neck swelling not move with swallowing or protrude tongue

a. Bronchial cyst
b. Dermoid cyst
c. Thyroglosal cyst

529. Neonate 16 days, 31 weeks, gestation and 1400 kg at birth, came with abdominal distension,
vomiting with bile. What is diagnosis?

a. Intussusception
b. Jejunal atresia

530. Patient with portal hypertension came to the ER with episode of hematemesis, after
rehydration done?

a. Endoscopic sclerotherapy
b. Ballon
c. Open repair

531. RUQ pain ultrasound showed 8 cm hypoecoic mass in segment and VIII with another small mass
also hypoecoic and capsulated?

a. Serology(elias) for Echinococcusgranulosus


b. Open drainage
c. Hepatic Transplant
d. Resection
e. Percutaneous drainage

532. Indication of hepatic transplantation?

a. Active hepatitis
b. Cirrhosis
c. HCC

533. Pancreatic necrosis?

a. Necroctomy
b. Drainage
c. Pancreatictomy

534. Patient with portal hypertension?

a. Portal caval shunt


b. Spleeno renal shunt

535. Neoadjuvanttheraphy means?

a. Chemotherapy followed surgery


b. Surgery followed chemotherapy
c. Radiation therapy

536. Best fluid for rehydration shock?

a. R.L for type I and II


b. Blood for type III and IV
c. Dextrose 5% saline

537. 5cm characinoid tumour in the tip of appendix?

a. Appendectomy
b. Right hemicolectomy

538. Psychiatric patient with signs and symptoms of Ca++?

a. Parathyroidectomy
b. Stop psychiatric

539. Primary hyperparathyroidism in a woman with depression is suggested by elevations in


parathyroid hormone level and which other finding?

a. Hypercalcaemia
b. Elevated serum albumin
c. Hypocalcaemia
d. Vitamin D deficiency

540. ECG showed prolonged QT and U wave?

a. Below K+
b. Below Ca++
c. Below Na+
d. Below Mg++

541. Rib # on RTA is least likely to injury?

a. Trachea
b. Oesophagus
542. Patient stabbed in the 8th inter costal with normal vital signs and C x R is clear and
hemodynamic stable?

a. Observation with vital signs


b. Laparoscopic examination
c. Chest tube insertion
d. Thoractomy

543. Patient with sigmoid polyps and there is H/O total proctocolectomy to his sister 23 years and
brother 25 years polypotomy was done and found non malignant, what you will do?

a. Repetsigmoidescopy after 1 year


b. Repet colonoscopy after 1 year

544. Pancreatic cyst aspiration and cystology found serus glycogen and gelatine fluid?

a. Pseudocyst
b. Serous cyst
c. Gelatinous cyst

545. Sentinel lymph node biopsy for?

a. 1cm impalpable L.N


b. 1cm palpable L.N
c. Fixed L.N
d. Mobile L.N

546. Common type of breast cancer?

a. Lobular
b. Ductal

547. Patient with H/O dyspepsia 5 years and symptoms of GERD, endoscopy reveals achalasia, what
will you do?

a. Endoscopic balloon dilatation


b. Surgery

548. Patient with symptoms of Hypercalcaemia, what will you do?

a. Vehydreation
b. Para thyroidectomy

549. Patient with chronic duodenal ulcer come to ER with severe abdominal pain on laparotomy
found perforated duodenal ulcer and peritonitis puss 1.5 litters, what is right?

a. Close with omental patch


b. Artrectomy
550. Chronic liver disease, billirubin 2.5 and INR 1.7. What is child pugh score, albumin 1.5 and mild
ascites?

a. 7
b. 9
c. 11
d. 13

551. During open appendectomy the ileum with whitish patch (T.B ileum).What is the right?

a. Appendectomy and anti TB therapy 6-9 months

552. Hashimotos thyroiditis, what is the right?

a. Thyroid hormone replacement if no improvement – surgery and hormone replacement

553. Highly intestinalfistula PH 7.56 . What is right?

a. Via cutaneous

554. CBD injury, what you will do?

a. T-tube

555. Stable patient after stab wound in 8th inter costal space, what you will do?

a. Observation
b. Laparoscopy examination

556. Treatment of testicular torsion of 10 days duration?

a. Orchidectomy and fixing up of other testis

557. Treatment of primary malignant melanoma of skin?

a. Radiotherapy

558. Repair of choice in a small child with bilateral inguinal hernia?

a. Right ligation of the sac

559. Treatment of choice for cetosarcoma phyllodes

a. Simple mastectomy

560 It is not clear in the acute appendicitis in children?

a. Can cause intestinal obstruction


561. Inhalation injury in burns patients is not true?

a. Bronchioles and alveoli could burn from hot smoke

562. In ischemia leg?

a. Nerves is the first structure to be damaged symptoms is numbness, light tough is the
first sensation to loss

563. Thyroid cancer associated with?

a. Euthyroid

564. 50 years old female patient noticed firm neck swelling with palpitation and loss of weight?

a. Toxic nodular goitre

565. Gastric lavage is contraindicated in?

a. Cleaning solution

566. Conscious with multiple trauma patient your priority?

a. Assess airway breathing and BL.P

567. 30 years old man with long history of crohn’s disease and indication of surgery is?

a. Intestinal obstruction

568. Best investigation to visualise the cystic breast mass?

a. Ultra sound

569. The bilateral breast mass?

a. Lobular carcinoma

570. Factor not associated with increased risk of breast cancer?

a. Menopause before 40 years old

571. Factor which determine recurrence of breast cancer?

a. Number of lymph node

572. Clear aspirated fluid from breast cyst will be?

a. Send to cytology
573. 32 years old healthy male presented with symptomatic varicosity of long saphenous vein,
duplex ultrasound showed sever saphenous femoral efflux, normal deep venous system competent
perforation?

a. Stripping of the long saphenous in the thigh only

574. Non operative treatment of inoperable hypernephroma?

a. Interleukin II

575. In unresectable Wilm’s tumour before tumour removal we should give?

a. Chemotherapy
b. Radiotherapy
c. Both a and b

576. Surgery in cancer colon is?

a. Curative
b.
577. Patient had colectomy colonoscopy follow up should be done at ?

a. 6 months

578. Thyroid cancer associated with?

a. Euthroid

579. Most common complication of acute pancreatitis?

a. Pseudocust

580. Best diagnosis tool in acute diverticulitis?

a. C.T

581. Surgical procedure to treat a chronic pancreatic pseudocyst?

a. Internal drainage to the gastro intestinal tract

582. Uncomplicated meconium ileus is best treated by?

a. N. Acetyl cysteine

583. 70 years old male presented with cancer rectum 3cm from anal vergo?

a. Abdomino perineal resection


584. The best operation for varicose vein?

a. Diversion or ligation at site of communication from deep to superficial system

585. The best operative treatment for B.P.H?

a. Transurethral resection

586 Localised carcinoma of the prostate is best treated by?

a. Radical prostatectomy

587. Posterior urethral valve is best treated?

a. Transurethral resection

588. Open fracture should be treated with?

a. Debridement

589. Anaerobic infection?

a. Clostridia
b. Actinomycosis
c. Bacteroids
d. Treponema

590. Coagulase produced by?

a. Staphylococcus auress

591. E coli secrete?

a. Endotoxins

592. E coli?

a. Gram negative bacilli


b. Enterotoxins – diarrhea
c. Enteroinvasive – dysentery
d. Enterohemoragic – hemorrhagic syndrome

593. 50 years ild female with breast cancer and elevate Ca125, so elevation is due to?

a. Associated with ovarian cancer

594. Most common cause of nipple discharge in non lacting women

a. Fibrocystic disease with duct ectasia


595. 10 years with scrotal swelling decrease flow on Doppler study?

a. Testicular torsion

596. Large uncomplicated Pneumotorax?

a. Trancheal Deviation

597. Cancer colon with stage III give the chemotherapy?

a. As soon as possible

598. Variceal bleeding secondary to portal vein thrombosis is best treated by?

a. Gastroesophageal davascularisation

599. Operative procedure for correction ofa choledochal cyst?

a. Resection of the cyst and Roux-en-y hepatoieiunostomy

600. Choledocholithiasis in a patient who recently had cholecystectomy is best treated by?

a. Endoscopic Sphinctemotomy and stone extraction

601. Ramstedt’s operation is done for?

a. Congenital pyloric stenosis

602. The acronym quarter stand for?

a. Quadrantectomy
b. Axillary dissection
c. Radiotheraphy

603. Most common late complication of heller’s operation of achalasia?

a. Reflux esophagitis

604. The best treatment for ulcerative carcinoma at cardiac?

a. Surgery

605. Truncal vagatomy for duodenal ulcer should be combined with?

a. Pyloroplasty
b. Gastrojejoostomy

606. Most important complication and cause of death in organ transplantation?


a. Infection

607. The following disease is not frequently associated with pyogenic liver abscess?

a. Urinary tract infection

608. Greatest risk of developing chronic hepatitis and cirrhosis occurs after?

a. Hepatitis B infection

609. The following liver tumour is often associated with oral contraceptive agents?

a. Liver cell adenomas

610. The key point of venous ulceration?

a. Incompetent valves causing high venous pressure

611. The following muscles are not the rotator cuff?

a. Deltoid

612. Child has tracheoesophageal fistula we can’t use?

a. Insertion of chest tube

613. Old patient with jaundice, gall bladder is palpable the most likely cause?

a. Cancer of the pancreases head

614. Not a complications of excision of abdominal aortic aneurysm?

a. Hepatic failure

615. Most important cause of hand infection?

a. Trauma

616. What is the most common intra operative bladder tumour?

a. Foley catheter

617. What are the ABCD signs of melanoma?

a. Asymmetric
b. Border
c. Colour variation
d. Diameter > 0.6cm and dark colour
618. Carcinoide triade seen with carcinoid syndrome FDR?

a. Flushing
b. Diarrhea
c. Right sided heart failure

619. Boa’s sign?

a. Right subscapular pain resulting from cholelethiasis

620. Prominent collateral circulation seen in portal hypertension ?

a. Inferior hemorrhoidal vein to internal iliac vein


b. Esophageal varices
c. Hemorrhoids
d. Patent umbilical vein
e. Caput medusa
f. Retroperitoneal vein via lumbar tributaries

621. Artery bleed in duodenal ulcer?

a. Gastroduodenal artery

622. Name of L.N between pectoralis major in minor?

a. Rotter’s Lymph node

623. 25 years old man case of L.N mass?

a. H.L (Hodgkin Lymphoma)

624. Carbuncle is?

a. Infection of hair follicles

625. Most common site for mixed tumour ?

a. Parotid

626. The most indication for renal transplant?

a. End stage glumerulo pyelonephtitis

627. Septic shock mortality rate?

a. 30%

628. Appendicitis most diagnostic?


a. Tender right lower quadrant with rebound

629. Patient known to have gall stones presented with central abdominal pain and bruising in the
flanks?

a. Acute pancreatitis

630. Supracondylar fracture patient presented with sweeling and cyanosis of finger after plaster?

a. Removal entir of all splint

631. Adduction hip and internal rotation in fixed position will be?

a. Posterior dislocation of hip

632. Treatment of testicular torsion of 10 days duration?

a. Orchidectomy and fixing up of other testis

633. Child with imperforated anus, the most useful diagnostic procedure?

a. Plain x-ray of abdomen with child inverted position

634. Laparoscopy not use in?

a. Intestinal obstruction

635. Patient with fluctunt redness of finger pulp?

a. Felon

636. RTA with urethral bleeding step management?

a. Stabilize the pelvis

637. Progressive post laparoscopic cholecystectomy, the most appropriate investigation?

a. ERCP

638. Thyroid cancer that secrete calcitonine?

a. Medullary

639. The most cost, effective and quick investigation for breast lump is?

a. FNA

640. Epidemilogically fibroadenoma is commonest in?


a. 2nd and 3rd decade of life

641. Felon is?

A. Infection in a closed area in the pulp of distal of finger

642. Anterior mediastinal mass the most common case?

a. Thymoma

643. an 80 years old gentleman presented to ER with decrease urine stream, hesistancy, urgency,
frequency, cause?

a. Benign prostatic hypertrophy

644. Paracetamol toxicity in ER?

a. N. Acetyl cystine. IV infusion 150mg in 200 ml dextrose 5% over 15mins.

645. Most a suggestive of acute pancreatitis?

a. Sitting and leaning forward

646. In lumbar disc prolapsed at L4-L5 patient will have?

a. Weakness of dorsiflexion of foot

647. Best treatment for tension pneumothorax and patient is distressed?

a. Aspiration of air by needle

648. Best method for temporary control bleeding?

a. Direct finger pressure

649. Premature ejaculation?

a. Most common in young men


b. Most common sexual disorder in men below 40 years
c. Sexual therapy must involve both partners

650. Indication of tracheotomy not includes?

a. Left recurrent nerve cut

651. Not suggestive of acute appendicitis?/

a. Sitting and leaning forward


652. Child fall on outstretched hand and flexed elbow, examination show no radial pulse, best
management?

a. Open reduction

653. Patient suspected to have brain abscess the most important question in history?

a. Frontal sinusitis

654. 20 years old male presented with volar wrist injury with median nerve involvement the clinical
picture?

a. Inability to oppose thumb towards fingers

655. Indication for valve replacement in infective endocarditic not include?

a. Viral endocarditis

656. A cervical L.N is found to be a replaced by a well differentiated thyroid tissue at operation no
palpable lesion in the thyroid gland the operation of choice?

a. Total throidectomy and modified dissection

657. In a conscious multiple trauma patients the priorities are?

a. To secure air entry, breathing then B.L Pr.

658. A 20 years old patient had deep laceration in his right wrist the complication will be?

a. Claw hand

659. The following mostly occurs in a patient with intracranial abscess?

a. Vomiting

660. Percentage of reinfection for patient undergoing non cardiac surgery?

a. 15% , 3 months after the infarct

661. Complication of colostomy not includes?

a. Excoriation of skin

662. A 82 years old patient with acute urinary retention the management?

a. To insert a foley’s catheter then send the patient home to come back in the clinic

663. In peritonitis?
a. The patient rolls over with agony pain

664. 80 years old man presented with dull aching pain there is high area and creatinine, ultra sound
show bilateral hydronephrosis, diagnosis?

a. Prostatic enlargement

665. Intestinal Obstruction it is not true?

a. Always requires surgery

666. 70 years old women show M.I two years ago, there is abdominal pain and diarrhea with passage
of blood x-ray show no fluid level , serum amylase slightly elevated, mild fever diagnosis
?
a. Ischemic colitis

667. A 5 months old baby presented to ER with sudden abdominal pain and vomiting, pain lasts 2-3
minutes with interval of 10-15 minutes in between, the most likely diagnosis?

a. Intussuscetion

668. 16 months old child crying on and off for about 24 hours then pass currently gelly stool, most
diagnosis?

a. Intussusception

669. 55 years old with central abdominal pain radiating to the back, diagnosis?

a. Acute pancreatitis

670. On the 6th post operative of colostomy, a 52 years old has fever and diarrhea, diagnosis?

a. Pelvic Abscess

671. A 40 years old female has central neck swelling, moving with swallowing the mass is hard with
past history of dysphagia, management?

a. Admit the patient as a possible cancer thyroid and manage accordingly

672. A 20 years old man with a deep laceration on the anterior surface of the wrist median nerve
injury?

a. Inability to oppose the thumb to the other fingers

673. 30 years old man has pelvic fracture due to the blunt trauma; there is disruption of the
membranous urethra?

a. Suprapubic catheter
674. I.V.P for a male indicate shadow

a. Uric Acid stone

675. 45 years old lady present with bloody nipple discharge

a. Ductal papiloma

676. Complication of Laparoscopic Cholecystectomy not include

a. Ascitis

677. Post Lap-chole present with progressive jaundice the appropriate investigation

a. ERCP

678. Most common cause of surgical intervention in Inflammatory Bowel Disease (IBD)

a. Intestinal obstruction

679. Patient with history of non calcular cholecystitis now in ER for jaundice

a. MRCP or ERCP
b. Ultra sound
c. C.T
d. EDG endoscopy

680. You are resident and request from your senior to do appendectomy with his following but when
you start your senior leave O.T, after that you find mass at ileum or cecum, and what will you do?

a. Close abdominal again and informed the relative this is unreachable?


b. Right hemicolectomy starting and call the senior
c. Call the senior and wait him
d. Inform the relative the truth

681. High K in?

a. Saliva

682. Patient vomiting 3 times there are attack of melena, what the next step?

a. Arteriography
b. Sigmoid – colonoscopy
c. Upper GI endoscopy
d. Abdominal Series

683. Hyperkalemia K above case in scenario (K=6.9) ECG? All correct except one?
a. U wave
b. T. Peaked case
c. QRS wide
d. P. Flat

684. Anal prolapsed case in scenario?

a. Altemeier procedure

685. Hashimoto thyroiditis (case in scenario)

a. Radio I
b. Organic Iodin
c. THyroidectomy
d. Follow up

686. Gastroschisis associated?

a. Intestinal atresia

687. Patient old age in ICU developed right side weakness and confusion, then the nurse tied his limb
for confusing and aggressively, patient on the therapeutic heparin, what happened?

a. CNS stroke
b. Cerebral hemorrhagic
c. Psychic element

688.. Young patient with torsion testis at right side, what you will do?

a. Bilateral orchiopexy
b. Right Orchiopexy

689. Gastric mass by UGI scopy, not seen


a. Total gastrectomy
b. Partial gastrectomy
c. Radical gastrectomy

690. After diverticulitis abscess for 2 weeks. How many time should wait to do resection and
anastomosis?

a. 2 weeks
b. 6 weeks
c. 8 weeks
d. 3 weeks

691. Hernia in picture there is blush dark patch, what will you do?

a. Resection and anastomosis


b. Hot fomenation
c. Excision

692. Stab wound in right hypochondrium BP 100/60 pulse 104, what the best management?

a. Follow up and observation


b. Exploration for wound
c. Laparoscopic diagnostic
d. Laparotomy

693. Nascomial infection, now we indentifying by disease pathogen?

a. Pseudonomonas
b. Staphylococcus
c. Stretococcus

694. Anus mass (ulcerative) extend to muscular, best management?

a. Excision
b. Radical Excision
c. Resection and anastomosis
d. Hamartoma

695. Baby inserted NGT for colloid, it is blocked?

a. CHPS
b. Esophageal atresia
c. Duodenal atresia

696. Stable patient with stab wound above level of mandibule with hematoma?

a. Exploration of wound
b. Angiogram
c. Observation
d. Embolization

697. Stab Abdominal patient is stable by DPL: 1.9mg, best management?

a. Observation
b. Laparotomy
c. C.T abdominal
d. Lab. Investigation

698. Pre-operative high risk?

a. Aortic stenosis
b. Valve regurgitation
c. Bronchectasis

699. Patient with portal HTN?


a. Portocaval
b. Distal spleno renal

700. Alcoholic man with end stage liver disease and portal HTN, best management for GI bleeding?

a. Distal spleno renal shunt


b. Liver transplant
c. TIPS (Transjugular Intrahepatic Portosystemic Shunt)
d. Portocaval shunt

701. In electrolyte imbalance with ECG show Osborn J wave?

a. Hypokalemia
b. Hyponatremia
c. Hypotermia
d. Malignant hypertemia

702. Spigelian hernia covered by?

a. Rectus Sheath
b. External Oblique
c. Internal Oblique

703. Old patient with ischemia in right low leg blackish foot , see pic. Right iliac complete
obstruction, best management?

a. Angioplasty
b. Axillo femoral
c. Ilio femoral
d. Ilio Popliteal

704. Irreducible hernia, vomiting , constipation tender?

a. Stragulated
b. Obstruction
c. Incarcerated

705. Pancreatic collection by aspiration of upper GI endoscopy revealed – mucinous?

a. Pseudocyst
b. Cystoadenoma Mucoius
c. Intracutal Mucious tumour

706. Patient in shock with COP below and above vascular resistance?

a. Low cardiac index


b. High glumecular Filtaration rate
c. Perferal vascular disease
707. Abdominal pain and dyspepsia, ultrasound:NAD, WBC 16000, bilirubin increase, best
investigation?

a. HIDA scan
b. Ultrasound
c. C.T
d. Liver function

708. Patient 74 years old age, history of abdominal surgery 30 years ago present to ER constipation
less than 4 days, x-ray erect multiple fluid level, best management?

a. Laparotomy
b. C.T with contrast
c. Follow up
d. Colonoscopy

709. One year old baby, came to ER with fever for 2 days, examination – tonsillitis and otitis media
with right leg cellulitis, best management?

a. Admission and antibiotics


b. Discharge and antibiotics
c. Follow up in ER until fever subside then decide

710. Hernia repair then bowel injury with obvious feces, bowel was done, best management?

a. Primary repair
b. Repair with mesh
c. Repair with tension suture

711. Patient with massive trunkal burn, the way for emergent resuscitation?

a. Unburned peripheral limb


b. Venous cut down
c. Central venous line

712. What is least tumour cause gynecomastia?

a. Colon
b. Lung
c. Prostate
d. Gallbladder
e. Testis
f. Renal
g. Bone

713. What is least likely acute surgical abdomen?

a. Thalasemia
b. DKA
c. Henoch –schoneien purpura
d. Lead poisoning

714. Hernia contains Micheal diverticulum?

a. Spigelion
b. Litters
c. Rechter
d. Bantallon

715. Patient came to ER with stab wound to the neck c/o S.O.B and bubbling from wound, x-ray chest
showing mediastinum air. What is diagnosis?

a. Ruputure trachea
b. Rupture esophagus
c. Traumatic Pneumothorax
d. Entrapped external air

716. Old patient with COPD an IHD, ASA grade III comes with rectal prolabse O/E 7 cm rectal from
anus, What is best operation?

a. Altemers op – ASA – 3-4


b. Open Method
c. Closed method
d. Closed and circle – ASA – 4-5

717. SIRS
a. Temperature 38.9
b. Tachycardia
c. Respiration rate

718. Case Scenario fever and dark urine

a. Toxic shock syndrome

719. Pregnant 15 weeks has tremor and palpitation and thyroid swelling, thyroid examination are
symmetric and soft, there is cold nodule, treatment should be?

a. Thyroidectomy
b. Remove nodule with anti thyroid drugs
c. Radio active iodine
d. Radiation therapy

720. Gastroschisis association with intestinal

a. Atresia

721. Case of pulmonary embolism, ask about initial investigation


a. ABG
b. D-dimer

722. Case of pulmonary embolism, D-dimer is positive, what will you do?

a. ABG
b. Chest x-ray
c. C.T pulmonary angiography

723. Case of billroth ll after 3 months, patient has sever vomiting ‘No food particles due to?

a. Dumping syndrome
b. Duodenal stump leak

724. Case of pancreatic carcinoma

725. Ulcer at distal of lesser curvature need surgical intervention?

a. Billroth l
b. Billroth l and Vagotomy

726. Leimyoma of stomach, treatment

a. Laparoscopic approach

727. Hypokalemia in ECG in below K QT prolonged, ST depressed, T flat, U present

728. Hypercalcemia treatment and dialysis


Diagnosis: Blood tests
: Parathyroid hormone level
Med. : Calcitonin, prednisane and (I.V fluid and diuretics)
Surg. : Remove parathyroid gland

729. Hypocalcimia treatment and diagnosis


Diagnosis: Blood test (serum phosphate, magnesium)
Treatment : Vit. D and magnesium + calcium glunat 10%

730. Scenario of sigmoid diverticulitis with abscess and the patient with chronic constipation since 5
years, now has colicoly abdominal pain with fever he did not pass stool in last 6 hours these clinical
anifestate due to?

a. Perfuration
b. Diverticular obstruction
c. Abuse of laxative
d. Intraperitoneal infection

731. Old age marasmus features of hypervolemia?


a. Sunken eye
b. Shrunken eye
c. Dilated vein
d. Prominant bone face

732. TIA, what will you do?

a. Carotid duplex
b. Angio
c. MRI
d. C.T

733. Which of the following normally non felt palpable?

a. Hyoid bone
b. Parotid gland
c. Sublingual gland
d. Thyroid gland
e. Cerfical L.N

734. Male patient 30 years old with acute abdominal pain, WBC 25000

a. Acute Pancreatitis
b. PID
c. Gut perforation
d. Acute cholecystitis

735. Most important determination of surgery for hernia

a. Size
b. Containet
c. Complication

736. Liver is divided anatomically according to?

a. Hepatic venous drainage


b. Hepatic arterial supply
c. biliary drainage
d. Portal vein

737. Gunshot – Injury- Esophagus and internal carotid artery. What do you do?

a. Repair A and esophagus


b. Ligated A and esophagectomy
c. Repair A and esophagectomy
d. Ligated A and repair esophagectomy

738. Post cholecystectomy 4 weeks jaundice?


a. CBD Stricture
b. Missed stone

739. Most operative risk factors?

a. Bronchitis
b. Aortic Stenosis

740. Tumour beside the lat end of the eyebrow, how can you differentiated the swelling from cyst?

a. Punctum
b. Consistency
c. Translucency

741. Femoral Embolectomy

a. Popliteal pulse is felt


b. Recent symptoms
c. Recent Emboles
d. DVT

742. Obstructed femoral hernia with ileal ring (dark purple) cause?

a. Arterial interruption
b. Venous interruption
c. Transimullation

743. Condition has high mortality rate in elderly?

a. Intussuscetion
b. Acute appendicitis
c. Aortic enteric fistula
d. Radiation Proctitis

744. Patient with dysphagia – gurgling, food particles?

a. Achalasia
b. Zenker’s Diverticulum
c. Tumour

745. In the second day after AAA repair patient pass grossly blood stool, next step?

a. immediate exploratory laparotomy


b. Colonoscopy

746. The best indicator of adequate resuscitation in shock is?

a. Normal BP
b. Normal Pulse
c. Adequate urine output
d. Improve mental status
e. Lactate Level

747. Post eating 15 minutes abdominal pain?

a. GU
b. Ischemic colitis

748. Patient with T1 lesion 3 cm from anal verge, No L.N, best treatment?

a. Transmural resection
b. Abdominal perineal
c. Chemo radiotherapy

749. Most common cause of acute pancreatitis?

a. Gall stone
b. Alcohol
c. Viral

750. After Excision of the melanoma pathology positive edge, what will you do?

a. Radiotherapy
b. Dissection ingainal L.N
c. Excision again more safety margin

751. Main physiology of gall stone pancreatitis

a. Activation of proenzymes
b. Reflux of bile

752. 23 years old female patient with breast mass (2.1x1.5) by ultrasound fibrodema, her mother is
knows with breast cancer. What is the plan?

a. Follow up every 3 months


b. Follow up every 6 months
c. Excision
d. FNA
e. Core Biopsy

753. Painless Hematuria

a. Renal Tumor
b. UTI
c. Bilharziasis
d. Bladder Carcinoma

754. Preoperative for colon resection, ceftrex is given, post operative abscess formatic due to?
a. Anaerobic bacteria not covered by anti biotic
b. Grome negative not covered

755. Trauma at chest patient has respiratory rate 40/min trachea shifted to right congested neck
veins, BP 90/60?

a. Left tension pneumothorax


b. Left pneumothorax
c. Cardiac Tamponade

756. Stricture in CBD usually due to?

a. Surgical trauma

757. Sigmoid Volvulus, diagnosis?

a. Plain abdominal radiography

758. Case of spherocytosis, ask about pre operative investigation

a. Fragility test
b. Ultrasound

759. Case for liver laceration with CT

a. CT Trauma

760. Trauma below angle of mandible investigation for carotid injury

a. Angiography

761. Origin of basal cell carcinoma

a. Arise from pluripotential cells in the basal layer of the epidermis or follicular structures

762. Endoscopy cause esophageal trauma investigation

a. Water soluble contrast


b. Barium swallowing

763. Case of anal haemorrhoid prolapsed and spontaneous reduction

a. Grade l
b. Grade ll
c. Grade lll

764. Pilonidal abscess, ask about cause?


a. Male growth of hair follicles
b. Bad hygiene

765. A preterm baby (32 weeks) with gastroschisis is more likely to have?

a. Extraphy
b. Intestinal atresia
c. Microcephaly
d. Macroglossia

766. A motorcyclist sustained a blunt abdominal trauma, 6 hours later he admitted to ER with
abdominal pain and distension, his BP is 100/40 and won’t respond to resuscitation fluids given in ER,
what is the most suitable initial investigation to do in the case?

a. DPL
b. Ultrasound
c. Laparoscopic exploration
d. Plain x-ray

767. Most frequent cause of liver transplantation in adult is?

a. Active HCV infection


b. Biliary cirrhosis
c. Hepatocecular carcinoma
d. Metastatic Liver disease

768. Most common cause of haemophilia?

a. Liver trauma
b. Gall bladder Stones
c. Pyocele
d. Mucocele

769. A patient had right hemicolectomy for FPC (familial polyposis coli) 10 years ago, now
presenting with a 14 cm parietal abdominal mass at the operation site, what is the proper
management?

a. Triple chemotherapy
b. Conservative management
c. Wide Local excision
d. Combined chemo-radiotherapy

770. What thickness of melanoma at sentinel L.N biopsy is indicated?

a. < 0.76mm
b. > 1mm
c. > 3mm
d. > 5mm
771. Small cell lung cancer is commonly associated with?

a. Horner
b. Cushing’s
c. Pilmonary osteodytrophy
d. Myasthenia gravis – like

772. Undescended impalpable testis best diagnosis tool is?

a. CT
b. MRI
c. U/C
d. Abdominal exploration

773. Bacteria which previously thought to baa normal skin commensal with no pathogenic activity,
now believed to cause major systematic infections with high morbidity?

a. Pseudomonas
b. Fusobacterid
c. Bacterid
d. E-coli

774. During an incisional hernia repair in the upper abdomen, an iatrogenic transverse colon injury
happened with soiling of the wound area with colon contents, the injury was primarily paired, wound
is cleaned and good peritoneal toilet was performed, what is the best decision to consider about the
hernia repair?

a. Proceed with mesh repair


b. Primary repair
c. Delay the repair for 4 days then proceed with mesh repair
d. Close the abdominal wall under tension with primary repair

775. What factor determines the most prognosis of colon cancer?

a. Nodal Status
b. Transmural spread
c. Direct Spread
d. Site of the tumor

776. A middle 1/3 Esophageal cancer spreads to the main left branhus, what is the best investigation
to assess the degree of involvement of the bronchus in tumor spread?

a. Bronchoscopy
b. Mediastinoscopy
c. Chest x-ray
d. Esophageal endoluminal ultrasound

777. Transient Ischemic attracts best diagnostic tool is?


a. Brain CT
b. Brain MRI
c. Carotid coloured douplex
d. Carotid Angiograpy

778. Which factor determines graft viability in 1t 24 hours?

a. Capilary
b. Plasma imbibition
c. Venous drainage
d. Arterial supply

779. What is the most important initial management in acute Hypercalcaemia?

a. Dialysis
b. Rehydration with I.V fluids
c. Furosemide administration
d. Fusamax administration

780. A patient had colonoscopy upon removal of the scope multiple swelling bulged from the anal
verge which retracted spontaneously, what is the best management?

a. Hemorrhoidectomy
b. Stool softners
c. Transmucosal band ligation
d. Sclerotherapy

781. A patient with acute right iliac pain, x-ray show air under diaphragm, DRE is painful, temp is
38.8, what condition is most likely underlying this pic?

a. Perforated appendix
b. Perforated typhoid ulcer
c. Perforated peptic ulcer (duodenal)
d. Perforated gastric ulcer

782. A surgeon is exploring the neck for a parathyroid adenoma; the thyroid area was explored with
no evidence of parathyroid tissue, what would be the best next step?

a. Hemithyroidectomy
b. Strenomastoid division
c. Sestamibi Scan
d. Total Thyroidectomy

783. A 58 years old female patient recovered from a attack of pancreatitis which resolved
conservatively , 2 weeks later she came back with epigastric pain and swelling which was investigated
revealing a retroperitoneal cyst. What is the propose management?

a. Internal drainage
b. External Drainage
c. Cystogastrostomy
d. Conservative management

784. A 30 years old man accidentally discovered a 3mm polyp in his gall bladder his labs are normal.
What would be the action to take place in this case?

a. Cholecystectomy
b. Follow up with ultra sound
c. Sphincterotomy
d. MRCP

785. Right hypochondrial pain, ultrasound dilated thin walled gall bladder,most lik;ydiagnosis is?

a. Empyema of gall bladder


b. Mucocele
c. Acute Cholecystitis
d. Chronic cholecystitis

786. Severe headache 4 days following an operation in which spinal anaesthesia was used in most
probably due to

a. Temporal arteries
b. CSF leak
c, Meningitis

787. Most common cause of recurrence in epigastric hernia is?

a. Obesity
b. Straining
c. Weakness of linea alba
d. Neglected another defect

788. Persist vomiting for 12h, what would be the ECG changes expected?

a. Tall T-wave
b. ST segment elevated
c. Perminent u-Wave
d. Widened QRS complex

789. A case of hematesis , upper gastric indoscopy revealed a bleder in the gastric mycosa erosion
above the atery, this is mostly a:

a. Gastric ulcer
b. Variceal Bleeding
c. Dieulafoy’s Lesion
d. Bleeding tendency

790. Child pugh score

a. 5-6
b. 7-9
c. 10-15

791. An old man with congestive heart failure admitted to orhopedic unit with a femur neck fracture,
he received clindomycin for 1 week before he suddenly developed a bloody, D. Sigmoidoscopy revealed
a hyperaemic rectal mucosa with yellowish white flakes, what is the proper antibiotic to use in this
case?

a. Ciprofloxacin
b. Ampicilin
c. Cefotaxim
d. Metronidazole

792. Regarding H. Pylori, which of the following is true?

a. It is associated with gastric ulcer morethan duodenal ulcer


b. It is flagellar anaerobic negative bacilli

793. Most common thyroid malignancy neoplasm is?

a. Papiloma
b. Follicular
c. Medulary
d. Anaplastic

794. Glasgow comascale

795. The most common nosocomial infection in surgical patients is?

a. Urinary tract infection


b. Surgical Site infection
c. Pneumonia
d. Line infection

796. An HIV patient develops an acute abdomen with severe acute abdominal pain is greatest in the
right lower quadrant of the abdomen at exploration of the abdomen the appendix is normal but a
perforation of the distal ileum is found. The diagnosis is most likely?

a. A typical tuberculosis
b. Kaposis sarcoma
c. Ischemic Necrosis
d. Cytomegalovirus infection
e. B-cell lymphoma

797. Middle age male a heavy smoker perforated gastric ulcer, what is proper surgical decision?

a. Primary repair after biopsy from ulcer edge?


b. Billroth ll gastrectomy
c. Total gastrectomy
798. Tetanic – wound culture- Rod’s terminal spores

799. Morbidly obese patient had a bariatric surgery, after surgery he suffers from snoring and sleep
apnea , what medication to give postoperatively with great caution?

a. Antacids
b. Analgesic
c. Sedatives

800. Anal itching at night no bleeding no soiling is most likely?

a. Fissure
b. Fistula
c. Abscess
d. Infection with oxiuri

801. Midline neck swelling, not moving with deglutition ot tongue protrusion is most likely?

a. Dermoid cyst
b. Bronchial cyst
c. Thyroglossal fistula
d. Lingual thyroid

803. An 8 years old male patient complains of few days history of fever pain in lower part of the right
thigh and in ability to walk examination reveals swelling and tenderness at the lower part of the thigh
movements of the knee joint are free. Which of the following organism is cause of the problem?

a. E-coli
b. Klebsiella
c. Staphylococcus aureus
d. Streptocossus pneumonia

804. Regarding cardiac patients pre-operative, preparation, which of the following status is true?

a. Recent infarction is contraindication to elective surgery


b. Supraventicular arrhythmia is more dangerous than ventricular arrhythmia

805. A patient who is receiving a psychiatric care is being prepared for a next morning surgery, what
would be the most suitable sedative to use?

a. NSAIDs
b. Benzodiazepines
c. Barbiturates
d. Opioids

806. Congenital biliary atresia, best time of operation after whch bad prognosis is more likely?

a. < 8 weeks
b. < 11 weeks
c. < 12 weeks

807. A patient had a right hemicolectomy for a cecal adenocarcinoma T1aNoMo 2 years ago, he has
been well since then, now admitted to surgical unit with colicky pain and vomiting , which
investigation is the most useful in this case considering that the abdomen is distended but soft?

a. C.T
b. Colonoscopy
c. Exploration
d. Plain x-ray

808. Best staging investigation to assess wall affection degree in oesophageal cancer?

a. Endoluminal ultrasound
b. Cervical CT
c. MRI

809. In case of shock, best option to start with in fluid resuscitation is?

a. Crystalloids
b. Colloids
c. Matched PRBCs
d. O negative blood

810. Boil infection in the calf area which developed a local cellulitis with swollen area and greyish
discharge, what is your decision?

a. Leg CT
b. Leg MRI
c. X-rays
d. Surgical Exploration

811. Regarding a case of oblique inguinal hernia, reading the scrotum, which statement is true?

a. It is not easily reducible


b. Cant separate from testis
c. Cant feel the vas D
d. Positive transllumination test

812. Skull base fracture diagnosed by?

a. Themotympanum
b. Deteroriation of conscious level
c. Above I.C.P

813. Gland involved in MENL?

a. Thyroid
b. Parotid
c. Adrenal
d. Pancreas

814. Most common condition during breast lactation?

a. Breast abscess
b. Bacterial mastitis
c. Breast cyst

815. Medication used in anal fissure caused headache?

a. Glycerin nitrate
b. Dittazen
c. Site bath
d. Local Analgesia

816. Most objective treatment of intractable olecranon bursita?

a. NSAID
b. Corticosteroid injection
c. Aspiration
d. Surgical excision

817. Early gastric asenocarcinoma in antrum the type of surgical resection is?

a. R0
b R1
c. R2
d. R4

818. Patient in burn unit suddenly complains fresh bleeding per rectum, after resuscitation NG Tube,
no bleeding, most appropriate investigation?

a. Proctoscop
b. C.T abdominal
c. Mesenteric angiography
d. Gastroendoscopy

819. 6 years old child come with constipation and x-ray abdomen show distended loop with
impacted faeces, most appropriate investigation help diagnosis?

a. Tissue biopsy
b. Endoscopy
c. Anorectal manometry
d. Soft reset enema

820. Huge umbilical hernia will do laparoscopy, most common complication?


a. Adhesion
b. Recurrence
c. Failed defect
d. Bleeding

821. Most common complication post operative rectal prolapsed?

a. Infection
b. bleeding
c. Incontinence
d. Itching

822. Most common malignant retroperiteal tumour?

a. Lyphoma
b. Liposarcoma
c. Leiomyoma
d. Carcinoma

823. Post operative lap inguinal hernia after 5 hours show severe pain?

a. Back to operation theatre


b. Local infiltration

824. E-coli in urine cause cystitis, most appropriate investigation?

a. Colonoscopy
b. Urine analysis
c. Cystoscopy
d. Urine culture

825. Difference between incomplete inguinal hernia and spematocele?

a. Position high level


b. Pulsatil
c. Translumination
d. Soft compression

826. Condition has high mortality rate in adult (old age)?

a. Intususception
b. Aortic enteric fistula
c. Acute appendicitis
d. Radiation Proctitis

827. 45 years old female ex. Breast mammogram show 1cm mass upper left quarter satellite
appearance with minimal calcification and affect the skin most appropriate next step?

a. MRI
b. Core biopsy
c. Ultrasound
d. Excision

828. Thyroid nodul, left hot with palpitation, carotid pulsation, T3 above, TSH normal, management?

a. Total thyroidectomy
b. Radioactive iodine
c. Anti- thyroid drugs
d. Left loboctomy

829. Melanoma upper arm, L.N metastases should be clearance level 1?

a. Medial to minor pectoral


b. Posterior to minor pectoral
c. Lateral to minor pectoral

830. 60 years old age patient come with fever lower abdominal pain, 3 days now moderate ascitis,
most appropriate management?

a. Antibiotics
b. Exploration
c. C.T Abdominal
d. Paracentisis

831. 56 years old male with constipation bleeding per rectum, cachexia, has cercumferentia anal
verg adenocarcinoma, management?

a. Defunctioning colostomy
b. Abdominal perineal resection with permanent colostomy
c. Radiotherapy
d. Chemotherapy

832. RTA renal tubular acidosis, the mainstay of therapy of all form is?

a. I.V Fluid
b. K. Citrat
c. Na bicarbonate

833. In large train accident has 5 victims presented in ER is fasting the patients whichc need early
assessment?

a. Old age
b. Multiple injuries
c. Life threatening injury
d. Super facial injury

834. Commonest cause of oesophageal injury?


a. Endoscopy
b. Trauma
c. Pathological
d. Spontaneous

835. Patient post eating 2 hour feeling hungry, but relieved after eating?

a. Late dumping
b. Early dumping

836. The vast historological majority of anal carcinoma

a. BCC
b. SCC
c. Adenocarcinoma

837. Regarding Mallory weiss syndrome

a. Endoscopic is diagnosis
b. Tears can be extended to gastric greater curvature

838. Male patient 45 years old, complain RUQ pain since months ago, investigation provided: He has
cjoleochal cyst 2cm, type 3. What is the management?

a. Roux-en-y hepaticojejunostomy
b. Simple excision with T-tube
c. Excision transduodenal approach
d. Endoscopic Sphincteretomy

839. Organ has auto regulation of blood supply in hypovolemic shock?

a. Skin
b. Heart
c. Intestine
d. Kidney

840. All of the following are symptoms and signs of hypothyroidism except?

a. Low body temperature


b. Amenorrhea
c. Weight gain
d. Cold intolerance

841. The most common malignant liver mass?

a. Hepatocecular carcinoma
b. Hepatoplastoma
c. Angiosarcoma
d. Lymphoma
e. Metastatic tumour

842. Esophagitis h. with scan?

a. Oesophageal barett
b. Short oesophageal
c. Oesophageal ring

843. Capacitating coupling fenomen?

a. Burn in gut by direct conduction with improper insulation electrical


b. Electric charge strong transfer between 2 electrodes
c. Electrostatic current discharge transfer from insulation electrode
d. Electrostatic current discharge transfer between 2 electrodes

844. Most common complication of parothydectomy

a. Facial nerve injury


b. Recurrent nerve

845. Clinical manifestation of early hemographic shock

a. Cold clammy extremity with tachycardia


b. Hot clammy extremity with tachycardia
c. Cold clammy extremity with bradycardia

846. 80 years old age man alcoholic with chronic pancreatitis high calcification?

a. Follow up (stent)
b. Pancreatectomy + splenic astery

847. 60 years old age man alcoholic with chronic pancreatitis high calcification?

a. Follow up (stent)
b. Pancreatectomy + splenic astery

848. Most common osteomyelitis cause?

a. Felon
b. Website infection
c. Mid palmar space
d. Thenar space infection

849. Glasgow coma scale of patient in ER with eye give response to pain, verbal response as
inappropriate words and motor response as withdraws to pain flexia from pain THE GCS score is?

a. 7
b. 8
c. 9
d. 10

850. Hernia below and meial pubic tubercle with positive ring test

a. Incomplete indirect
b. Complete indirect
c. Direct

851. A patient 50 years old alcoholic with chronic pancreatitis, C.T Abdominal show main pancreatic
duct 3cm cyst and small cysts on small branch pancreatic duct, management?

a. Pancreatectomy
b. ERCP and stent
c. Side to side pancreaticojujenostomy

852. RTA oriented with face trauma loss of 2 teeth, after presented in ER become confused, most
appropriate cause?

a. Hypoglycaemia
b. Head trauma
c. Hypoxia

853. Patient in hypovolemic shock with hypokalemia for resuscitation

a. Give him bolus I.V fluid normal saline ni rate 40mg/l with kcl in same time until
resuscitated then give him kcl
b. Give him normal saline in rate 40 mg/l with kcl in same time until resuscitated

854. Most common cause respiratory distress during delivery

a. Thyroid cyst
b. Cystic hygroma
c. Thyroglossal cyst
d. None of the above

855. 11 years old RTA, abdomen truma, spleen teared 3cm , what do you do?

a. Exploration splenectomy
b. Exploration splenorrhapy
c. Conservative

856. Recurrent inguinal hernia with history of prostatectomy

a. Open repair
b. TEP
c. TAPP
d. Truss

857. Patient in CVP take TPN, take2 unit blood – coma


a. Electrolytes disturbance
b. Hypoglycaemia
c. Take blood transfusion

858. Patient comes with DKA, keton body in urine, hyperglycaemia, treatment?

a. I.V fluid
b. Insulin
c. I.V fluid + Insulin

859. Liver lesion cause thrombocytopenia

a. Hemangioma
b. HCC (hepatocellular carcinoma)

860. Exploration of intestinal obstruction, gave treatment, metronidazole, WBC was 18000. Became
11000, creatinine + urea above

a. Discontinued and start imipenem


b. Continued treatment

861. Female after C.S 4 time has lower abdominal bulging swelling when standing, disappear when
lying down, most appropriate investigation?

a. C.T abdominal
b. Ultrasound
c. MRI

862. After oesophageal upper endoscopic doctor was worried about oesophageal perforation, what
do you do?

a. X-ray plain chest


b. Repeat upper endoscopy
c. Barium meal
d. Water soluble swallowing

863. Abdominal exploration, midline pulsatile mass, 2 days ago M.I, renal failure most appropriate
earliest complication?

a. Rupture aneurysm
b. Ischemic colitis

864. Common presentation of rectal prolapsed

a. Bleeding
b. Discharge

865. ERCP show biliary duct (double duct sign) for patient with jaundice and MLD
a. Pancreatic cancer

866. Patient with RTA, C.T abdominal show

a. Liver laceration

867. Infection + burn above + high creatinine

a. Severe septic

868. Dark arm + hypotension

a. Toxic septic shock

869. Gastroschisis

a. Intestinal atresia

870. Difference between thyroid nodule and thyroid cyst

a. Ultrasound

871. Serious side effect of hyperkalemia

A. Cardiac arrest
B.
872. Treatment of pseudomonas colitis

a. Metronidazol

873. Picture of haemorrhoid grade 4

a. Hemorrhidectomy

874. Vomiting recurrent

a. Hypokalemia hypochloemia metabolic alkalosis

875. Organism a common cause of nosocomial infection is

a. Staphylococcus epidermidis

876. In pseudomembranous colitis clindamycin for a strong toxin of

a. Clostridum diffucile

877. Oranism cause streptokinase of cellulites


a. Streptococcus pyogenes

878. All of the following are symptoms and signs of hypo thyroidism except?

a. Low body temperature


b. Constipation
c. Weight gain
d. Cold intolerance
e. Amenorrhea

879. All of the following are true regarding chronic lymphocytic (Hashimoto) thyroiditis except?

a. It is the most common inflammatory thyroid disorder


b. Leading cause of hypothyroidism
c. Malignancy found in 15% of the case
d. Age of presentation 30-50 years

880. All of the following are indication for thyroidectomy in thyroid cyst except?

a. Failure to do complete aspiration


b. Large cyst >4cm
c. Complex cyst (solid-cystic)
d. Recurrence after frequent aspiration
e. It is painless

881. The most common malignant liver mass?

a. Hepatocellular carcinoma
b. Hepatoblastoma
c. Lymphoma
d. Metastatic Tumour
` e. Metastatic tumour

882. Most of the cases of liver cancer are advanced at time of diagnosis because all of the following
except?

a. Liver cancer grow quickly


b. Liver tumours resemble those of other medical conditions
c. Liver cancer cant b diagnosis using a standard liver tests
d. Liver tissue is not available to regenerate to replaced the damaged tissue

883. Mondors disease is a variant of thrombaphlebitis that involves the superficial veins of?

a. The anterior chest wall and breast


b. The thighs
c. The legs
d. The scrotum
e. The vagina
884. The dominant etiologic factor in acute appendicitis is?

a. Travelling infection from large bowel


b. Decrease local immunity of appendix
c. Part of inflammatory bowel disease
d. Obstruction of the lumen by fecaliths
e. Distal colonic obstruction

885. The body mass index of the normal weight is?

a. 15-19 kg/m2
b. 20-25 kg/m2
c. 26-29 kg/m2
d. 30-34 kg/m2
e. 35-39kg/m2

886. Disadvantages of laparascopy when compared with laparotomy include all of the following
except?

a. Difficulty controlling severe bleeding


b. Poorer visualization of the operative fields
c. Greater difficulty placing sutures
d. Loss of factile sensation
e. higher operating room costs

887. Laparoscopic cholecystectomy is indicated for all of the following except?

a. Biliary dyskinesia
b. Initial treatment in patient with severe cholanzi
c. Acute cholecystitis
d. Symptomatic cholelethiasis
e. Biliary pancreatitis

888. Most common type of breast carcinoma is?

a. Paget’s disease
b. Anaplastic carcinoma
c. Scrrhous carcinoma
d. Atrophic scirrhous carcinoma
e. Mastitis carcinoma

889. Anal polyp in patient of leukaemia

a. HPV
b. Immunodeficiency
c. CMV

890. Most common leak post operation of cholecystectomy


a. Gall bladder bed
b. CBD
c. Stump of cystic duct

891. Left tip of shoulder pain post trauma

A Spleen
b. Diaphragma
c. Stomach

892. Most common complain of chronic pancreatitis?

a. Pain
b. Loss of weight
c. Vomiting

893. Pancreatic duct stone treatment?

a. Drugs
b. ERCP
c. Operative

894. 80 years old patient bleeding per rectum otherwise full healthy, causes?

a. Dieulafoy lesion
b. Radiation colitis
c. Ischemic Colitis

895. Most common late complication of repair AAA?

a. Eschemic solitis
b. Aorto enteric fistula
c. Rupture

896. Late complication of huge umbilical hernia repair?

a. Adhesion
b. Recurrence
c. Bleeding

897. Massive hemotorax due to?

a. Large vessel injury


b. Lung injury
c. Intercostals vessel injury

898. The most common cause of pyogenic liver abscess is?

a. Hematogenous infection
b. Lymphatic spread
c. Secondary to infected abdominal viscena
d. Secondary infection of hydatic cyst
e. Biliary obstruction

899. Which of the following conditions are associated with increased risk of breast cancer?

a. Fibrocystic mastropaty
b. Fibro adenoma
c. Atypical hyperplasia
d. Papillomatosis
e. Breast lipoma

900. All of the following are true regarding fibro adenoma of the breast except?

a. It is the most common breast tumour in adolescents and young women


b. Painless, mobile and slow growing mass
c. Firm and rubbery texture
d. Are not pre malignant lesions
e. Excisional biopsy is necessary for every fibro adenoma

901. A 15 years old boy awakens with sudden onset of right scrotal pain and tenderness
accompanied by nausea and vomiting. Which of the following is the most appropriate diagnosis and
represents a surgical emergency?

a. Acute Prostitis
b. Acute epididymitis
c. Torsion of the testicle
d. Acute Appendicitis
e. Orchistis

902. All are true regarding Achalasia except?

a. Dysphgia to both solids and liquid


b. Barium swallow reveals a birdlike
c. Treatment of choice is total gastrostomy
d. Usually associated with weight loss
e. The underlying defect is failure of the lower oesophageal sphincter to relax

903. One of the following is the most common cause of massive upper G.I bleeding?

a. Gastritis
b. Duodenal ulcer
c. Esophagitis
d. Mallory Weiss syndrome
e. Oesophageal varrices

904. What is the most reliable method for precisely locating the site of upper GI bleeding?
a. Barium meal study
b. Expletory laparotomy
c. Arteiography
d. Upper endoscopy
e. Radionuclide scanning

905. On the following is not true regarding Meckel’s diverticulum?

a. It is the most common diverticulum of the GI tract


b. It can cause bleeding
c. It can cause intussusception and obstruction
d. Asymptomatic Meckel diverticulum should be resected
e. More common in male

906. Uncomplicated acute pancreatic is best managed with all of the following except?
a. Intravenous hydration

b. Naso – gastric decompretion


c. Abdominal imaging with ultrasound or C.T
d. Strong painless killer
e. Urgent laparotomy to clean the abdomen

907. The most common complication after haemorrhoidectomy is which of the following?

a. Urinary retention
b. Rectal bleeding
c. Incontinence
d. Wound infection
e. Intestinal obstruction

908. Which of the following types of colonic polyps is associated with the highest incidence of
malignant degeneration?

a. Tubular adenoma
b. Tubulovious adenoma
c. Villous adenoma
d. Hamartomatous polyp
e. Lipomatous polyp

909. The major problem with Ogilvie syndrome (acute obstruction of the colon in the absence of any
identifiable mechanical obstructing lesion) is the risk of?

a. Cecal perforation
b. Malignancy
c. Rectal prolapsed
d. Duodenal ulcer
e. Small bowel perforation

910. Characteristic injuries of blunt the include all the following except?
a. Fracture sternum
b. Transacted Aorta
c. Pulmonary Contusion
d. Rupture urinary bladder
e. Bilateral rib fracture

911. Concerning clinical evaluation of pelvic injuries on is true

a. Rectal examination is often unnecessary


b. The patient should have uretal catheter inserted in ll cases to monitor urine output
c. Lateral compression injury are usually mere sever than antero-posterior compressic
d. Pelvic stabilization is the first priority
e. Unstable pelvic ring fractures are associated with a high mortality

912. The following are indication for laparotomy in abdominal trauma except?

a. Peritonitis
b. Persistent shock
c. If there is no abdominal free fluid on CT and ultrasound
d. Uncontrolled Haemorrhage
e. Gunshot wounds

913. The following would be indication to transfer a patient to a burns centre except?

a. Chemical burns
b. Burns involving inhalation injury
c. Cirumfeventid burns of the extremities
d. Third degree burn to the genital organs
e. First degree burn of 15% of TBSA to the chest of an adult man

914. The following are the risk factor for peri operative DVT except?

a. Obesity
b. Operation with long duration
c. Patient with malignancy
d. Pelvic Operation
e. Young age

915. The best policy to treatpost-operative pain is to give?

a. NSAID
b. Subscutaneous
c. Paracetamol
d. Regional Anaesthesia
e. Multimodal analgesia

916. What is the most common breast condition that a middle aged women can have of the
following?
a. Breast Cancer
b. Fibro adenoma
c. Fibrocyst disease
d. Ductesia
e. Lipoma

917. Ketamine is contridicated?

a. In an asthmatic patient
b. For caesarean section
c. In a patient with facial burn
d. In a patient with increased (ICP)
e. In hypotension patient

918. A 45 years old man presents with progressive cervical lymph nodes enlargement, since 4
months, the most appropriate investigation is?

a. X-ray soft tissue


b. FNA
c. Lymph node biopsy
d. Thyroid isotopescan
e. Tuberculin Test

919. Clinical features of paralytic ileus include the following except?

a. Effortless vomiting
b. Absolute Constipation
c. Abdominal Distension
d. Severe Abdominal Pain
e. Absent Bowel Sounds

920. The most precise diagnostic screening procedure for differentiating benign thyroid nodules
from malignant ones is?

a. Thyroid ultrasonography
b. Thyroid scintiscan
c. Thyroid hormone level
d. Neck C.T
e. FNA

921. The mainsty of therapy for all forms of shock is?

a. I.V fluid
b. Epinephrine
c. Phenylepherine
d. Red Blood cells
e. Oxygen
922. All of the following are expected in patient with pheochromocytoma except?

a. HTN
b. Palpitation
c. Headache
d. Anhiodrosis
e. Nausea and vomiting

923. Which of the following is the best guide to fluid replacement in a burns patient?

a. Hemoglobin and hemetocrit


b. Urine output
c. Urine Osmolality
d. Plasma Volume
e. Specific gravity

924. The first manifestation of latrogenic removal of all parathyroid glands after total thyroidectomy
is?

a. Carpal spasm
b. Facial twitching on tapping the facial nerve
c. Circumoral numbness
d. Pedal Spasm
e. Diarrhea

925. The best prophylaxis against infection in dirty wound is achieved by?

a. Avoiding the use local anaesthesia


b. Inserting a drain into the wound
c. Administration a large appropriate antibiotics
d. Administration tetanus antitoxin
e. Removing foreign body and devitalized tissue

926. Which of the following is not a sign of portal hypertension?

a. Caput medusa
b. Hepatomegaly
c. Ascites
d. Splenomegaly
e. Lower limb varicose veins

927. All of the following are common causes of small bowel obstruction except?

a. Post surgical adhesion


b. Inguinal hernia
c. Incisional hernia
d. Small bowel tumour
e. Fecal Impaction
928. At which level end of the spinal card?

a. Forth lumbar vertenbrae


b. L3-L4 interspace
c. Second Lumbar Vertebrae
d. T12-L1 Interspace
e. S2-S3 Interspace

929. 20 years old women with a bloody single duct discharge and a palpable mass at the areola no
malignant cells are isolated from discharge the likely diagnosis?

a. Ductal Careinoma
b. Duct Ectasia
c. Intraductal Papilloma
d. Prolactinoma
e. Periductal Mastitis

930. The most common type of intracranial haemorrhage?

a. Extradural
b. Subdural
c. Subarachnoid
d. Intra cerebral stroke
e. Intra venticular

931. Which of the following statement about the anatomy of the oesophagus is correct?

a. The cervical oesophagus passes behind and to the right of the trachea
b. The oesophagus deviates anteriorly and to left as it enters the abdomen
c. The thoracic oesophagus enter the posterior mediastinumanteriorly to the aortic arch
d. The thoracic oesophagus passes behind the main stem bronchi and the pericardium
e. The oesophagus enters the diaphragmatic hiatus at the level of T8

932. Most common hernia in females is?

a. Femoral hernia
b. Umbilical hernia
c. Obturator hernia
d. Direct Inguinal hernia
e. Indirect Inguinal hernia

933. The preferred operation for initial management of a thyroid nodule the is considered suspicious
of malignancy by FNA is?

a. Nodule excision
b. Partial thyroidectomy
c. Total thyroidectomy
d. Near Total thyroidectomy
e. Total Lobectomy and isthmusectomy
934. Acute appdendicitis is most common associated with which one of the following signs?

a. Temperature above no celcius


b. Frequent loose stole
c. Anorexia abdominal pain RLQ tenderness
d. WBC count >20000 per cu ml
e. None of the above

935. Hyperthyroidism can be caused by all except?

a. Medulary thyroid cancer


b. Gravis disease
c. Plummer disease
d. Struma Ovary
e. Hashimoto disease

936. A major bleeding in the thigh is best controlled in emergency room by?

a. Tourniquent
b. Direct compression by hand
c. Exploration under local anaesthesia and clamp
d. Exploration and ligation under local anaesthesia
e. Send to operation room for exploration under general anaesthesia

937. One is false?

a. Femoral artery is lateral to femoral vein in the inguinal area


b. Femoral nerve is present within the femoral sheath
c. Femoral triangle is bounding by sartorious muscle laterally
d. Sciatic nerve injury can be avoided by grieving I.M injection on the upper lateral
quadrant on the gluteal area
e. great saphenous vein drain to the femoral vein below the inguinal ligament

938. The principal blood supply to the parathyroid glands is by?

a. Superior thyroid artery


b. Inferior thyroid artery
c. Thyroid I.M artery
d. Parathyroid arterial branches from external carotid artery
e. Highly variable

939. Most common cause of goitorus hyperthyroidism in adult?

a. Hashimoto disease
b. Gravis disease
c. Riedel thyroid
d. Deqeuvain thyroid
e. Iodine deficiency
940. The first step in management Hypercalcaemia is?

a. IV hydration normal saline .9 %


b. hydrocortisone
c. Dialysis
d. Administration of furosemide
e. Administration of mithramyycine

941. All predispose to necrotizing fasciitis except?

a. DM
b. Severe head injury
c. Cortison theraphy
d. Advance malignancy
e. Renal failure

942. The most precise diagnostic screening procedure for differentiating benign thyroid nodules
from malignant once is?

a. Thyroid ultra sonography


b. Thyroid Scintision
c. Thyroid hormone suppression
d. FNA
e. C.T

943. All can be seen in MEN 1 except?

a. Hypercalcemia
b. Diarrhea
c. Hypoglycemia
d. Diabetis mellitus
e. Paroxysmal hypertension

944. Hairy cell leukaemia

a. Can be cured with splenectomy


b. Is n aggressive form of leukaemia
c. Death is usually related to infection complication
d. Is a T-cell leukaemia

945. The most common indication for surgery In chronic pancreatitis


a. Jaundae
b. Pain
c. Pseudocyst
d. Gastric outlet obstruction
e. Endocrine deficiency

946. The most common cause of spontenea intestinal fistula is?


a. Radiation Injury
b. Malignancy
c. Cronh’s Disease
d. Diverticular Disease

947. Abnormal bleeding with normal prothrombin time occurs with

a. Heparin overdose
b. Cirrhosis
c. Haemophilia
d. Von wille brand disease

948. 30 years old female patient with 1.5cm cervical lymph node along the anterior border of
sternocleidomost old muscle, clinical examination and routine blood tests are unremarkable she
undergoes on excisional biopsy of the lump as a day case, the histology raport reveals iuncapsulated
infiltrative carcinoma with marked fibrosis and cystic changes within the lump node, what is the most
likely pathology?

a. Anaplastic thyroid carcinoma


b. Follicular thyroid carcinoma
c. Malignant lymphoma of the thyroid
d. Medulary thyroid carcinoma
e. Papillary Thyroid carcinoma

949. What is the percentage of water in a healthy adult?

a. 30-40 of body weight


b. 45-65 of body weight
c. 70-80 of body weight
d. 15-20 of body weight

950. Which one of the following is prodominant cation in intercellular fluid?

a. Chloride
b. Calcium
c. Sodium
d. Potassium

951. Which of the following presents with decreased deep tendon reflexes?

a. Hypomagnasemia
b. Hypocalcaemia
c. Sodium
d. Potassium

952. The most frequent heredity bleeding disorder?

a. Hemophelia A
b. Hemophelia B
c. Von willebrand disease
d. Immunogenic thrombocytopenic purpose
e. Thrombasthenia

953. About heparin all the following statement are true, except?

a. Unfractionated heparin needs monitoring of activated partial thrombaplastine time


b. Excess bleeding from heparin overdose is treated by protamine sulphat
c. Prolonged administration may cause osteoporosis
d. Heparine may cause Thrombocytosis
e. Low molecular weight heparin can be given without monitoring the coagulation tests

954. Acute Hypercalcaemia in a patient with advanced breast cancer?

a. Is cause by parathyroid hyperplasia


b. Is a better treated at home
c. Bilateral Lower imb Edema is a constant feature
d. I.V fluid Administration is an important line of treatment
e. Chemotherapy is a main method of reducing serum calcium

955. The following statement about post mastectomy arm edema are alltrue except?

a. Is always transient
b. Adding radiotherapy to the exilla after clearance of lymph nodes increases the riskof
edema
c. Rough dissection of the axillary at surgery is one of the causes
d. Insertion of an I. V line in the ipsilateral upper limb is a predisposing factor
e. May be caused by recurrence in the axilla

956. For modified radical mastectomy all the following statement one true except?
a. The aerola and nipple are removed
b. The whole breast tissue are removed
c. Block dissection of the auxiliary vein
d. Nerve to serratus anterior should be separed
e. The pectoralis major muscle is separed

957. Regarding duct carcinoma in situ of the breast, one statement only is true?

a. Maybe discovered on screening mammography for symptomatic women


b. Is not true malignancy
c. It is not progress to invasive cancer but is a powerful marker of increased cancer risk
d. Close observation is all that is needed
e. Is always bilateral

958. Concerning sampling of the breast lump all the following statement are true except?

a. The most occurate is excion biopsy


b. FNA does not require general anesthesia
c. FNA shows tissue achrituture
d. Frozen section biopsy is sometimes inconclusive
e. True cut biopsy obtains a core of tissue

959. The following statement about lymphatic drainage of the breast are all true except?

a. Lymphatic from medial half of the breast may drain in internal mammory nodes
b. From anywhere in the breast lymphatic drain mostly to auxiliary nodes
c. Division of auxiliary nodes into levels l, ll and lll and depends upon the anatomical
relationship with the pectoralis minor muscle
d. Level one auxiliary nodes is the highest in the axilla
e. The sentinel node to be involved with cancer spread.

960. Regarding traumatic diaphragmatic rupture. All the following statement are true except
a. It mainly affects the left side
b. Plain chest x-ray can be diagnostic
c. In is very easy to diagnose
d. It is likely to be associated with abdominal and thoracic injury
e. The only treatment is surgical repair

961. A work man fell on a bar that hit the perineum, on presentation in the emergency room, he is
found to have a drop of blood at the tip of the urethra, there is also a perineal hematoma management
includes all the following except?

a. Advice the patient not to pass the urine


b. Immediate catheterization
c. Antibiotic administration
d. Urgent ascending urethrography
e. Suprapubic cystostomy if a urethral tear is diagnosed

962. Missed instrument after laparotomy, best to prevent


a. Time out
b. Count towels and instruments before closure
c. Choose the stuff with well experience

963. RTA patient with respiratory distress present even after chest tube with subcutaneous
emphysema?

a. Bronchoscopy
b. Endoscopy
c. Bronchography

964. Patient with wire wound (open), last vaccine 12 years, he took the doses?

a. Toxoid
b. Toxoid + IG
c. Anti-tetanus serum
965. Patient with dark stool, anaemia with negative colonoscopy

a. Dieulafoy lesion
b. Oesophagus varices

966. Femoral embelectomy

a. Popliteal pulse is felt


b. Rec symptoms
c. Rec embolism
d. DVT

967. Which of the following normally non felt palpable

a. Hyoid bone
b. Parotid gland
c. Sublingual gland
d. Thyroid gland
e. Cervical L.N

968. Patient with controlled diabetes with mixed insulin and oral hypoglycaemic drugs ASA

a. l
b ll
c. lll

969. Cholecystitis with above creatinine no hypotension

a. SIRIS
b. SEPSIS
c. Sever SEPSIS
d. Septic shock

970. Appropriate treatment for Grav’s disease

a. Anti-thyroid drugs
b. Thyroidectomy
c. Radioactive

971. Case with his of thyroidectomy for multi nodular goitre 2 years ago, now lab TSH above, normal
T3, T4. Cause of recurrent goitre?

a. Hypothyroidism
b. Hyperthyroidism
c. Iodin deficiency
d. Ant T4 drugs

972. Sign of dehydration in old age


a. Sunken eye
b. Bony prominence
c. Muscle wasting
d. Shrunken eye

973. Colon cancer with multiple hepatic metatisis – colon resected – treatment for hepatic metatisis

a. Angiographic embolization
b. Ablation
c. Chemotherapy
d. Resection

974. Multiple colorectal polyps > 100, treatment?

a. Total colectomy
b. Follow up
c. Mile’s operation

975. Post central line, dyspnea, breath sounds not auscultation in right side 1st to do?

a. Remove central line


b. Needle aspiration
c. Miles operation

976. Scenario with case post laparotomy – drain culture – gr negative organism best?

a. Cipromycine
b. Vancomycine
c. Meriomycine

977. Patient presented with abdominal pain + vomiting with past history of colectomy for colon
cancer, best investigation?

a. X-ray
b. CT
c. Barium enema

978. The pathophysiology of haemorrhoid?

a. Enlarged caution
b. Above porto systemic pressure

979. Patient with chronic abdominal pain, dyspepsia with H/O attack of sever epigastric pain and
vomiting 3 mage, show with palpable epigastric mass, appropriate treatment?

a. Conservation
b. Internal drainage
c. Antibiotics
980. Check ulcer with negative cervical LN

a. BCC
b. Melanoma
c. SCC

981. Best evaluated biliary tract

a. C.T
b. Ultrasound
c. MRCP

982. Best evaluated pancreatic duct

a. C.T
b. Ultrasound
c. MRI

983. Patient prepared for whipple operation the most worse prognostic factor?

a. BMI < 15
b. Bilirubin = 50
c. HB = 80

984. Patient with parathyroid gland hyper function, a surgeon he wants to remove one parathyroid
gland that is producing excessive amount of parathyroid hormone, which the method that good help
him?

a. C.T
b. Ultrasound
c. S.s (Sestambi Scan)
d. MRI

985. The best investigation for GERD?

a. Ph. Monitoring
b. Endoscopy
c. Manometry

986. Patient with severe diarrhea, best fluid?

a. N.S
b. Ringer
c. Dextrose

987. Patient with dehydration, best fluid for maintenance?

a. N.S
b. Ringer
c. Dextrose

988. Patient with DKA with limb ischemic need amputation, relative refused

a. Follow relative wish


b. Got advice from court
c. Do amputation

989. Physiopathology of septic shock

a. High arterial pemiability


b. low peripheral tissue gas exchange
c. low COPD

990. Rectal prolapsed old female with partial circumferences and short segment?

a. Delorme procedure
b. Resection rectopexy
c. Mesh rectopexy

991. Patient presented with severe abdominal pain 1 hour ago with absence of liver dullness, BP
110/70, pulse 130?

a. Perforated live abscess


b. Leaking aortic aneurysm
c. Perforated D.U

992. About succession splash all the following is correct except one?

a. The test is not valid after eating or drinking


b. The test is fit for curling ulcer
c. The test confirm diagnosis for gastric outlet obstruction
d. Hear the splashing when rocking a patient with fluid collection

993. The most common secretion of the K in?

a. Stomach
b. Small intestine
c. Saliva
d. Colon

994. Origin of BCC

a. Keratinocyte
b. Hypoderm
c. Epidemal stem Cell
d. Sweat gland

995. Patient with calf pain 2 days ago with coldness at foot with delayed capillary refilling?
a. DVT
b. Acute ischemia
c. Chronic ischemia

996. Patient with pain and discoloration of leg up to knee joint absent popliteal artery and absent
femoral superficial artery, pain within hours appropriate investigation?

a. ABI (Ankle Brachial Index)


b. Doppler
c. Angiography
d. X-ray

997. Patient male with hematemesis which stop spontaneously, endoscopy done shows visible
artery at cardia

a. Dieulafoy
b. Peptic ulcer
c. Perforation

998. Patient with oesophageal varicose post initial resuscitation, bleeding stop, what next?

a. Endoscopic and sclerotheraphy


b. Octreotide
c. Vasopressin I.V
d. Devascularization

999. Patient with epigastric pain endoscopic biopsy document H. Pylori?

a. Billroth ll
b. Eradication of H. pylori
c. Nothing

1000. Patient with peptic ulcer on the blocker but not respond well, doctor advice to him
pantoprazole patient ask for side effect of it on long run up to years?

a. Anaemia
b. Diarrhea
c. Unknown disorder
d. Ten fold increase osteoprotic and biopsy

1001. The treatment of choice for Barrett’s esophagis with severe dysplasia is?

a. Follow up endoscopic and biopsy


b. Esophagectomy
c. Nissen Fundiplication
d. Proton pump inhibitors
1002. Patient with gastric ulcer, vomiting 4 times per day, which pathophysiolgy change with
alkalosis?

a. Hyperchloremic
b. Hypokalemia
c. Hypernatremia

1003. The most complication of blood transfusion?

a. Hemolytic reaction
b. Human Immunodeficiency virus transmission
c. Allergic reaction
d. Volume overload
e. Coagulopathy

1004. Patient with abdominal succusion splash which patjophysology occur?

a. Hypokalemic hypochloromic metab acitosis


b. Hypokalemic hypochloromic metab alkalosis

1005. Baby with (CHPS)Congenital Hypertrophic Pyloric Stenosis with abdominal succsion splash,
which pathophysology occur?

a. Hypokalemic hypochloramic metabolic alkalosis

1006. At an Operation for small bowel obstruction, cecal volvulos is diagnosed, the cecum is viable, he
procedure of choice is?

a. Cecopexy
b. Tube cecostomy
c. Right Hemicolectomy
d. Resection, ileostomy and mucousfistula

1007. Patient with loss loose stool and lower abdominal pain with bleeding red rectum –
pathophysiology changes occur to?

a. Na
b. K
c. Mg
d. Cl

1008. Patient with intra peritoneal infection with ABG changes, PCO2=25, HCO3=17, PH=7.2, PO2=85?

a. Metabolic Acidosis
b. Metabolic Alkalosis
c. Respiratory Acidosis
d. Respiratory Alkalosis
1009. Patient with intestinal resection and anastomosis develop infection abdomen tense and leakage
patient develop intestinal fistula PH=7.2, PCO2 15, HCO3=16, this change due to?

a. Bacterial \infection
b. Intestinal fistula

1010. Patient with wound at finger, take antibiotic then improve once patient stop antibiotic develop
swolling and pus and fever pathophysiologicaly changes?

a. Above cardiac output


b. Below Peripheral Venous Pressure
c. Above Arterial Permeability

1011. Patient with Abdominal operation postoperative day 5 patient hypotesion and oligurie and
afebrile what is cause of hypotension?

a. Decreased peripheral vascular resistence


b. Below cardiac output
c. Below glomerular filtration rate

1012. Best investigation to determine pancreatic duct?

a. C.T
b. EUS
c. ERCP
d. MRCP

1013. Acute pancreatitis go years with TLC 21x10, AST 244, bleeding normal LDH 440, other
investigation normal?

a. Severe attack
b. Mortality rate <5%
c. Need pancreatectomy
d. ERCP

1014. Most association with chronic pancreatitis?

a. Weight loss
b. Pain
c. Fever

1015. Patient with jaundice with epigastric pain with above alkalinphosphat, upper billirubin above
GGT, above GPT, above GOT, this condition more than 2 weeks, most common cause?

a. Pancreatic Tumour (Cancer of the head)


b. Cholangiocarcinoma
c. Choledocholithiasis

1016. Patient with Mucoccele gall bladder, patient DM, and IHD (ischemic heard disease) , Treatment?
a. Open cholecystectomy
b. Interval Cholecystectomy
c. Laparoscopic cholecystectomy
d. Percutaneous cholecystectomy

1017. Patient with (RUQ) pain, with dyspepsia with pain referred to shoulder with ultrasound show
thickness wall?

a. Acute cholecystitis
b. Peptic ulcer
c. Acute pancreatitis

1018. Patient palpable mass at RUQ with ultrasound dilated gallbladder, thin wall

a. Mucocle
b. Acute cholecystitis

1019. Appendicitis at day 10 post operation on removed of stitches profuse pus discharge. Patient a
febrile, WBC normal?

a. Irrigate with saline


b. Secondary suture
c. Povidon iodine gause at wound (dressing)

1020. 6 years child with upper respiratory tract infection, 5 days later develop distress and
hypercapnia and generalized abdominal pain?

1021. 90 years old with constipation, distended abdomen, bleeding per rectum fresh generalized
abdominal pain.
a. Appendicitis
b. Pneumococcal bacterial infection peritonitis

1022. Patient with Na 162 with diarrhea

a. Dextrose
b. Nacl
c. Ringer

1023. Two patient on admission, one right nephrectomy other left nephrectomy to avoid mistake

a. Sign In
b. Sign out
c. Login

1024. Patient post surgery travel at airport there is alarm show later forceps at abdomen to avoid?

a. Time out
b. Sign out
c. Collect instrument before closure

1025. Patient 12 years old with appendicitis his mother refuses operation

a. Connect with ethics committee


b. Do appendectomy
c. Refer to other hospital
d. Respect mother order

1026. Patient during resuscitation (CPR) , doctor give error order to medication and time of
medication, the nurse inform the doctor this order is wrong but the doctor neglect, what should you do
in this situation as you are a participant in CPR?

a. You agree with nurse


b. You neglect with other doctor
c. You repair the situation
d. Inform and take advice from supervisor

1027. Patient with peritoneal infection on clindomycine, culture result positive for organism not
sensitive to clindomycine but doctor not informed post 3 days patient died on review this error
discovered?

a. Nurse error
b. Bad system and safety
c. Surgical Error

1028. Child post trauma for operation?

a. Father should consented


b. Family should consented
c. Ethic committee

1029. Father with disturbed conscious for danger operation his son consented

a. Proceed for surgery


b. Not proceed
c. Consent all family

1030. Before renal transplantation, donor paid?

a. Cancel
b. Proceed
c. Refer to other hospital

1031. Businessman post care follow you at OPD (outpatient) patient need to be friend.

a. Refuse any relation may affect professional care


b. Accept
c. Inform committee ethics
1032. Patient homosexual with stenosed anus show ulcerated mass at anus you can’t do DRE this?

a. Anal cancer
b. Anal fissure
c. Sinus

1033. Patient come to E.R with dilated anus and bleeding with history of something entered to anus,
on laparotomy you find 6cm tear < 50% circumferences above anal verge with cup?

a. Primare repair
b. Resection anastomosis + colostomy

1034. Patient with mass above umbilicus you can attach abdomen free above umbilicus

a. Epigastric
b. Paraumbical
c. Inguinal

1035. Female post severe weight loss develop pain at inner thigh (severe sudden) with abdominal
distension?

a. Urgent laparotomy
b. C.T
c. Ultrasound
d. Antibiotic

1036. Patient at ER with all examination normal BP 100/70 except swelling at temporal bone # vault
of skull during transmission patient to C.T room BP become 70/40?

a. I.V fluid + anti biotic


b. I.V mannitol
c. Drill to skull to evacuate hematoma immediately
d. Arabic

1037. Most immediate life threatening?

a. Aortic injury
b. Tension Hemotorax
c. Pneumothorax
d. Tracheobroncial injury

1038. During repair of injured diaphragm 5cm defect by thoracoscopy patient suddenly distress
hypercapnia?

a. Airembolism
b. Pneumothorax
c. Hemothorax
1039. Femur bone # patient become distress + hypercapnia

a. Respiratory tract infection


b. Fat embolism

1040. Post operative 8 hours patient become fever 40 due to?

a. Wound infection
b. Normal inflammatory response

1041. Patient medically free prepared for inguinal hernia repair/

a. ASA l
b. ASA ll
c. ASA lll
d. ASA lV

1042. Patient controlled DM, for cholecystectomy?

a. ASA l
b. ASA ll
c. ASA lll
d. ASA lV

1043. Patient with wbc 15000, PCO2=25=, To=41, urine output 55ml/h, B/P 110/70?

a. SIRIS
b. Sepsis
c. Severe sepsis
d. Septic shock

1044. Patient with t037. Puls=70, PCO2=40, creatinine 0.8, wbc 8000?

a. Siris
b. Sepsis
c. Severe Sepsis
d. None of the above

1045. Patient with urine output 20ml/h, creatinine 7.5, wbc 20000, T040?

a. Siris
b. Sepsis
c. Severe Sepsis
d. None of the above

1046. Patient with BP 70/40, creatinine 8 PCO2 27, WBC 16000 T035?

a. Siris
b. Sepsis
c. Severe Sepsis
d. Septic shock

1047. Patient with T40, BP 90/70, pulse 140 with low urine output?

a. Sepsis
b. MOF (Multiple Organ Failure)
c. Septic shock
d. SIRIS

1048. K more in?

a. Gastric Juice
b. Saliva
c. Urine
d. Plasma

1049. Patient post gastric bypass 2 years ago, post dinner usually develop confusion and fullness and
epigastric pain condition relieved by drinking cola?

a. Biliary reflux
b. Food reflux

1050. Patient post trauma to abdomen stable with shoulder pain?

a. Discharge to home
b. Admit for further investigation
c. Follow up at outpatient clinic

1051. How to differentiate between femoral hernia and saphena

a. Dupler ultrasound
b. Ultrasound
c. C.T

1052. Hernia sac sho cecum part of the wall?

a. Spigelion
b. Sliding
c. Inguinal

1053. Hernia with vomiting abdominal distension not tender?

a. Obstruction
b. Strangulated
c. Inflamed

1054. Gastrosduosis associated with?


a. Intestinal Atresia
b. CHPS
c. Biliary atresia

1055. 3 months baby with non bilious vomiting ?

a. Biliary atresia
b. CHPS
c. Gastroschisis

1056. Diverticulitis with pericolic abscess 4x6 cm, best management?

a. Exploration
b. Antibiotic
c. Percutaneous drainage

1057. Diverticulitis with recto sigmoid best investigation?

a. colonoscopy
b. A=C.T abdominal with contrast
c. B-enema
d. Cystoscopy

1058. Most common fistula of diverticulitis disease?

a. Entero – enteric
b. Entero-colic
c. Colo –vesical

1059. 33 years old female with dyspepsia bleeding for 3 years ago for last month’s symptoms last
menstrual period 3 weeks ago initial investigation?

a. Colonoscopy
b. 24h ph monitoring
c. HCG

1060. Patient with crohn disease with small intestinal fistula, fluid for resuscitation?

a. Saline
b. Ringer lactate
c. DSW

1061. Patient with pyloric stenosis and dehydrates fluid for resuscitation?

a. Saline
b. Ringer
c. DSW

1062. Patient with burn gas method for saving airway?


a. Endotracheal Tube
b. Cricothyrotomy
c. Tracheotomy
d. Tracheal intubation

1063. Patient with pyloric stenosis, metabolic obstruction?

a. Hyperkalamic Hyponatremic Metabolic Alkalosis


b. Hypochloremic Hyponatremic Metabolic Alkalosis
c. Hypochlorenmic Hyponatremic Metabolic Alkalosis
d. Hypochlorenmic Hypokalamia Metabolic Alkalosis

1064. Sheep former with right hypochandrium pain HCV + V diagnosis?

a. HCC
b. Hydatid Cyst

1065. Patient with burn gas, method for saving airway?

A. Endotracheal intubation
b. Cricothyroid tube
c. Tracheastomy tube

1066. Female old patient complaining of I.O had history of constipation one month ago patient believe
constipation due to change diet, examination tender at right iliac fossa, empty rectum patient had
history of hysterectomy, diagnosis mostly?

a. Adhesion I.O
b. Volvulus sigmoid
c. Cancer

1067. Old patient with abdominal pain, rectal bleeding history of uncomplicated M.I one month ago,
xray showing thumb printing, diagnosis?

a. Mesenteric artery occlusion


b. Mesentric veins
c. Ischemic Colitis

1068. Patient with intrahepatic biliary dilatation CBD is normal, diagnosis?

a. PTC
b. MRCP
c. ERCP
d. C.T

1069. Patient with history of gastrectomy suffered from sweating, tumour one to two hours after
eating, resolves with eating diagnosis?
a. Early dumping
b. Late dumping

1070. Child with trauma with fracture ribs on left side, pain radiated to left shoulder affected origin?

a. Spleen
b. Diaphragm
c. Liver

1071. Old patient with right groin swelling reaching to upper part of scrotum the diagnosis?

a. Indirect Complete hernia


b. Indirect incomplete hernia
c. Direct incomplete hernia
d. Direct complete hernia

1072. Patient with inguinal hernia repair sac showing viable intestinal with Meckel’s D treatment?

a. Hernia repair
b. Hernia repair with mesh
c. Resection with anastomosis
d. Diversion colostomy

1073. Female patient with history of lap chole complain swelling below xiphisternum 5cm diagnosis?

a. Fatty hernia of linia alba


b. Umbilical hernia
c. Para umbilical
d. Stone

1074. Abnormal bleeding with normal prothrombin time occurs with?

a. Heparin overdose
b. Cirrhosis
c. Haemophilia
d. Von willebrand disease

1075. The most frequent major complication of blood transfusion is?

a. Volume overload
b. Hemolytic reaction
c. Hepatitis C
d. Hepatitis B

1076. Female patient H/O open cholecystectpmy one month ago, now presented with swelling at
lateral edge of the wound, intact skin mostly?

a. Incisional hernia
b. Wound dehiscence
c. Infection

1077. Female patient with pancreatic mass ERCP showing mucous coming from pancreatic duct,
diagnosis?

a. Resection and anastomosis


b. Wedge resection
c. Umbilicoplasty for proboscoid

1078. Male patient with pancreatic mass ERCP showing mucous coming from pancreatic duct,
diagnosis?

a. Pseudocyst
b. Intraductal papillary mucinousneoplasm
c. Serous cystedenoma

1079. 77 years old male patient with preauricular smooth swelling 3cm with free movement not
attached to skin or under lying tissue, mass above gradually over years but dropling of the angle of his
mouth, diagnosis?

a. Pleumorphic adenoma
b. Cystadenocarcinoma
c. Infection timpanium

1080. Female with pain during defecation nitrate relieves pain?

a. Pile
b. Fissure
c. Fistula

1081. Female patient with fissure, taking medication causing headache, what is?

a. Nitrate
b. Calcium channel
c. Botulinum

1082. Female patient with pain at calf muscule 2 days ago patient had history of atherosclerosis,
diagnosis?

a. Acute thromboembolic ischemic


b. Chronic lower limb ischemic
c. Chronic venous congestion

1083. Patient with obstructive jaundice found to have pancreatic head cystic lesion aspiration of the
cyst showed serous fluid with glycogen containing cells?

a. Pancreatic pseudocyst
b. Pancreatic cancer
c. Mucinous cystadenoma
d. Serous cystadenoma

1084. Case of pancreatic cyst with papillary projection?

a. Total pancreatetomy
b. Pancreatico duodenoctomy

1085. Patient with right hypochondrium pain, O/E there is tenderness in the right hypochonch and
fever, patient is dehydrated, investigatic show evidence of obstructive jaundice and above wbc,
ultrasound show dilatation of CBI and intrahepatic ducts and gallstone, what is next step?

a. C.t abdominal
b. ERCP
c. MRCP
d. PTC

1086. Bleeding peptic ulcer, best management (pt. Stable)?

a. Oversewing
b. Oversewing + Gastrojejunostomy
c. Oversewing + hughly selective vagatomy
d. Oversewing + antrectomy truncal vagatomy

1087. Bleeding peptic ulcer, patient not stable, best management?

a. Oversewing
b. Oversewing + Gastrojejunostomy
c. Oversewing + hughly selective vagatomy
d. Oversewing + antrectomy truncal vagatomy

1088. Post operation of aneurysm abdominal aortic artery 3 days ago occurred abdominal pain and
rectal bleeding, investigation?

a. Angiography
b. Upper GI endoscopy
c. Abdominal ultrasound
d. Sigmoidoscopy
e. Abdominal C.T

1089. 64 years old patient had elective operation for duodenal ulcer has diarrhea more than 20
movements per day, medication has been ineffective, what operation was most likely performed?

a. Antrectomy and billroth l anastomosis


b. Gastric surgery combined with cholecystectomy
c. Truncal vagatomy
d. Highly selective vagatomy
e. Selective vagatomy
1090. Female 54 years old, artist patient, develop epigastric pain that is relieved by antacid, she also
complains that her stool has change color and is black and tarry, what is the most important cause of
the entity presenting above other than H. Pylori?

a. Submucosal islet cells


b. Hyperglycemia
c. Diet
d. Acid secretion
d. Acute erosive gastritis

1091. Female patient 56 years old, chest xray showing posterior mediastinal mass, diagnosis is
oesophageal duplication cyst, which of the following is true regarding these congenital cysts?

a. Communication with the true lumen is uncommon


b. Malignant degeneration is common
c. Most cysts are symptomatic
d. Thorachoscopic excision is contraindicated

1092. 70 years old male patient, diagnosis is ulcer is greater curvature of the stomach by barium,
gastric analysis is maximal acid stimulation shows achlorhydria, what is the next step in management?

a. Antacid, hz blockers and repeat barium in 6-8 weeks


b. Proton pump inhibitor and repeat barium in 6-8 weeks
c. Prostaglandin E and repeat barium in 6-8 weeks
d. immediate elective surgery
e. Upper endoscopy with multiple biopsies at least 8 or 9 for the ulcer

1093. The most liable hernia to strangulate?

a. umbilical
b. Femoral
c. Inguinal
d. Epigastric

1094. 55 years old male patient, present with a massive UGI bleeding after, resuscitation endoscopy
is performed, no oesophageal varices, gastritis, or gastric ulcer are seen after copious irrigation, a pin
point lesion is seen near the GE junction, what can be said about this lesion?

a. It is a characinoid
b. It is related to alcohol use
c. It is exclusively a mucosal lesion
d. Surgery is from a sub mucosal
e. Bleeding is from a submucosal

1095. 11 years old child, RTA spleenic tear 3cm BP 90/60, management?

a. Conserve
b. Splenectomy
c. Splenorraphy
1096. About of parathyroid gland supply?

a. Superior parathyroid from interior thyroid artery


b. Inferioir parathyroid gland from inferior thyroid artery
c. All parathyroid gland from superior thyroid artery
d. All parathyroid gland from inferior thyroid artery
e. Superior from superior and inferior from inferior

1097. 22 years old male patient has undergone appendectomy for perforated appendicitis with
generalised peritonitis, 7 days post operation, his temperature continues to spike despite antibiotic
therapy, a C.T reveals large pelvic abscess, soon afterward he has bleeding from the mouth and nose
with increasing oozing from the surgical wound and all I.V puncture sites, what is the most likely
diagnosis?

a. Anaphylactic reaction to I.V dyr


b. Dissemenated intravascular coagulation
c. Antibiotic induced coagulopathy
d. Liver failure
e. Congenital bleeding disorder

1098. Patient with right shoulder pain + right side neck, at morning and inability to abduct right
shoulder above head?

a. Compression on root C4, C5


b. C.T mylography
c. Should not do x-ray in cervical spine

1099. GERD, oesophageal healing lead to?

a. Short oesophagus
b. Barrets oesophagus
c. Bleeding Oesophagus

1100. Patient with PH= 7.45, PCO2= 48 HCO3= 30

a. Respiratory acidious uncomp


b. Metabolic alkalosis part comp
c. Metabolic alkalosis full comp
d. Respiratory alkalosis uncomp

1101. Patient show intra billiary radical dilatatior by ultrasound with normal CBD, what is the next
step?

a. ERCP
b. MRCP
c. C.T abdominal
d. PTC
1102. Most common cause of acute pancreatitis is KSA?

a. Alcoholic
b. traumatic
c. Calcular
d. Idiopatic

1103. Male patient 30 years old, right hypochondrium swelling, tender ultra sound show cyst contain
inside daughter cyst , management?

a. Ambendazole
b. Ultrasound guided aspiration
c. Tinidazole
d. PAIR & mebendazole

1104. The most common cause of death related to a central venous catheter is?

a. Central vein perforation


b. Tension Pneumotorax
c. Air embolism
d. Catheter Embolism
e. Catheter related sepsis

1105. Hematocrit threshold for blood transfusion?

a. 30%
b. 25%
c. 21%
d. 18%

1106. A 55 years female with 3cm breast mass with palpableaxillary lymph nodes, modified radical
mastestectomy done, pathologic reveals 3.2cm infiltrating ductal carcinoma with 5 of 15 auxiliary
nodes positive, no metastosis, otherwise negative, laboratory normal, what is her tumour nodes
metastasis (TNM) stage?

a. T2N1M0
b. T1N2M1
c. T2N2M0
d. T4N1M0
e. T3N2M0

1107. The most likely cause of jaundice in patient with alcoholic pancreatitis is?

a. Cholecyst with gall stone


b. Pancreatic pseudocyst
c. Alcoholic hepatitis
d. Chronic pancreatitis

1108. What is true about solitary non parasitic cyst?


a. Echoic with ultrasound
b. Cyst acquired
c. Contain clear watery fluid
d. Contain pus

1109. Patient with benign insulinomas 2cm ini the tail of the pancreases, treatment?

a. Distal pancreatectomy
b. Pancreatecoduodenectomy
c. Enucleation of the tumour only
d. None of the above

1110. Cancer of the lower end of CBD with obstructive jaundice, enlarged gallbladder

a. Kehr’s sign
b. Couvoiser sign
c. Kosher sign
d. Chvostek sign

1111. Patient with two trial of unsuccessful Esophagoscopy presented in ER with chest pain, the
surgeon was worrisome about perforation, the best for confirmation is:

a. Barium swallow
b. Esophagus
c. C.T
d. Water soluble contrast swallows

1112. Patient female presented in ER with H/O years duration dysphagia is more liquids and
associated with H/O coughing and sore throat, diagnosis is?

a. DES (Diffuse oesophageal sposm)


b. Dysphagia Lusoria
c. Dysphagia Cardia
d. Esophagial cancer

1113. Patient with H/O 15 years of duration of duodenal ulcer presented with multiplevomiting, best
I.V fluid to be given it?

a. Ringer lactate
b. Normal Saline
c. 3% saline
d. Hetastarch

1114. Patient with severe epigastric pain generated, chest x-ray – air low diaphragm, on exploration
one cm perforation is found in stomach 3cm proximal to pylorus with 2cm induration around
perforation, treatment?

a. Billroth ll gastrectomy
b. Gastro-jejunostomy
c. Plorostomy with vagatomy
d. Close perforation with toilet

1115. Patient with upper GI two episodes of bleeding )/E abdominal distension (ascites), diagnosis?

a. Peptic ulcer
b. Oesophageal varices
c. Dieulafoy lesion
d. Duodenal ulcer

1116. The most common cause of colo-vesica fistula is?

a. Cancer
b. Chron
c. Diverticulitis

1117. The indication for surgeon in ulcerative colitis

a. Toxic mega colon

1118. H/O RIF pain and appendectomy done 1cm of characinoid is found post surgery, best?

a. Nothing
b. Right hemicolectomy

1119. H/O hemorrhoidectomy since 2 years the patient presented with difficulty defecation,
diagnosis?

a. Stenosis
b. Missed cancer

1120. Patient admitted in hospital because partial intestinal obstruction, what is indication of surgeon
during observation?

a. Hypotension
b. Persistent abdominal pain

1121. Patient with spheroctosis send to surgeon before doing surgery we need the test?

a. C.T
b. Fragility test
c. Ultrasound abdominal
1123. Liver segment is divided accordingly to?

a. Portal veins
b. Hepatic arteries
c. Hepatic veins
d. Biliary drainage
1124. History of lap chole since few days then bile collection in GB bed, definite treatment?

a. Reoperation
b. ERCP + stent

1125. Patient with right side right upper abdominal pain with fever, not responding to medication,
ultrasound abdominal shows G.B distended with thin wall, diagnosis?

a. Acute cholecystitis
b. Chronic cholecystitis
c. Cholangitis
d. Mucocele

1126. Billirubin?

a. Most of direct billirubin is a result of hemolysis


b. Indirect billirubin is a result of biliary ree obstruction
c. Most of billirubin in serum s indirect and carried by albumin

1127. Patient with uneventful lap chole since 6 months presented with jaundice since 1 mont with
MRCP foto: diagnosis?

a. Cholangio Ca
b. Stricture
c. Missed stone

1128. Female patient with H/O breast feeding stopped since 6 months presented with breast mass
FNA – whitish fluid, diagnosis?

s. Cancer
b. Galactocele

1129. Most common breast disease on lacting female is

a. Cancer
b. Mondo’s disese
c. mastitis
d. Abscess

1130. Young female with 3cm breast mass mobile and firm, FNA fibro adenoma, breast is?

a. Follow up
b. Excision biopsy
c. Danazol

1131. Patient with history of right hemicolectomy for T1a,N0,M0 colon cancer at present complain of
right lower quadrent pain with constipation, best investigation?
a. C.T
b. Colonoscopy
c. MRI

1132. Patient present with colitis and temp 3g2 pulses 95/min, RR 22/min, BP 90/60, diagnosis?

a. SIRS
b. Sepsis
c. Severe sepsis
d. Septic shock

1133. Patient with SIRS, retention is?

a. Peripheral temperature > 39 degree celcius


b. PCO2 > 32
c. WBC < 4000
d. Pulse < 80/min

1134. Patient female with long time BIL leg swelling and skin change, best investigation?

a. Dopplex ultrasound
b. Venography
c. ABPI

1135. Female patient with big toe bluish discoloration and severe pain with photo gangrene, ABPI is?

a. >1
b. 0.5-0.9
c. 0.5-0.6
d. < 0.3

1136. Female patient with leg adema, need to be treated by gradual peneumatic compression and
garment compression which are contraindication of ABI?

a. >1
b. 0.8-0.9
c. < 0.5

1137. Clotting factor correlating with Vit. K is?

a. Vll
b. Xl
c. X

1138. Patient with RTA a fracture femur only complain dysnea + petechine in upper chest + neck,
diagnosis?

a. Fat embolism
b. Tension pneumothorax
1139. Patient with hand injury with wire took tetanus completed since 12 years best to given?

a. Tetanus toxid
b. Immunoglobin
c. Tetanus + toxid + immunoglobingin

1140. Which type to shock needs vasopressors?

a. Hypovolemic shock
b. Distributive shock
c. Endocrine shock

1141. Patient in blast injury, have pneumothorax burn 40% , fracture femoral and sharpnels, injury
orientation?

a. Fracture frum
b. Burn
c. Sharpnels
d. Pneumothorax

1142. Patient in blast injury, best IVF is?

a. Normal saline
b. Ringer Lactate
c. 3% saline

1143. Patient with RTA BP = 110/70, pulse 106/min degree of the shock is?

a. l
b. ll
c. lll
d. lV

1144. Patient with hernia repair after few days complain fever, swelling in wound, pain with pus
discharge, diagnosis, treatment?

a. Penicillin G
b. Debridement
c. Debridement + penicillin G

1145. H/O infected wound in back (flank) presented with discharge and fever, today cardiac and dark
urine, diagnosis?

a. Toxic shock syndrome

1146. Patient with H/O urinary bladder cancer, surgery done and uretro-sigmoidostomy, EKG shows
interval T-Wave and U –WAVE?
a. Na low
b. K low
c. Na high
d. K high

1147. Patient with wide QRS, T peaked

a. High K

1148. Patient with known case of hypothyroidism presented with thyroid lumb and hoarseness and
dysphagia, 0/e – woody firm fixed thyroid swelling, best investigation?

a. FNA
b. Open biopsy
c. TSH
d. Iodine isotop

1149. Patient with thyroid mass, FNA shows amyloid, Treatment?

a. Lobectomy with neck dissection


b. Lobectomy with radiation
c. Total thyroidectomy with modified neck dissection
d. Total thyroidectomy with radiation

1150. Most common in MEN1 is?

a. Parathyroid
b. Thyroid
c. Adrenal
d. Pancreas

1151. Patient present with central obesity, striae and buffalo hump, best investigation is?

a. C.T brain
b. Urine cortisone

1152. Patient with inguinal swelling, presented inguinal swelling and herniplasty done without
opening hernia sec, type of this hernia is?

a. Direct
b. Indirect
c. Femoral
d. Ventral

1153. Patient with H/O inguinal swelling presented with painful inguinal swelling + abdominal
distension and constipation since day 1, no fever, diagnosis?

a. Irreducible hernia
b. Incarcerated hernia
c. Obstructed hernia
d. Strangulated hernia

1154. 35 weeks newborn underweight, presented with intestines outside of abdomen, right of
umbilicus, without covering diagnosis of gastroschisis done, which is the most common anormally?

a. Intestinal atresia

1155. 3 weeks old infant with multiple non-bilious vomiting, diagnosis?

a. Duodenal atresia
b. Pylorus hypertrophy
c. Annular pancreas
d. Malrotation

1156. 2 years old boy brought by his mother o/e testicles not found in scrotum and inguinal region,
best investigation to confirm diagnosis?

a. C.T
b. MRI
c. Laparoscopy

1157. Patient with chronic chest empyema, which is non-tubercolosis, best treatment associated with
antibiotics is?

a. Thorachocentesis
b. Decortication
c. Pleurodesis

1158. Patient with chest truma, chest tube is inserted, but still air leak associated with increasing
pneumomedistinium and subscutaneous emphysema, best investigation

a. Esophagiscopy
b. Brochoscopy
c. Brochography

1159. Photo of old age female patient with chest skin lesion (like bcc), skin biopsy margins is?

a. 0.5mm
b. 1mm
c. 2mm
d. 10mm

1160. Patient with old burn about 15 years, presented with unhealed ulcer, which type of ulcer?

a. Curling ulcer
b. Cushing ulcer
c. Marginal ulcer
d. Marjolin ulcer
1161. Young patient with pilonidal swelling since childhood, presented with pain + swelling and
discharge white material caseating, by PR and pressing on sacral bone – white material discharge,
diagnosis?

a. Dermoid cyst
b. Sacrococcygeal teratoma

1162. Which cancer is most common making ovaries seeding (BIL)?

a. Stomach
b. Colon
c. Pancreas

1163. Capacity of coupling in laparoscopy?

a. Direct thermal injury


b. Injury due to defect in insulation of instrument
c. Injury due to energy charging in non-detective insulation instruments

1164. Seat belt in chest and abdomen, mostly cause injury to?

a. Retroperitonium
b. Pancreas
c. Hollow viscus
d. Rim of pelvis

1165. Patient presented in ER with lower bleeding, o/e pigmentation peritoneal, hands and feet, with
family history of colon cancer in his father, diagnosis?

a. Peutz-jeghers

1166. Helicobacter pylori

a. Most common in gastric ulcer than D.U?


b. Cause gastric malignancy
c. Positive in 70-80% in oesophagus and duodenal in normal people

1167. Characteristic of septic shock?

a. Hypotension
b. Tachycardia
c. Worm hand
d. Tachypnea

1168. Multiple Trauma patient on exploration multiple pancreatic laceration and pancreatic duct
tears proper management?

a. Pancreaticoduodenstomy
b. Distal pancreatectomy
c. Ext. Drainage

1169. 22 years old medical student is seen by the student health service prior to beginning school,
routine labs are drawn, the medical student immunized aginst hepatitis B in childhood will have which
hepatitis profile?

a. Hbs Ab +, Hbs Ag + , Hbc ab +


b. Hbs Ab +, Hbs Ag+, Hbc Ab-
c. Hbs Ab -, Hbs Ag-, Hbc Ab-
d. Hbs Ab+, Hbs Ag-, Hbc Ab-

1170. Upper RQ pain male patient 44 years old present in ER with fever, H/O lap chole since 4 days,
ultrasound reveals abdominal collection in gallbladder fossa and hepatorenal pouch, MRCP shows
leakage from cystic duct stump, proper management?

a. ERCP + stent
b. Open surgery and lagation of tear
c. Surgical Drainage

1171. Medulary thyroid Cancer?

a. Most common than papillary thyroid cancer


b. Tend To metastasis by lymphatics

1172. Male with grave disease after thyroidectomy fever, tachycardia, tachypnea, hypotension?

a. Thyroid storm
b. Septicemia
c. Infection

1173. Most effective treatment of flail chest is?

a. Adhesive tapes
b. Fixation with wire and scrow
c. Epidural catheter and analgesic

1174. In TNM and staging of breast cancer according to fibro adenoma what is incorrect?

a. Mostly benign
b. Mostly mobile
c. Diagnosis by mammogram
d. May be huge

1175. Likely associated with vascular injury most least?

a. Mid ulnar bone fracture


b. Mostly mobile
c. Diagnosis by mammogram
d. Maybe huge

1176. The diagnosis of oesophageal achlasia is confirmed by?

a. Brid’s beak appearance on barium swallow


b. Subatmospheric intraluminal esophageal pressure on manometry
c. Endoscopic evidence of distal esophageal
d. Failure of lower oesophageal sphincter relaxation on manometry
e. Hyperperistalisis of the body of the oesophagus

1177. Trauma caesarean section is indicated if the fetus is viable and?

a. The mother is unstable and cardiac arrest is anticipated


b. 5 minutes of cardiopulmonary resuscitation were successful
c. 5 minutes of cardiopulmonary resuscitation were unsuccessful
d. Cardiopulmonary resuscitation has just been initiated

1178. Presentation of patient with neurogeneric shock?

a. Hypotension, bradycardia, warm hands


b. Hypotension, tachcandia, warm hands
c. Hypotension, Tachycardia cool and clammy hands
d. Hypertension, Tachycardia cool and hands
e. Hypertension, tachcandia, warm hands

1179. After vigorous exercise an athlete develops pain on dorsiflexion of the foot and decreased
sensation in the first web space appropriate action should be?

a. Color duplex scan and immediate heparinization


b. Leg evaluation ice packs and nonsteriodal anti inflammatory medication
c. Immediate fasciotomy
d. Immediate femoral arteriogram

1180. The most characteristic metabolic abnormality in glucagoma is?

a. Hypoglycaemia
b. Hypo aminoacidemia
c. Hypercholesterolemia

1181. Tissue lost almost always results from legation of the?

a. Popliteal artery
b. common femora; artery
c. Superficial femoral artery
d. Portal vein

1182. Female 34 years old is G4P2, complain abdominal pain, Nausea vomiting during her first
trimester of pregnancy, HCG>100000, perlvic ultrasound shows snowstorm, which of the following is
true?
a. Risk of malignancy 5%
b. Risk of malignancy 10%
c. Risk of malignancy 20%
d. Risk of malignancy 40%

1183. Post operation of thyroidectomy fever, hypertension, tachycardia?

a. Thyroid storm

1184. Patient with BP= 170/90, T3,T4,TSH are normal, HB is normal to control BP should send the
patient to?

a. Cardiology
b. Pulmonary
c. Endocrinology
d. Hematology

1185. Landmark to differentiate between indirect and direct hernia?

a. Scrotum
b. VAS deferens
c. Pubic tubercle
d. internal ring

1186. Para umbilical hernia, vomiting, constipation, sac irreducible, not tender, type of hernia?

a. Obstructed
b. irreducible
c. Incarcerated
d. Strangulated

1187. Hernia with crescentic shaped umbilical is?

a. Umbilical
b. Epigastric
c. Para umbilical
d. Spigelion

1188. Gastric cancer with right gastric 3 LNs needs resection?

a. D1
b. D2
c. D3

1189. Gastric Cancer reach musculan’s Propia?

a. T1
b. T2
c. T3
d. T4

1190. Alcoholic old man suddenly vomited and developed with chest pain and dysphagia diagnosis?

a. Oesophageal perforation

1191. Post UGi endoscopy, physician was worried about oesophageal perforation, what is the best
investigation to confirm it?

a. Siluted gastrographin with thin film barium

1192. Minimal safety margin for SCC?

a. 2mm
b. 4mm
c. 10mm
d. 15mm

1193. RTA, echymosis, midabdomen at site of seatbelt, possible injury?

a. Spleen
b. Liver
c. Intestine
d. Masentric

1194. Oesophageal cancer with dysphagia L.N at celiac artery, the treatment?

a. Stent
b. Esophagectomy
c. Chemotherapy

1195. Gastric cancer (adenocarcinoma) removed completely during surgery but after operation and
wit histologic examination there was positive margin because it was seen by microscopy?

a. R0
b. R1
c. R2
d. R4

1196. Primary energy in acute injury?

a. Fat
b. Muscle
c. Glycogen
d. Keton bodies

1197. Female with 4cs she has big intra abdominal mass, post lapasoscopic, most common
compilation?
a. Infection
b. Wound failure
c. Adhesion
d. Fistulization

1198. Ileostomy, best fluid?

a. Ringer lactate

1199. Child with pyloric stenosis?

a. Start with normal saline before give K

1200. Myeloproliferative with splenomegaly. What is the best investigation?

a. C.T with contrast

1201. Gastrochisis associated with?

a. Intestinal

1202. Child with splenomegaly, jaundice yellow sclera dark urine and positive H/O family both
parents are have blood dysrasia, HB 9.2, best treatment?

a. Splenectomy
b. Corticosteroid
c. PRBCs transfusion
d. Whole blood transfusion

1203. Splenic injury with 3cm lacertion, which grade?

a. l
b. ll
c. lll
d. lV

1204. Organ resistant to infection?

a. Intra abdominal
b. Pulmonary
c. Blood

1205. Patient with controlled diabetes?

a. ASA l
b. ASA ll
c. ASA lll
d. ASA lV
1206. Patient with RTA BP = 90/60, HR=30, RR=26, best fluid?

a. Blood
b. Colloid
c. Crystalloid

1207. Patient 35 years presented with breast mass in upper router quadrant with non cyclic pain –
next investigation?

a. FNA
b. Mammogram
c. MRI breast

1208. Child has wound in right feet with wood broke – red, hot, tender, most organism?

a. Clostridium tetani
b. Clostridium perfringens
1209. Fluid resuscitation for infant

a. 50-60
b. 60-65
c. 65-70
d. 70-80

1210. Gallbladder stone asymptomatic and patient has history of enteric fever best treatment?

a. Cholecystomy
b. Conservative

1211. Gallbladder stone suggested by ultrasound + dilated CBD without stone patient 60 years old,
best investigation?

a. ERCP
b. MRCP
c. CT

1212. Gallbladder + fever + pain in right UQ tenderness, she started on antibiotics after 3 days no
improved, best investigation?

a. ERCP
b. MRCP
c. CT

1213. What is the cause rectal prolapsed in patient with 40 years old?

a. Pelvic floor weakness


b. Long term history of diarrhea of constipation
c. Previous injury to the anal or hiparea
1214. Patient 40 years old with rectal prolapsed, best management?

a. Altemeier
b. Delorm
c. Thiersch

1215. Sigmoid volvulus, next appropriate management?

a. Flatus tube
b. Nasogastric tube

1216. Female 30 years old she has history of diarrhea and blood and mucos since 5 years back and her
mother has the same history from long time, most appropriate diagnosis?

a. Diverticulitis
b. Crohn’s diseases
c. Ulcerative colitis
d. Colon carcinoma

1217. Patient was on anti biotic, developed with diarrhea, diagnosis done pseudomembranous colitis,
treatment?

a. IV vancomycrine
b. Oral metronidazole
c. Ciprofloxacine

1218. Patient with anti biotic and rectum – hyperaemic, yellow fluke, what is the best treatment?

a. Ciproxin
b. Ceftazidime
c. Metronidazole
d. Triclabendazoe

1219. ITP, patient underwent splenectomy but still platelet is 50000. What is the treatment?

a. Steroid
b. Azathioprine
c. immunoglobuline

1220. Trauma to right hypochiindrium most liable organ to injury?

a. Spleen
b. Stomach
c. Liver
d. Lung

1221. Enlarge thyroid gland with normal T3, T4, TSH high + thyroid antibodies, what is the treatment?
a. thyroxin
b. Radioactive
c. Antithyroid

1222. Patient with dyspepsia, polunia hyper natremia and hypokalemia?

a. Aldosterone excess

1223. Patient with hypernatremia, hypokalemia + mass at upper pole of kidney?

a. Adrenal carcinoma
b.
1224. Child presents at emergency with wound in his head, with him was his brother and nurse maid
but father and mother were ni way starting to come at the hospital?

a. Start to stitches the wound


b. Cover the wound and wait father and mother

1225. Cardiologist his patient need operation in heart but the patient doesn’t know the procedure
with manycomplication, what should do?

a. Start to work without knowing the patient


b. You should inform the patient of all the details of the process
c. Abandoning the operation

1226. Occurred medical mistake and you want to learn the students, what is the appropriate method
to learning?

a. Make a meeting about mortality and morbidity


b. Root cause analysis
c. Must tell anyone from hospital about what happened
d. Make interrogating about what happened

1227. Patient underwent for haemorrhoidectomy after two days he came at OPD for follow up, he was
with his son, doctor asked the nurse to nurse the patient, which one of the following exceeds the
privacy of the patient?

a. Examine in presence of nurse


b. Examine without his son
c. Examine without white sheet

1228. Male patient C.T abdominal hepatocelular carcinoma, what next investigation for work up?

a. C.T brain
b. MRI abdominal and pelvis
c. PET scan
d. Bone scan
1229. Patient*4 years old, his wife 29 years old, he present to ER with vomiting nausea, abdominal
pain, distended and tympanic, rebound, Tdegree 38.5, RR/20. HR = 101, H/O two M.I and CVA 5 years
ago, after CVA he has trouble in speaking, he doesn’t have power of attorney, patient need operation,
his wife refused , his son demands that everything possible be done to save his father’s life. What level
of care appropriate?

a. Attempt intervention by local district court judge


b. Initiation of hospice care
c. Medical and supportive care only
d. Urgent surgical exploration

1230. Patient undergo for surgery in abdominal neoplasm he discusses it with his surgeon describes
the procedure to him in detail, what is still necessary for the patient to be able to give informed
consent to this procedure?

a. A medical screen including ECG and coagulation panel


b. A psychiatric assessment
c. Consultation with a family member of the patient coagulation panel
d. A nonsurgical second opinion

1231. Which of the following provides the main energy source during critical illness/injury?

a. Skeletal
b. Liver
c. Kidney
d. Gut

1232. Cervical sympathectomy is least likely to improve?

a. Hypetidosis
b. Scleroderma
c. Causalgia
d. Frostible

1233. Girl came in ER with her mother, the father out of the town diagnosis was appendicitis?

a. Prepare for surgery


b. Take verbal consent from the mother
c. Wait until the father comes back
d. Explain the procedure to the girl and take the mother signation

1234. Female face skin lesion (pic) about 65 years old beside nose (cheek) for excision (basal cell
carcinoma), safety

a. 0.5mm
b. 1mm
c. 2mm
d. 10mm
1235. Basal cell carcinoma originating from?

a. Pluripotent
b. Folicular
c. Papillary

1236. Picture with ulcer at porterior lateral aspect of knee, femal 35 past history of burn at same site
20 years back with scar, what is the proper management?

a. Wide local excision


b. Wide local excision with sentinel LN biopsy
c. Wide local excision with sentinel LN dissection
d. Above knee amputation

1237. Female 75 years old current attacks of loss of conscious with blurring vision, no H/O chronic
illness?

a. C.T
b. CTA
c. Doplex

1238. Female old age with diabetic uncontrolled came in ER with blackish discoloration at left foot, 2
weeks duration, no palpable anterior and posterior tibial artery , leucocytosis, CTA complete occlusion
popliteal artery, no collaterals, no run off, management?

a. Bypass graft
b. Below knee amputation
c. above knee amputation

1239. Male 72 years old post prandial pain 15 minutes, ancrexia, most appropriate diagnosis?
a. Colon cancer
b. Chronic mesenteric disease

1240. Most common cause of chronic pancreatitis?

a. Cholelitiasis
b. Alcohol
c. Hyperlipedemia

1241. Most common site of biliary carcinoma?

a. Distal part of CBD


b. Hilum
c. Cystic
d. Intrahepatic duct

1242. Patient upper right quadrant pain, fever, yellow sclera, billirubin high?
a. Pancreatitis
b. Cholangitis

1243. Upper abdominal pain recurrent, steatorrhea, diabetic alcoholic, with picture of CBD, multiple
dilatation pancreatic duct?

a. CBD stone
b. Pancreatic carcinoma
c. Chronic pancreatitis
d. Pancreatic cyst

1244. Female anemia, pallor, jaundice splenimegaly, target cell at blood smear?

a. Thalassemia
b. Sphreocytosis
c. Sickle cell

1245. Myloproleferative disease with spleen enlargement?

a. Ultrasound
b. C.T with contrast
c. C.T

1246. Post repair after AAA repair 2 days, there is discharging bloody from rectum, you suspect
colonic ischemia, what do you do?

a. Mesentric angiograpy
b. Sigmoidoscopy
c. Upper Git. Endoscopy
d. C.T

1247. Patient RTA, urine output 25ml/h, cool clammy sweating, mild anxious, RR=25, BP=N, pulse
=110, what is the volume of bloody was loosed?

a. <15%
b. 15-30%
c. 30-40%
d. >40%

1248. During laparotomy cholecystomy, you injured the CBD and told the patient post operative.
What does this mean?

a. Disclosing an apparent complication


b. Disclosing complication
c. Disclosing error
d. Disclosing an apparent error

1249. Study conduct to see the effect of earphone, Bluetooth and brain tumour?

a. Cross sectional
b. Cohort
c. Case control
d. Correlation

1250. A diagram of surgical resident and non surgical resident followed for 4 years outcome of
depression study, what is the type of study?

a. Cohort
b. Series
c. Control study
d. Navy Tape test

1251. Study on effect of dairy products on group with colon cancer and group no colon cancer, wat is
the type of the study?

a. Cohort study
b. Control study
c. Section Study
d. Correlation

1252. Which one is not correct in four principles of Beauchamp and childness?

a. Beneficence
b. Non-maleficence
c. Respect for persones
d. Justice

1253. Patient gave you a gift and you refused, next day patient attempted suicide because she felt
rejected, what you should do in the situation?

a. Accept the gift and give it to patient charity


b. Explain for patient that it’s unethical to accept gift then refuse
c. Write paper on OPD door that gifts are unacceptable
d. Accept the gift for yourself

1254. Patient RTA, he had fracture laeg, needs foot amputation, after you explained the wise and
benefits of procedure, he agreed because he hears voices, telling him to do it?

a. Find another decision maker because patient is incompetent


b. Proceed to the life saving surgery (since it’s an emergency intervention)
c. Involve another surgeon to confirm the importance of the surgery benefits

1255. Seizure patient taking drug decrease attention and he has a bus driver your action?

a. Respect confidentiality and don’t tell anymore


b. Tell no to drive
c. Tell his supervisor
1256. A study Conducted to evaluate medication among depressed patients, they were not taking
their medications regularly?

a. Unacceptable study because it affects drug efficacy


b. Acceptable because the patient signed the consent voluntary
c. Unacceptable because we cannot take consent from depressed patients

1257. HIV newly diagnosed worry to until his wife and asks not to tell his wife, what will you do?

a. Tell his wife mother and ask her to tell her daughter
b. Tell his wife
c. Protect the patient confidentiality
d. Reassure him and offer your assistance to until his wife

1258. Police man came with a guy to ER while you are the physician asking you for alcohol screening
but the guy refused, what do you do?

a. Respect confidence of the patient


b. Respond to a policeman request
c. Call a supervisor
d. None of the above

1259. One of the following is a medical error?

a. Mistaken Diagnosis or prescription


b. Negligence
c. Management plan not completed as intended
d. All of the above

1260. Parameters to compare by chi-square?

a. Age and BP
b. Smoking and BP
c. Gender and smoking
d. Heart disease and smoking

1261. Mean of cholesterol in Arab area, American and European people’s?

a. Chi-square
b. Student T-test
c. ANOVA
d. Correlation

1262. Diabetes mellitus among health care workers is 30 out of 100 doctors were diabetic as
compared to 50 out of 200 nurses, remember (doctors-nurses) ( diabetic – non diabetic), which is he
most appropriate statistical test?

a. T-test
b. ANOVA
c. Chi-square
d. Correlation

1263. Best study to assess the relationship between, Bluetooth headsets and brain cancer?

a. Retrospective cohort
b. Case control
c. Ecological
d. Correlation

1264. 30/500 babies who were exposed to Di-ethyl-stilbestrol in utero were followed up to for 20
years and the occurrence of cancer in this group was compared with that of 150 babies not exposed to
DES, over the same period of the time. What is the study design?

a. Cohort
b. Case control
c. Cross sectional
d. Correlation study

1265. Gastrinoma location the investigation of the choice?

a. MRI
b. C.T
c. SRS
d. MIBG

1266. Patient very malnourished, what is the element be normal if the patient for surgery?

a. Vitamin C
b. Copper
c. Folic acid
d. Ca++

1267. Patient with myeloproleferative disease and splenomegaly, next step?

a. C.T
b. Bone marrow biopsy
c. Splenectomy
d. Observation

1268. Abnormal bleeding with normal prothrombin time occurs with?

a. Heparin overdose
b. Cirrhosis
c. Haemophilia
d. Von willebrand’s disease

1269. 24 hours after admission in ICU, a post operative patient has bright blood through the
nasogastric tube, all of the following have shown efficacy in preventing stress gastric except?
a. Sucralfate
b. Proton pump inhibitors
c. Enteral diet
d. Histamin – 2(H20 receptor antagonist
e. Antacids

1270. 41 years old female undergoes complex repair of a deep laceration in her hand, when
removing the dressing of post operative day 2, a large clot with mild surrounding erythema is
encountered, which of the following statement regarding the inflammatory phase of wound healing?

a. The complement component C5a and platelet factor attract neutrophils to the wound
b. The presence of neutrophils to the wound is essential for normal wound healig
c. It lasts up to 24 hours after the injury

1271. A 70 years old man with sepsis has PH 7.18, which of the following is true regarding his
metabolic acidosis?

a. The most common cause of excess acid is prolonged nasogastric sunction


b. Metabolic acidosis results from the loss of bicarbonate on the gain of fixed acids

1272. A 70 years old man with pyloric obstruction secondary to ulcer disease is admitted to the
hospital for resuscitation, after 1 week of prolonged vomiting, what metabolic disturbance is expected?

a. Hypokalemic, hyperchloremic metabolic acidosis


b. Hyperkalemic, hyperchloremic metabolic alkalosis

c. Hyperkalemic, hyperchloremic metabolic aciodosis

d. Hypokalemic, hyperchloremic metabolic alkalosis

1273. Arrest of cell cycle in the metaphase in the action of?

a. Cychlophosphamide
b. Methotrexate
c. Doxorubicin
d. Vincristine

1274. Succinylcholine is contraindicted for?

a. Patient with hepatitis


b. Parotidectomy
c. Celiotomy 2 weeks after spinal cord injury
d. Thyroidectomy

1275. Splenic artery aneurysm?

a. Can present with double rupture penemoneon


b. Is more common in males
c. Is seldom multiple
1276. One week after Coumadin therapy, a patient severe pain in right leg with areas of skin necrosis,
appropriate action is?

a. Transfusion of fresh frozen plasma


b. Intravenous Vit. K
c. Stopping Coumadin and starting heparin

1277. Mixed venous saturation is increased in?

a. Hypovolemic shock
b. Septic Shock
c. Cardiogenic shock
d. Neurogenic shock

1278. Paraneoplastic syndrome is most commonly associated with?

a. Small cell lung cancer


b. Bronchia carcoid
c. Bronchoalveolar carcinoma

1279. The endothelial cells are the source of?

a. Factor ll
b. Factor V
c. Factor Vll
d. Factor Vllll

1280. Which of the following is likely to be multiple?

a. Gastrinoma
b. Insulinoma
c. Somatostatinoma
d. Gluconoma

1281. The mechanism of action of heparin is?

a. Potentiation of anthithrombin lll action


b. Inhibition of cyclo-oxtgenase
c. Direct inhibition of thrombin

1282. Which of the following amino ester?

a. Lidocaien
b. Tetracaien
c. Bupivacaien
d. Mepivacaien

1283. Vit. K?
a. Is water soluable
b. Is absorbed in proximal small bowel
c. Requires bile salts for absorption
d. Is required for factor Vlll synthesis

1284. Hyponatremia is a complication of topical application of?

a. Silver nitrate
b. Povidone iodine
c. Sulfamylon
d. Bacitrcin

1285. Radical cystectomy + uretro sigmoid anastomosis

a. Hypokalemia

1286. Repeated vomiting + distal small bowel obstruction

a. Hypokalemia

1287. Gastric outlet obstruction metabolic problem

a. Hypokalemia, hypochloremia metabolic alkalosis with aciduria

1288. Hypertrophic pyloric obstruction (CHPS) stenosis

a. Metabolic alkalosis
b. Best inv. – ultrasound

1289. Gastroschisis accompanied with?

a. Intestinal atresia

1290. Infant continues crying, on examination, swelling scrotal firm, bluish in color, not fluctuant no
groin in abdominal swelling?

a. congenital hydrocele
b. Testicular tertoma
c. Testicular torsion
d. strangulated inguinal hernia

1291. Old age male with lesion lateral to eye, lesion show pigmentation and lateral ulceration?

a. S.C.C
b. B.C.C
c. Melanoma
d. Marjolin ulcer
1292. B.C.C?

A. Pluripotent cells

1293. Best investigation to detect GERD?

a. Ph monitoring

1294. Symptoms of GERD + barium swallow shows that gastroesophageal junction above diaphragm
but the stomach in place?

a. Heller mytomy
b. Partial fundolication
c. Nissen fundoplication

1295. Female patient abdominal distinction and vomiting on examination structure 6 cm from
ileocecal valve no passable (only one structure)?

a. Strictutoplasty
b. Right hemicolectomy
c. Resection and anastomosis
d. Resection and ileostomy

1296. Male patient knew divericulosis symptoms and signs of diverticulitis best investigation?

a. C.T abdomen

1297. Male patient old age with pneumaturia, what is the commonest cause of this condition?

a. Diverticulosis
b. Cancer
c. Crohn’s
d. Radiation

1298. Male patient presented to clinic with abdominal pain and weight loss for 3 months, he
underwent right hemicolectomy 3 years ago for cncer of colon, what is the best investigation?

a. Colonoscopy
b. C.T abdomen
c. PET
d. Ultrasound

1299. Barium swallow show corkscrew esophagus with normal K.S manometry?

a. Diffuse oesophageal spasm


b. Achlasia

1300. Vigorous Vomiting 4 hours ago and retrostenal pain?


a. Heorhave syndrome

1301. Male patient cancer oesophagus T8H2M0 undergoing stent directly after procedure patient
complain of left chest pain, peripheral cyanosis, absent air entry on left side?

a. Pneumothorax
b. Oesophageal injury (perforation)
c Surgical Emphysema

1302. Patient has nausea and vomiting came with left chest pain peripheral cyanosis absent air entry
on left side?

a. Pneumotorax
b. Rupture oesophagus

1303. Patient ulcerative colitis abdomen pain, fever distention, tachycardia, hypotension, not relieved
by non surgical treatment, what you do?

a. Right hemicolectomy
b. Total proctocolectomy
c. Subtotal colectomy, ileostomy and mucos fistula

1304. 35 years old male with family history of FPC presented with abdominal pain and diarrhea for
months, stool analysis – loose, black in color, blood and mucous?

a. Amebiasis
b. Colon cancer
c. Meckel diverticulitis
d. hirschsprung’s disease

1305. Patient with dyspepsia endoscopy – ulcer at 1st part of duodenum, biopsy confirms H. pylori
infection and no malignancy. What is the priority for treatment?

a. Eradication of H. pylori
b. Decreasing acide secreation

1306. RUQ pain, tenderness, fever rigor, CT show cystic liver lesion, aspiration reveal chocolate like.
What is organism?

a. E. Coli
b. Echinococcose
c. Enteamoeba histolytica

1307. Patient is fall of building accident, kept down for 4 hours. What is the electrolyte disturbance?

A Hyperkalemia

1308. Patient is fall from building kept down for 3 hours, has dark urine. What is the cause of dark
urine?
a. Myoglobinuria

1309. Patient with thyrotoxicosis (Grave’s Disease). What to give to decrease vascularity 7 days
before of?

a. Beta blocker
b. I 131
c. Lugol’s iodine
d. Antithyroid drugs

1310. Female 22 years old age, neck swelling, hard?

a. Medullary ca
b. Papillary Ca
c. Follicular Ca
d. Anablastic Ca
e. Riedel Thyroiditis

1311. Female 25 years old presented with thyroid swelling her mother had thyroid swelling and died
of stroke last year?

a. Medullary
b. Toxic Goitre
c. Riedel thyroiditis

1312. The best treatment for achalasia?

a. Nissen Fundoplication
b. Heller myotomy and Dor. Fundoplication
c. Collis gastroplasty and nissen fundoplication
d. Toupet fundoplication
e. Cruroplasty and nissen fundoplication

1313. The best treatment for GERD?

a. Nissen Fundoplication
b. heller myotomy and Dor. Fundoplication
c. Collis gastroplasty and nissen fundoplication
d. Toupet fundoplication
e. Cruroplasty and nissen fundoplication

1314. Best treatment for GERD and short oesophagus?

a. Nissen Fundoplication
b. heller myotomy and Dor. Fundoplication
c. Collis gastroplasty and nissen fundoplication
d. Toupet fundoplication
e. Cruroplasty and nissen fundoplication
1315. The best treatment for GERD and weak oesophagus?

a. Nissen Fundoplication
b. heller myotomy and Dor. Fundoplication
c. Collis gastroplasty and nissen fundoplication
d. Toupet fundoplication
e. Cruroplasty and nissen fundoplication

1316. The best treatment for rolling hiatus hernia?

a. Nissen Fundoplication
b. heller myotomy and Dor. Fundoplication
c. Collis gastroplasty and nissen fundoplication
d. Toupet fundoplication
e. Cruroplasty and nissen fundoplication

1317. 35 years old, non cyclic pain, palpable auxiliary L.N, what do you do?

a. MRI
b. Mammogram
c. C.T
d. FNA

1318. Female with breast mass palpable, aspiration revealed clear fluid cytology no malignant cells
but the mass not completely regressed what to do next?

a. Mammogram
b. Core biopsy
c. Ultrasound
d. Nothing to do

1319. Patient marjolin ulcer (post burn) at the posterior aspect of knee (poplitealfossia) scheduled for
excision, what the best method of closure of wound after excision?

a. Split thickness graft


b. Flap
c. Primary
d. Leave for secondary intension

1320. Patient post I.M injection in buttock 7 days, swelling, hot, tender, after incision and drainage.
What to do?

a. Closure
b. Late closure
c. Leave open

1321. Patient discovered to have pancreatic tumor in CT, shehave diarhea, investigation show
hypokalemia?
a. Vipoma

1322. Patient came with confusion, pancraeatic, tumor, blood glucose below, normal insulin level,
what to do?

a. Enuclaetion
b. Pancreatectomy

1323. Patient RTA, Lipase above level, what is the best investigation for diagnosis of disease?

a. CT abdomen

1324. What is the best to diagnosis of mucinous cyst adenocarcinoma?

a. High CEA level in aspiration

1325. Patient post 4 C.S, large incisional hernia for lap repair, what is the common late complication?

a. Wound failure
b. Mesh migration
c. Adhesion

1326. Patient came to clinic suspected to have right inguinal hernia, what is the best method to
diagnosis?

a. Clinical examination
b. C.T
c. Ultrasound

1327. Richter hernia gangrenous during hernia repair, you found that lesion, what is the common
presentation of that lesion?

a. Constipation
b. Diarrhea
c. Toxemia

1328. Patient have painless bleeding per rectum, protoscopy reveal 3 mucosal elevation at 3,7,11
oçlock above dental line that descend below dental line on straining and there reduced spontaneously?

a. Grade l
b. Grade ll
c. Grade lll
d. Grade lV

1329. Myeloproliferative disorder patient has left upper quadrant pain referred to shoulder, on
clinical examination he is suspected to have huge spleen, what investigation to do to confirm?

a. Ultra sound
b. C.T abdominal
c. Diagnostic Lap
d. MRI

1330. During incisional hernia, injury of colon, colon repaired primary, what to do to hernia?

a. Primary closure
b. Mesh repair
c. Tension suture
d. Delay 4 days then proceed to mesh repair

1331. Most common hernia to strangulate?

a. Femoral
b. Inguinal
c. Epigastric
d. Spigelion

1332. Hernia above umbiliculus, you can feel linia alba between the swelling and the umbiliculis?

a. Umbilicus
b. Paraumbilical
c. Epigastric

1333. 25 years old male foot baller come by sudden swelling inguinal region, tender, scrotum normal ,
no bruises?

a. Torsion of spermatic cord


b. injury of inguinal canal post wall
c. Congenital sac
d. Ruppture of information hypogastric vessels

1334. Lesion cause osteomyelitis?

a. Felon
b. Paronychia
c. Tenosinovitis

1335. Patient post surgical exploration for pyogenic liver abscess presented with abdominal
distention, fever tachycardia, tachypnea, oliguria, urinary catheter connected to monometry pressure
is 28?

a. Evaculation of collection
b. Laparotmy and colestomy
c. Laparatomy and laparostomy

1336. Patient tachycardia tachypnea, elevated urea and creatinine, P140, BP= 80/30 temperature 37?

a. Sepsis
b. Severe sepsis
c. SIRS
d. Septic shock

1337. Patient trauma to spinal cord paraplegic which parameter indicated if has neurogenic shock?

a. Bradycardia
b. Hypotension
c. Tachycardia
d. Trauma on spinal cord

1338. Patient with Above T0, High pulse, low BP, what is the cause of hypotension?

a. Low cardiac output


b. Increased systematic vascular resistance
c. Decreased arteriolar resistance

1339. Trauma- tension pneumothorax, what is the initial management?

a. Needle Thoracostomy
b. Tube Thoracostomy (Chest Tube)

1340. Trauma to chest examination revealed centralised trachea paradoxical movement of 6,7,8, left
rips, x-ray show, 6,7,8, left ribs and blurred cosh phrenic angle, PO2 92% parameter is normal, what is
the definitive management?

a. Intubation and meconical ventilation


b. Tube thoracostomy
c. Adhesive strap

1341. 65 years old came to clinic with 3 years history of bleeding per rectum with and without stool,
urgency 3 years ago has coronary heartsurgery and on antiplattet, 6 years ago has been treated by
radiotherapy for cancer of prostate, what is the cause of his condition?

a. Radiation proctitis
b. Colon Ca
c. Bleeding due to antiplatelet drugs

1342. 25 years old female has breast lumb 3cm proved by ultrasound to be fibro adenoma, what to
do?
a. Excision
b. Mammogram
c. Biopsy
d. Self examination every month

1343. Case long – liver hydatid cyst, what is the layer of excision?

a. Germinal layer and laminated layer


b. Laminated layed and adventitia
c. Germinal layer and adventitia
d. Adventitia & capsule

1344. Stab wound to the neck 2cm below angle of mandible with progressive hematoma, Low blood
pressure, tachycardia?

a. Exploration
b. Angiography
c. Conservative and follow up

1345. A case of pancreatitis with age 60, WBC 21000, glucose high, High LDH 450 AST 244?

a. Severe case of pancreatitis


b. Mortality < 5%
c. Amylase is indicator of severe disease

1346. Case of iatrogenic oesophageal perforation, diagnosis?

a. Water soluble swallow

1347. A case of achalasia, investigation?

a. Barium swallows

1348. A case of parameter of septic shock?

a. Sepsis + org. dysfunction


b. SIRS + infection
c. Severe Sepsis + hypotension
d. Pulse > go + wbc> 12000

1349. Case of male alcoholic with post vomiting esophageal tear, management?

a. Conservative and follow up


b. Thoracotomy and oesophageal repair

1350. A case of periampullary tumours, the nest staging?

a. Endolumincal ultrasound
b. C.T
c. ERCP
d. MRCP

1351. Male patient gastric carcinoma in pylorus with celiac lymph node “Advanced Tumour” with
gastric outlet obstruction.

a. Total gastrectomy
b. Subtotal gastrectomy
c. Gastrojejunostomy
1352. Patient male on treatment for arthritis complening of epigastric pain and right iliac pain wiith
signs of peritonitis?

a. Perforated peptic ulcer

1353. Male patient Felty syndrome?

a. Peritonitis – splenomegaly – rheumatoid


b. Rheumatoid – splenomegaly – pancreatitis
c. Splenomegaly – neutropenia – peritonitis
d. Neutropenia – splenomegaly – rheumatoid

1354. Rectal tumour 5cm from the anal verge, advanced the best treatment?

a. AP resection
b. Neo adjuvant therapy

1355. Patient male with oesophageal cancer the best method for palliative treatment?

a. Laser ablasion
b. Stent

1356. Female patient with breast mass 1cm by mammogram stellate mass with irregular border
micro calcification and distraction of surrounding structures by true cut biopsy there is hyperplastic
tissue, management?

a. Excisional biopsy by wire detection preoperative


b. Modified radical mastectomy
c. Simple mastectomy

1357. Female patient with diffuse enlargement of the thyroid with exophtalmy with T3,T4 low and
TSH high, the most common cause?

a. Activation of lymphocytes B4
b. TSH stimulator antibodies
c. TSH inhibitor antibodies

1358. Patient female with past history of subtotal thyroidectomy for multi nodular goitre, now there
is recurrent of goitre with T3, T4, normal and TSH high elevated, what is the cause?

a. Hypothyroidism

1359. Female patient with solitary thyroid nodule- FNA – follicular cell, management?

a. Total thyroidectomy
b. Subtotal
c. Lobectomy
1360. Female patient with open cholecystectomy with exploration the CBD, post operative there is
drain and T tube after 2 days, drain removed after 4 days, T tube 800ml/day bile the cause is?

a. CBD injury
b. Missed stone
c. CBD structure

1361. Patient male 52 years old case pancreatic mucinous adenocarcinoma, only one valid of the
following for diagnosis?

a. CEA > 192ng/ml & Ca 19-9 > 2900


b CEA > 50ng/ml & Ca 19-9 > 900
c. CEA >100ng/ml & Ca 19-9 >700

1362. Another case of pancreatic duct stone, best management?

a. ERCP
b. Ultrasound
c. X-ray

1363. Another case of pancreatic Mucinious adenocarcinoma with aspiration?

a. Amylase
b. CEA
c. Billirubin

1364. Preterm with cystic hygroma need to?

a. Endo tracheal intubation


b. O2 mask
c. Tracheotomy

1365. Preterm baby with neck swelling translumination positive?

a. Cystic hygroma

1366. Preterm with gastroschisis associated with?

a. Intestinal atresia

1367. Preterm baby with oesophageal atresia and distal T.E fistula on x-ray?

a. Coiled Ryle tube in proximal oesophagus


b. No air in GIT

1368. Patient female with regurgitates chewed not digestive food with swelling at the neck,
treatment?

a. Oesophageal myotomy
b. Cricopharyngeal myotomy
c. Diverticulectomy

1369. Female patient with umbilical swelling the cause is?

a. Congenital
b. Traumatic
c. Inflammatory

1370. Male patient with inguinal hernia the swelling si tension and severe tenderness?

a. Obstructed
b. Stragulated
c. Irreducible

1371. Male patient irreducible inguinal hernia during operation long structure 5cmx1.5cm inside the
sac, treatment?

a. Diverticolectomy
b. Intestinal resection
c. Resuscitation

1372. Male patient with lower limb trauma followed by infection with high fever and on x-ray air S.C,
Management?
a. Antibiotic
b. Debridement
c. Resuscitation

1373. Bacteria with previous concept that it is communal but it new pathogen?

a. Staphylococcus
b. pseudomonas
1374. Female patient with gluteal abscess with discharge containing gram positive Rods, Treatment?

a. Penicillin G
b. Clindomycin

1375. Female patient with nodule beside the nose the best management?

a. MOh’s Surgery

1376. While you are assisting your senior at doing major operation, a surgical mistake happened that
leaded to an intra operative bleeding but the surgeon could handle the situation and the operation
completed without any harm to the patient you were responsible for writing the operation details,
What you will do?

a. Don’t mention the event to keep relation with your senior


b. Write all operative details as it happened
c. Inform the medical admin

1377. Patient with gastric ulcer with vomiting 4 times/ day which path physiology changes with
alkalosis?

a. Hyperchloremic
b. Hypokalemic
c. Hypernatremic

1378. Patient with abdominal succession splash, which path physiology occur?

a. Hypokalemic hypochloremic metabolic acidosis


b. Hypokalemic hypochloremic metabolic alkalosis

1379. Baby with (CHPS)?

a. Hypokalamic hypochloremic metabolic alkalosis

1380. Patient with loose stool and lower abdominal pain with bleeding red/rectum – path physiology
changes occurs to?

a. NA
b. K
c. Mg
d. Cl

1381. Patient with intra peritoneal infection with ABG changes (PCO2)25, (HCO3) 17, (PO2) 85, (PH)
7.2?

a. Metabolic acidosis
b. Metabolic alkalosis
c. Respiratory acidosis
d. Respiratory alkalosis
1382. Patient with intestinal resection and anastomosis develop infection abdomen tense and leakage
patient develop intestinal fistula ph 72, Pco2 15, Hco3 16, this change due to?

a. Bacterial infection
b. Intestinal fistula

1383. Patient with wound at finger take antibiotics then improve, once patient stop antibiotics
develop swelling and pus and fever pathophysiological changes?

a. High COP
b. Low PVR
c. High arterial Permeability

1384. Patient with abdominal operation, post operative day 5, patient hypo tension and oliguric and a
febrile, what is cause of hypotension?

a. Decrease peripheral vascular resistence


b. Low COP
c. Low Glomerular filtration rate

1385. Best investigation to determine pancreatic duct?

a. C.T
b. EUS
c. ERCP
d. MRCP

1386. Acute pancreatitis 60 years old with TLC 21 x 10ᶾ, AST 244. Bleeding normal, LDH 440, other
investigation normal

a. Severe attack
b. Mortality rate <5%
c. Need pancreatectomy
d. ERCP

1387. Most associated with chronic pancreatitis?

a. Weight loss
b. Pain
c. Fever

1388. Patient with jaundice with epigastric pain with alkaline phosphatise high, billirubin high, GOT
and GPT high, this condition more than 2 weeks, most common cause?

a. Pancreatic tumour (head)


b. Cholangiocarcinoma
c. Choledocholithiasis
1389. Patient with mucole gallbladder patient DM, ischemic h. Disease, treatment?

a. Open cholecystectomy
b. Laparoscopic cholecystectomy
c. Interval cholecystectomy
d. Percutaneous cholecystestomy

1390. Patient with RUQ pain with dyspepsia with pain referred to shoulder with ultrasoundshow
thickness of gallbladder thin wall

a. Mucocele
b. Acute cholecystitis
c. Peptic ulcer

1391. Patient palpable mass at RUQ with ultrasound dilated gallbladder thin wall

a. Mucocele
b. Acute Cholecystitis

1392. Appendicitis at day 10 post operation on removal of stitches prefuse pus discharge, patient a
febrile?

a. Irrigate with saline


b. Secondary suture
c. Povidone iodine gauze at wound dressing
d. Nothing

1393. 6 years child with repiratory tract infection (upper), 5 days later develop distress and
hypercapnea and generalized abdominal pain

a. Appendicitis
b. Pneumooccal bacterial infection
c. Peritonitis

1394. 90 years with constipation, distended abdomen, bleeding per rectum fresh, generalised
abdominal pain

a. Perforation
b. Intussusception
c. Dieulafoy

1395. Patient Na162 with diarrhea

a. Dextrose
b. Ringer
c. Nacl

1396. Patient on admission on right nephrectomy other left nephrctomy to avoid mistake
a. Sign In
b. Sign out
c. Log in

1397. Patient post surgery travel at airport there is alarm show later forceps at abdomen, to avoid?

a. Time out
b. Sign out
c. Collect instrumental before closure

1398. 12 years old patient with appendicitis his mother refuse.

a. Connect with ethics committee


b. Do appendectomy
c. Refer to other hospital
d. Respect mother order

1399. Patient during resuscitation (CPR) doctor give error order to medication and time, the nurse
inform the doctor this order is wrong but the doctor neglect, what to do?

a. Nurse administration
b. Nurse apply doctor’s order
c. Nurse tell other doctor
d. Nurse refuse a doctor’s order

1400. Patient peritoneal infection on clindomycine culture result positive for organism not sensitive
to clindamycin but the doctor not informed post 3 days patient died, on review this error discovered

a. Nurse error
b. bad system and safety
c. Surgical error

1401. Child post trauma for operation

a. Father should consented


b. Family should consented
c. Ethic committee

1402. Father with disturbed conscious for danger operation his son consented

a. Proceed for surgery


b. Consent all family
c. Not proceed

1403. Before renal transplantation, donor paid, what to do?

a. Cancel
b. Refer to other hospital
c. Proceed
1404. Business man post care follow you at OPD (outpatient), patient need to be friend

a. Refuse any relation may affect professional care


b. Accept
c. Inform committee ethics

1405. Patient homosexual with stenosed anus show ulcerated mass at anus, you cant do DRE, this?

a. Anal cancer
b. Anal fissure
c. Sinus

1406. Patient come to ER with dilated anus and bleeding with history of something entered to anus,
on laparactomy you found 6cm tear above anal verge with glass cup, tearing was < 50% circumferem?

a. Primary repair
b. Resection anastomosis + colostomy

1407. Patient with mass above umbilicus you can attach abdomen free above umbilicus

a. Epigastric
b. Paraumbical
c. Inguinal

1408. Female post severe weight loss develop pain at inner thigh (severe sudden) with abdominal
distension.

a. Urgent laparotomy
b. C.T
c. Ultrasound
d. Antibiotics

1409. Patient at ER with all examination was normal BP 100/70 except swelling at temporal bone £
vault of skull, during transmission patient to C...t room Blood Pressure become 70/40, management?

a. I.V fluid + antibiotics


b. Drill to skull to evacuate hematoma immediately
c. IV mannitol

1410. Most immediate life threatening?


a. Aortic injury
b. Tension hemothorax
c. Pneumothorax
d. Tracheobronchial injury

1411. During repair of diaphragm injured 5cm defect

1500. The first investigation for patient with liver trauma (non-operative treatment), which of the
following

a. Ultrasound
b. Abdominal x-ray
c. C.T
d. MRI

1412. Femur bone £ patient become distress and hypercapnia ?

a. Fat embolism
b. Respiratory tract infection

1413. Post operative 8 hours patient become fever 40 due to?

a. Wound infection
b. Normal inflammatory response

1414. Patient medically free prepared for inguinal hernia repair?

a. ASA l
b. ASA ll
c. ASA lll
d. ASA lV

1415. Patient with temperature 40, Blood Pressure 90/70, Pulse 140, with urine output low?

a. Sepsis
b. SIRS
c. Septic Shock
d. MOF

1416. K more in

a. Gastric juce
b. Saliva

1417. Patient controlled DM for cholecystectomy

a. ASA l
b. ASA ll
c. ASA lll
d. ASA lV

1418. Patient post gastric bypass 2 years ago post dinner usually develop confusion and fullness and
epigastric pain condition relieved by drinking 7up.

a. Biliary reflux
b. Food reflux

1419. Patient post trauma to abdominal stable with shoulder pain

a. Discharge at home
b. Follow up at outpatient clinic

1420. How to differentiate between femoral hernia and saphena varix

a. Dupler Ultrasound
b. Ultrasound
c. C.T

1421. Hernia sac show cecum part of the wall

a. Spigelian
b. Sliding
c. Inguinal

1422. Hernia with vomiting abdominal distension not tender?

a. Obstruction
b. Stragulated
c. Inflamed

1423. Diameter of oesophagus in adult normal in what mm?

a. 10mm
b. 20mm
c. 30mm
d. 45mm

1424. Air enema pressure Intraoperative

a. 80 mmhg
b. 100 mmhg
c. 120 mmhg
d. 140 mmhg

1425. Risk of infection after operation of open viscous or pus?

a. 10%
b. 15-25%
c. 40%
d. 5%

1426. Duodenal atresia (double bubble sign and diagnosed already – search for

a. Down Syndrome

1427. Gastroschisis

a. Intestinal atresia

1428. 2 days after cholecystectomy – subphreni collection, by HIDA showed leakage from cystic duct

a. Lap clipping of duct and drain

1429. Hernia below & lateral to public tubercle

a. Femorala

1430. Old man had hernia repair and open prostate surgeries – now hernia recurrence

a. Open hernia with mesh


b. TAPP
c. TEP
d. Open hernia without mesh

1431. 4mm malignant melanoma safety margin?

a. 1-2 cm
b. 2-3 cm
c. 2cm
d. 4cm

1432. Oesophageal perforation after endoscopy

a. Barium
b. Water soluble

1433. X-ray show for patient old female after trauma abdominal distended, xray – gaseous distension
with air fluid level

a. Ileus

1434. 65 year old female with it big toe severe pain and black in color, history for 12 hours, on
examination no pulse, what appropriate management?

a. MRI
b. Anticoagulant
c. Antibiotics
d. Analgesics

1435. 25 years old man during right inguinal hernia repair found to have small bowel content in the
sac with Meckel divide, what to do?

a. Hernia repair
b. Hernia repair and diverticulectomy
c. Hernia repair and segmental resection of the ileum

1436. What is the causative organism of cholongitis

a. Staph aureu
b. E-coli

1437. Best investigation to evaluate pancrease duct?

a. MRCP
b. ERCP
c. C.T
d. MRI

1438. Patient 47 years old female patient gall stones and thin wall and distended GB by ultrasound
there is suspicion of mass in distal C.B.D, what to do next?

a. MRCP
b. ERCP
c. biliary scintagraphy
d. C.T

1439. Most common sarcoma in lower limb?

a. Leiomyosarcoma
b. Fibrous histcytoma
c. Fibrosarcoma

1440. Child brought to E.R in distress and his condition wprsened he is to be admitted for critical care
but the parents refused to sign, what to do?

a. Try to convince the parents


b. Dont admit
c. call another relative to sign
d. Admit anyway if his condition becomes life threatening

1441. Female patient with no co morbidity presented with 3 hours history of abdominal pain, relieved
by IV morphine she has history of 3 episodes of vomiting and 1 episode of melena, what investigation
to order to reach the diagnosis

a. Endoscopy
b. WBC
c. Abdominal x-ray

1442. Burn patient with face erytema and blisters anterior chest and bilateral arm burn, he is
tachycardic and hypotensive and agitated, what to give?

a. H2 blocker
b. blood transfusion
c. Measure PCO2 and PO2 and C0
d. Intubation

1443. After a massive blood transfusion, what is the usual metabolism abnormality

a. Coagulopathy hyperthermia and calcium high, K high


b. Coagulopathy hyperthermia and calcium high, K low
c. Coagulopathy hyperthermia and calcium low, K high

1444. What is the abnormality in gastric outlet obstruction?

a. Cl low, Na low, paradoxical aciduria


b. Cl low, Na low, alkaline urea
c. Cl low, K low, metabolic alkalosis
d. Cl low, K high, metabolic acidosis

1445. How do you identify the patient in OR?

a. Log in
b. Sign in
c. Sign out
d. Time out

1446. (Photo of anal region with soft tissue mass or skin dog), patient with chronic anal pain bleeding
PR pruritis known case of Hodgkin Lymphoma under went treatment long time ago, see picture patient
as anal soft lesion, what is the cause?

a. chronic anal fissure


b. HPV
c. Immunocompromised
d. Intraepithelial neoplasia

1447. Same anal picture shows soft tissue mass patient is known to have condyloma acuminate, what
is the treatment?

a. Intraepithelial neoplasia
b. Biopsy with wide local excion
c. Flap
d. Chemotherapy
1448. Police officer came with gunshot injury to thigh has skin infection features of crepitus, red
tender skin, please mention treatment?

a. Penicillin G
b. Clindamycne
c. Tetracycline
d. Impenem

1449. Trauma to chest patient came with tension pneumothorax, needle thoracostomy done, what is
the next step?

a. Chest x-ray, Pa view


b. Tube thoracostomy
c. PEEP
d. Leave him alone

1450. Cyclist met with accident came to ER with eye opening to command, verbal to confused, motor
response withdraw to pain, GCS score?

a. 9
b. 8
c. 11
d. 14

1451. Motorcyclist met with RTA, e3, V3, M4, what is the management?

a. Oxygen mask
b. Intubation endotracted (ET)
c. Orotracheal intubation
d. PEEP

1452. Sharpnel injury to neck

a. Explored
b. Conservative management

1453. Gastric outlet obstruction with vomiting, what is the treatment?

a. Nasogastric tube
b. Open and miotomy
c. Laparoscopic miotomy
d. Antiemetic

1454. Upper gastrointestinal endoscopy (UGI) was done, and they found a 5 cm lesion involving less
than 50% circumference, C.T showed lymph node along celiac access, what is the management?

a. Radiotherapy
b. Chemo radiotherapy
c. Esophagectomy
d. Follow up

1455. Critical patient wit gangrene of foot and in coma, family says don’t operate him as they would
rather see him die in one piece than have amputation done, what to do?

a. Respect family orders as they are his valid guardians


b. Amputate
c. Inform Ministry
d. Inform court of law

1456. Melanoma classification called as?

a. Breslow
b. Ranson
c. Gleason
d. Rockall

1457. Melanoma thickness of 1mm, what is the normal margin clearance required?

a. 20mm
b. 10mm
c. 40mm

1458. Post thyroid surgery patient develops carpopedal spasm, what is your management?

a. I.V calcium
b. Oral calcium
c. Phosphate and calcium
d. Calcium and Vitamin D

1459. What parameter is required to calculate osmolarity?

a. Glucose
b. calcium
c. Phospate
d. Magnisium

1460. Patient has hypoglycaemia polyuria gave values of K, Na, Ca, PCO2, what is the pathology?

a. Adrenal tumour
b. Pheochromocytoma
c. Adrenocortical excess

1461. Patient operated on wrong side, what wold have prevented this?

a. Senior registrar mark operation site


b. Nurse mark
c. Nurse at time out confirm site
1462. Swelling with impulse with cough third the way from xiphoid to umnilicus

a. Paraumbilical hernia
b. Epigastric hernia

1463. What is medical error?

a. Patient while walking on crutches


b. Pain after abdominal surgery
c. Bleeding after nephrectomy

1464. Female with history of dysphagia and chest pain relieved with loud belch, what is commonest
complication?

a. Barrett’s oesophagus
b. Gastric volvulus
c. Gastric perforation
d. GERD

1465. Scenario of perineal fistula in patient with recurrent lower abdominal pain and diarrhea, what
is the best investigation?

a. Barium enema
b. MRI
c. C.T
d. Small bowel follow through

1466. Patient presented with upper GIT bleeding best initial treatment?

a. Sclerotherapy of bleeding varices


b. Octriotide
c. Corticosteroid
d. Vasopressin

1467. Same day after 8 hours of operation (Chole or cancer rectum ) developed SIRS featurs,
commonest cause

a. Normal response cause


b. Wound infection

1468. Child with pyloric obstruction, what is the best resuscitation fluid?

a. Normal saline

1469. Commonest metabolic abnormality in pyloric obstruction

a. CL low, K low, Met. Alka


b. Na low, Met alka.
1470. Main physiology of tissue expansion?

a. Rupture of connective tissue


b. Mechanical stretch of skin
c. Mitosis

1471. BCC arise from?

a. Pleuripotent cells
b. Actinic keratoses

1472. Patient presented with features of acute ischemia 6P, best initial management?

a. Heparin I.V
b. Angiography
c. Exploration

1473. Esophageal perforation by caustic in child?

a. Giversion and jejunostomy


b. Surgical repair and drainage
c. Conserve

1474. Post dyspnea and fever after abdominal operation treatment?

a. Chest physiotherapy
b. Antibiotics
c. Bronchoscopy

1475. After abdominal operation, foly catheter pressure 28mmgh, what is the management?

a. Laparotomy and drainage


b. Laparotomy and Laparostomy

1476. Midline neck swelling, move with deglutition and tongue protusion commonest statement?

a. Pulmonary embolism
b. Reaction effect

1477. Post operation after 7-8 days developed severe dyspnea, cyanosis

a. Pulmonary embolism
b. Reaction effect

1478. After surgery of peptic ulcer at gastro duodenal junction, patient complain pain and distension
after meal 1-2 hours and vomiting, what is the diagnosis?

a. Duodenal stump
b. Afferent loop
c. Recurrence of ulcer

1479. Patient with left irreducible inguinal swelling with partial small bowel obstruction, mostly?

a. Littre hernia
b. Mydle hernia
c. Richter
d. Amyand hernia

1480. C.T showed fluid collection through and around sigmoid, what is the cause?

a. Perforation
b. Diverticulitis

1481. Intraoperative for intestinal obstruction with gross soiling found cancer sigmoid?

a. Hartman
b. Resection and anastomosis
c. Stoma

1482. Colon polyp rsected with colonoscopy snaer and found cancer invading submucosa, what is the
treatment?

a. Nothing
b. Resection

1483. C.T show liver cyst with echinococcus (may vibration), what is the best investigation?

a. ELISA
b. Aspiration

1484. Picture left sided portal hypertension

a. Splenectomy
b. TIPS
c. DSRS

1485. Child came to ER need urgent airway management and parents refuse to sign consent?

a. Not to do anything
b. Save child without consent
c. Look for other public guardian to sign

1486. terminal patient, what to tell him?

a. terminal but will get better


b. have nothing
c. Tell his family to tell him
d. Tell him the truth
1487. Gallbladder polp presented with pseucocyst after 12 weeks, what to do?

a. Internal drainage
b. Cholecystectomy

1488. Most important feature of chronic pancreatitis?

a. Pain
b. Weight loss
c. DM diabetes mellitus

1489. 18 years old female with breast fibro adenoma bf FNA, what is the treatment?

a. Excision
b. Breast self exam every month

1490. Hip pain after from bone malignant metastasis, what is the treatment?

a. Opioid
b. Radiotherpy
c. Bisphophonates

1491. Swelling and pain in groin after playing sport by 2 hours (penis and scrotum are normal), what
is the cause?

a. Rupture inferior epigastric artery


b. Tear posterior wall in canal
c. Congenital hernia sac

1492. What is Euthanasia?

a. Doctor prescribe medication to end patient life


b. Doctor tell patient something to terminate his life
c. Give pain killer that shorter patient life

1493. Myasthenia gravis caused by?

a. ACTH receptor blocked


b. ACH receptor blocked
c. CRH receptor blocked
d. FSH receptor blocked

1494. In breast cancer mass more than 5 cm , no stage?

a. 2A
b. 2B
c. 1B
d. 3A
1495. Patient on TPN, need blood transfusion, he developed comatosed, what is the cause?

a. Hyperkalemia
b. Hypocalcemia
c. Hypoglycemia
d. Hyperglycemia

1496. Patient on TPN, doctor ask the nurse to give the patient 2u RBC, the nurses by fault stop TPN
and give the 2u on same line of TPN, after 2 hours patient become comatose, what is the cause?

a. Electrolyte imbalance
b. Using the same line of TPN

1497. Patient with liver trauma (severe injury) which one from the following is not allowed to be
ligated?

a. Cystic artery
b. Proper hepatic artery
c. Parenchmal vessel
d. Some branching veins

1498. In the liver trauma when we have should ro applied ballon Tamponade?

a. Hepatic avulsion (grade v injury)


b. Central gunshot wounds that pass through both lobes
c. Post operative perihepathic abscess
d. deeper laceration

1499. In the liver trauma when we can use electrocautery?

a. Avulsion (grade v injury)


b. Mild bleeding from raw liver surface
c. Severe liver injury
d. None of the above

1501. Patient liver trauma moderate bleeding management?

a. Hepatic artery ligation


b. Portal clamping
c. Perihepatic packing
d. Parenchmal vessel ligation

1502. The most protocols allow for observation in liver trauma?


a. Up to 10 units of blood
b. up to 8 units of blood
c. Up to 6 units of blood
d. Up to 4 units of blood
1503. Truncal vagatomy for duodenal ulcer, should be combined with?

a. Pyloroplasty
b. Gastrojejunostomy
c. Pyloroplasty and Gastrojejunostomy

1504. 6 months after cholecystectomy MRCP image show defect in CBO, diagnosis?

a. CBD stone
b. Hepatic duct structure
c CBD injury
d. Cholangiocarcinoma

1505. Bile found in peritoneum and cystic duct injury occur post cholecystectomy management?

a. T. Tube
b. ERCP
c. Primary Repair

1506. Aspiration of pancreatic sac reveals fluid rich in carbohydrate, diagnosis?

a. Pseudocyst
b. Mucinous cystadenocarcinoma
c. Serous cystadenocarcinoma

1507. A.S.A?

a. Controlled DM 2
b. Uncontrolled DM, HTN

1508. Patient with blunt trauma fails to resuscitate?

a. Ultrasound
b. C.T
c. DPL
d. Lap diagnosis

1509. Blunt trauma with pain in upper abdomen and kehr sign, injured organ
a. Stomach
b. Spleen
c. Liver
d. Pancreas

1510. Liver abscess operated then, fever pain, distension, oliguria U.B manometry 28?

a. Antibiotic
b. Drainage
c. Re explore and drainage
d. Re explore and laparotomy

1511. How to measure ACS by device?

a. Connected to urinary bladder

1512. Piles can’t reduce manually which grade?

a. l
b. ll
c. lll

1513. ERCP shows dilation and multiple stones in tail treatment?

a. Resection of tail
b. Lat pancreojejunostomy
c. Whipple
d. Conserve

1514. Most common presentation in chronic pancreatitis?

a. Weight loss
b. Diabetes
C. Steatorrhea
d. Abdominal pain

1515. Patient severe paint with H/O recurrent abdominal pain 20 years ago become addict to
morphine with DM steatorrhea with normal lipase?

a. Acute pancreatitis
b. Acute on top of chronic
c. Chronic
d. Autoimmune

1516. Patient with high suggestion of acute pancreatitis

a. Ultra sound
b. C.T
c. Amylase
d. MRCP

1517. Treatment of acute anal fissure?

a. Conservative
b. Operation

1518. Lat. Sphenterotomy for anal, after one week occurred incontinence of fluid stool, DRE no
sphenter tone to termine anatomic.
a. Pudendal nerve study
b. Endoanal ultrasound
c. Sigmiodoscopy

1519. Melanoma in arm depth move than 4mm resection

a. 10mm
b. 20mm
c. 30mm

1520. Masses on both ovaries cancer

a. Stomach
b. Pancreas
c. GB
d. Colon

1521. Explosion with shrapnel pneumothorax and fracture and burn 35% which in secondary injury?

a. Pneumothorax
b. Fracture
c. Burn
d. Infection

1522. Mass over the kidney with hyper natremia hypokalemia, polydipsia?

a. Aldosterone
b. Glucocorticoid

1523. Pseudomembranous colitis best diagnostic

a. ELISA
b. Endoscopy
c. Stool Cytotoxin

1524. Diarrhea after ciprofloxacin for one week BP=100/60, treatment?

a. I.V fluid
b. Oral metrinidazol
c. I.V vencomycine

1525. Patient with blunt chest trauma with normal breath sound and distended neck veins fast reveal
cardiac tamponade, patient arrive BP 70?

a. Er. Thoracotomy
b. Median Sternotomy
c. Pericardiocentesis
1526. Cancer oesophagus with palliative stent do endoscopy – perforation occurred?

a. Conservative
b. Esophagectomy

1527. T tube maximal amount in drain in earl post operative days

a. 50
b. 100
c. 250-500
d. 500-1000

1528. G00, abnormality

a. Met acidosis
b. Resp. acidosis
c. Met. Alkalosis with high Na.
d. Met. Alkalosis with low K

1529. Severe pain after hard stool

a. Abscess
b. Fissure

1530. Drug cause headache in treatment of anal fissure

a. Lidocaine
b. Nitroglycerin
c. Bruten

1531. Most diagnostic for obstruction jaundice?

a. ALT
b. ALP
c. Direct and indirect billirubin
d. Amylase

1532. Patient with known GB disease present with acute attack, investigation?

a. Ultrasound
b. C.T
c. X-ray

1533. Palpable thick GB with fever, pain rigor, wbc 18000

a. Acute cholecystitis
b. Mucocele
c. Empyema
1534. Patient 60 years with mass at RIF 4x4, tender, fixed with olgurea

a. Appendiceal mass
b. Appendiceal cancer
c. Cecum

1535. Breast mass in 35 years old male 4x4 with small L.N in axila – next?

a. Mammogram
b. FNA
c. MRM
d. Reassure

1536. Patient with TPN need blood, nurse gave it through same line after stop TPN then coma

a. Electrolyte
b. Hypoglycemia

1537. Patient after 5 days post lap chole pain fever distension absent intestinal

a. Sound with normal BP


b. Septic shock
c. Bile leak
d. Bowel injury

1538. Intra hepatic dilation with normal extra, level of obstruction

a. Confluence
b. Right hepatic duct
c. CBD

1539. 10 days post operative, (for removal T-tube) after clamping 24 hours if no pain or jaundice

a. Observation
b. Removal
c. Reclamping 12 hours

1540. Patient 65 years normal with acute limb ischemia to determine source of emboli

a. ECG
b. Echo
c. Duplex lower limb
d. MRI angiograpy

1541. Lateral boundary of triangle of doom in lap hernia

a. Ductus deferens
b. iliac vein
c. Genitofemoral nerve
d. Gonodal vessels

1542. Surgeon mak surgical error appendectomy but correct it, operation pass successfully?

a. Inform patient and apologise


b. Inform director and apologise
c. No inform
d. Write in operative sheet

1543. Female patient conscious good mental status has cancer need operation, patient when he
knows about complication he refused operation, but his family ask you to do the operation?

a. Perform the demand the family


b. Perform the demand of the patient
c. Transfer him to another doctor

1544. Patient female with morbid obesity, she undergo for bariatric surgery, surgeon when he
discussed with her he observed she not completely good mental status.

a. Need consent of the husband


b. Refer to psychiatric hospital
c. Proceed
d. Offer diet control

1545. Old male, lesion lateral to eye shows pigmentation and lateral ulceration?

a. Melanoma
b. B.C.C
c. S.C.C

1546. Uncontrolled diabetic, uncontrolled hypertension?

a. ASA l
b. ASA ll
c. ASA lll

1547. Basal Cell carcinoma arises from?

a. Inter follicular epidermis


b. Pluripotent stem cells
c. Follicular epidermis
d. All of the above

1548. GCS score for patient with eye – open with pain verbal – inappropriate word motor – withdraw
from pain?

a. 7
b. 8
c. 9
d. 10

1549. MELD score in assessment of severity of chronic liver disease patient considered candidates for
liver transplantation?

a. MELD score 8
b. MELD score 10
c. MELD score 17
d. MELD score 15

1550. MELD score depend on?

a. Billirubin + Amylase + creatinine


b. Lipase + INR + billirubin
c. INR + billirubin + Amylase
d. Billirubin + INR + creatinine

1551. Patient with chronic liver disease mortality (19.6%) ,MELD score is?

a. 30-39
b. 20-29
c. 10-19
d. <9

1552. Which part is the most affected by crohon’s disease?

a. Terminal ileum
b. Sigmoid
c. Duodenum

1553. 58 years female she has pancreatitis, has been treated, after 2 weeks he came with epigastric
pain diagnosis was retroperitoneal cyst, management?

a. Internal drainage
b. External drainage
c. Cytogastrostomy
d. Conservative management

1554. Cancer at the lateral part of the tongue, treatment?

a. Hemiglossectomy with radical neck dissection


b. Wide local excision with Chemo radiotheraphy
c. Wide local excision only

1555. Post cancer sirmoid resection anastomosis was done, patient came with abdominal signs of
septecemia, diagnosis?

a. Inadequate anitibiotics
b. Inadequate fluid
c. Anastomic leak
d. Old age

1556. Most common post operation complication of rectal prolapsed?

a. Bleeding
b. Infection
c. Incontinence

1557. Split thickness graft viabilityin the 1st 48 hours by?

a. Plasma Imbibition

1558. Which lectrolyte is more in saliva?

A Cl
b. Ca
c. K
d. Na

1559. Piles first degree descend on straining and spontaneously reduced most appropriate treatment?

a. High fiber diet


b. Band ligation
c. Hemorrhidectomy

1560. IHBD with normal CBD best investigation?

a ERCP
b. MRCP
c. C.T
d. Ultrasound

1561. The origin of the inferior pancreaticoduodenal artery is the?

a. Gastroduodenal artery
b. Superior Msenteric artery
c. Superior pancreaticoduodenal artery
d. Common hepatic artery

1562. What is the second most common location for an accessory spleen?

a. Gastroplenic omentum (great omentum)


b. The gastrocolic ligament
c. The tail of the pancreas
d. The splenocolic ligament

1563. Which of the following is not of Reynold’s pentad?


a. Hypovolemic shock
b. Jaundice
c. Mental status changes
d. Fever

1564. Which of the following is the most sensitive imaging study to identify and localize a gastrinoma?

a. C.T
b. MRI
c. PET
d. Octriotide Scintigraphy

1565. Patient with ABI 0.6, what is the interpretation and recommendation?

a. Acceptable and treatment risk factor


b. Normal and none
c. Severe arterial disease and treatment risk factor to vascular specialist
d. Moderate arterial disease and refer risk factor to vascular specialist

1566. Case of piles first degree descend on straining and spontaneously reduced, what is the most
appropriate treatment?

a. High fiber diet


b. Band ligation
c. Hemorrhoidectomy

1567. What is true about pagets disease of the nipple?

a. Arise from lactefirious ducts


b. Once present 80% of breast tissue affected by cancer
c. The cancer cells present directly below the lesion

1568. T.B enteritis with caseation L.N with a photo small intestinal affection by ulcer at terminal ileum
and perforation treatment?
a. Resection and anastomosis
b. Right hemicolectomy
c. L.N dissection and terminal ileostomy

1569. More than 80% of accessory spleens are found in the splenic helium, what is the second most
common location for an accessory spleen?

a. Gastrosplenic omentum
b. The gastocolic ligament
c. The tail of the pancreas
d. The splenocolic ligament

1570. The origin of the inferior pancreaticoduodenal artery is the?


a. Gastroduodenal artery
b. Superior mesenteric artery
c. Superior pancreaticoduodenal artery
d. Common hepatic

1571. Which of the following is not a part of Reynaold’s pentad?

a. Hypovolemic shock
b. Jaundice
c. Mental status
d. Fever

1572. Which of the following is the most sensitive imaging study to identify and localize a gastrinoma?

a. C.T
b. MRI
c. PET
d. Octrotide scintigraphy

1573. The most common location of the superior parathyroid glands is?

a. Dorsal to the recurrent laryngeal nerve within 1cm of the junction of the RLN and
inferior thyroid artery
b. Ventral to the RLN within 1cm of the junction of the RLN and inferior thyroid artery
c. Dorsol to the RLN within 3cm of the junction of the RLN and inferior thyroid artery
d. Ventral to the RLN within 3cm of the junction of the RLN and inferior thyroid artery

1574. A premature infant with bloody stools, bilious emesis, and the following KUB, should be treated
with?

a. N.G decompression and antibiotics


b. Percutaneous abdominal drainage with peritoneal irrigation
c. Exploratory laparotomy, resection of the involved bowel and primary anastomosis
d. Exploratory laparotomy, resection of the involved bowel, and end ostomies

1575. Intesinal duplication are not commonly located in the?

a. Duodenal
b. Jejunum
c. Ileum
d. Colin

1576. Total body water in a full term infant is approximate?

a. 60ml/kg
b. 80ml/kg
c. 100ml/kg
d. 120 ml/kg
1577. The glomerular filtration rate of a new born is approximately?

a. 30% less than a normal adult


b. 60% less than a normal adult
c. 30% more than a normal adult
d. 60% more than a normal adult

1578. In children, hernis are most common?

a. In a boy left side


b. In a boy Right side
c. In a girls left side
d. In a girls right side

1579. Most common site fir Richter’s hernia?

a. Umbilical
b. Femoral
c. Inguinal
d. Epigastric

1580. After marjolin ulcer excision in popliteal fossa (picture) repair with?

a. Flap
b. Graft
c. Primary intention
d. Secondary intention

1581. Which of the following patients should undergo elective repair of an asympotomatic abdominal
aortic aneurysm?

a. Man with 4.5 cm aneurysm


b. Woman with 5cm aneurysm
c. Man with aneurysm grown 0.5cm in the last year
d. Woman with aneurysm grown 0.5 cm in the last year

1582. Which of the following is the gold standard for the diagnosis of renal artery hypertension?

a. Captopril renal scanning


b. Renal artery duplex ultrasound
c. Selective angiography
d. MRI and IV gadolinium

1583. Amaurosis fugax is symptoms of occlusion of the?

a. Posterior cerebral artery


b. Circle of willis
c. Internal carotid artery
d. External Carotid Artery

1584. 52 years old female, nipple discharge (thick green), which one is not true?

a. Majority of patients are smokers


b. Typically procedure blood stained nipple discharge
c. No high risk of breast cancer
d. Lead to mammary duct fistula
e. need total duct excision if it fails to resolve

1585. 22 years old with hyperthyroidism, diagnosis, grave disease. Which of the following describes
the patho physiology of the condition?

a. Formation of Ig G antibodies to the TSH receptor on the thyroid gland


b. Formation of Ig G antibodies to the TSH receptor on the anterior pituitary
c. Formation of TGA antibodies to the TSH receptor on the thyroid gland
e. Formation of IgM antibodies to the TRH receptor on the anterior pituary

1586. 20 years old female thyroid cancer, otherwise well, no h/O family, O/E she has nodule in left
lobe of the thyroid with small mass separate from the gland, which of the following is the most likely
cause?

a. Folicular carcinoma
b. Anaplastic carcinoma
c. Medulary carcinoma
d. Papillary carcinoma
e. B-cell lymphoma

1587. 43 years old female is carrier of BRCA, from breast cancer, which of the following is great risk of
developing?

a. Colon cancer
b. Ovarian cancer
c. Pituitary adenoma
d. Follicular carcinoma of thyroid

1588. 22 years old female with episode of renal colic and by investigation she is suffering from MEN ll,
which of the following abnormal of the parathyroid glands?

a. Hypertrophy
b. Hyperlasia
c. Metaplasia
d. Adinoma
e. Carcinoma

1589. 45 years old female fracture (shaft) poor history, she underwent a thyroidectomy one year
previously, what is the most likely underlying diagnosis?

a. Hyperporathroidism
b. Metastatic papillary carcinoma of the thyroid
c. Metastatic medullary carcinoma of the thyroid
d. Metastatic follicular carcinoma of the thyroid
e. None of the above

1590. Which of the following is not included in MEN ll?

a. Pheochromocytoma
b. Visceral ganglipneuromas
c. Thyroid medullary carcinoma
d. Zollinger Ellison syndrome
e. Marfanoid Elllison syndrome

1591. 25 years old male with carcinoid syndrome, which of the following hormones is released
carcinoids?

a. Serotonin
b. Dopamine
c. Nor adrenaline
d. adrenaline
e. Aldosterone

1592. 56 years old male, with episodes of facial pain and discomfort while eating, he suffered from
halitosis and dry mouth, he has a smooth swelling under right mandible, what is the most likely
underlying diagnosis?

A Adenoma of the submandibular gland


b. Adecarcinoma of the submandibular gland
c. Squamous cell carcinoma of the submandibular gland
d. Stone impacted in whartons duct

1593. Which of the following genes protects against neoplasmas?

a. Sis gene
b. P53 gene
c. Ras gene
d. Myc gene
e. SRC gene

1594. Which of the following is associated with the brown tumours of bone?

a. Hyperthyroidism
b. Hypothyrodism
c. Hyperparathyroidism
d. hypoparathyroidism
e. Osteopetorisis
1595. 56 years old male, diagnosis AAA, undergoes a C.T for size of aorta, during the investigation
there is lesion of adrenal gland 1.5cm but the gland in otherwise is normal, what is the most likely
diagnosis?

a. Adrenal gland metastasis


b. Adrenal gland artero-venous malformation
c. Adrenal cyst
d. Pheochromacytoma
e. Adrenalcortical adenoma

1596. 55 years old female complain discomfort in in right breast O/E, small lesion suggest fibro
adenoma, but there is a small area of lobular carcinoma in situ, what is the management?

a. Whole breast irradiation


b. Simple mastectomy
c. Mastectomy and sentinel lymph node biopsy
d. Wide local excision and sentinel lymph node biopsy
e. Breast MRI scan

1597. 30 years old male burn 30% full thickness, after 3 day of admission he has hematemesis, which
of the following event?

a. Dieulafoy lesion
b. Curling ulcers
c. Mallory Weiss tear
d. Deplation of platelets
e. Deplation of clotting factors

1598. 59 years old complain urinary sepsis and suffered from attacks of lest iliac fossa pain since few
months, also he has bubbles in his urine, C>T shows a long inflammatory mass in left iliac fossa most
diagnosis is?

a. Ulcerative colitis
b. Crohon’s disease
c. Mesentric ischemia
d. Diverticular disease
e. Rectal cancer

1599. 17 years old male, complain Meckels, diverticulum from which of the following embryological
structure is derived?

a. Foregut
b. Hindgut
c. Urachus
d. Cloaca
e. Vitello –intestinal duct

1600. 20 y/o female from Africa undergoes an open appendectomy, she is reviewed for unrelated
problem 8 months later. On examination: Abdominal the wound site is covered by shiny dark
protuberant scar tissue that projects beyond the limits of the skin incision, which of the following is the
most likely underlying process:

a) Repeated episodes of wound sepsis


b) Keloid scar
c) Marjolin’s scar
d) Hypertrophic scar
e) Mycosis fungoides

1601. Female 4 y/o right loin pain, Lethargy, hematuria On examination: pyrexia with large mass in
RUQ, the most likely underlying diagnosis:

a) Periphrenic abscess
b) Nephroblastoma
c) Renal cortical adenoma
d) Grawitz tumor
e) SCC of the kidney

1602. 64 y/o male, RUQ discomfort, he has never attended the hospital. He is retired, he was worker in
PVC (poly vinyl chlorate) factory ( mechanic, CT shows a large irregular tumor in the right lobe of this
liver, which of the following lesions is the most likely:

a) Liposarcoma
b) Angiosarcoma
c) Hamartoma
d) Hydatid liver disease
e) Benign angioma

1603. 28 y/o Male has carcinod tumor identified in his appendix, which one of the following helpful for
follow up:

a) CA-19-9
b) AFP
c) CEA
d) Chromogranin A
e) Alkaline Phosphate

1604. 34 y/o female history of colectomy for familial adenomatous polyposis, presents with firm lesion
inferior of her rectus abdominal muscle, which cell type is most typically associated with such tumors:

a) Myocytes
b) Proliferation of apocine glands
c) Chondrocytes
d) Lipoblasts
e) Myofibroblast

1605. 23 y/o male with inguinal hernia, on examination: he has pigmented spots around his mouth and
his palms and sales, history of: in 12 years of age he underwent a reduction of intussusceptions, which
of the following lesions is most likely to be identified if a colonoscopy were performed:

a) Hamartomas
b) Tubulovillous adenoma
c) Colorectal cancer
d) Crohns disease
e) Hyperplastic polyps

1606. Which of the following from micro organisms in clinical picture resembling achalasia of the
esophagus:

a) Epstein Barr
b) Candida SPP
c) Wucheneria bancrofti
d) Trypanosoma cruzi
e) Helicobacter pylori

1607. 34 y/o female, she undergoes elective cholecystectomy due to gallstone, microscopic assessment
of the gallbladder is most likely to show which of the following:

a) Widespread necrosis
b) Rokitansky aschoff sinuses
c) Metaplasia of the fundus
d) None of the above
e) Dysplasia of the fundus

1608. Male 45 y/0 with urinary colic, history of: he has left loin mass and a varicocele, the most likely
diagnosis is:

a) Renal adenocarcinoma
b) Renal cortical adenoma
c) SCC of the renal pelvic
d) Retroperineal fibrosis
e) Nephroblastoma

1609. Male 72 y/o, complaint of: BPH (benign prostatic hyperplasia which of the following structure is
most likely to be enlarged:

a) Posterior lobe of the prostate


b) Median lobe of the prostate
c) Right lateral lobe of the prostate
d) Anterior lobe of the prostate

1610. Female 46 y/o diarhea, weightloss of 10kg and a skin rash of erythematous blisters involving the
abdominal and buttocks, the blisters have an irregular border and both intact and ruptured vesicles,
what is the most likely diagnosis:

a) Colonic adenocarinoma
b) Pancreatic adenocarcinoma
c) Tropical sprue
d) Glucagonoma
e) Insulinoma

1611. One of the factors which prevents thrombosis in the normal venous system is:

a) Smooth muscle contraction in the wall of the vein.


b) Distention of the vein as volume increases in diastole
c) Endothelial production of prostaglandin
d) Endothelial production of endothelial relaxing factor

1612. Thrombolytic drugs work by

a) Converting plasminogen to plasmin


b) Increasing the activity of plasmin
c) Converting thrombin to fibrin
d) Decreasing the activity of thrombin

1613. Which of the following veins does not have valves:

a) Cranial sinues
b) Portal vein
c) Iliac vein
d) All of the above

1614. Warfarin inhibits carboxylation of which of the following

a) Factor III
b) Factor VII
c) Factor VIII
d) Factor XI

1615. Which of the following is not a risk factor for venous thromboembolism.

a) Factor XI elevation
b) Protein C elevation
c) Nephritic syndrome
d) Travel >6 hours

1616. Male 25 y/o, RTA , right tibia number managed by fasciotomies and application external fixator,
after 48 hr. his serum creatinine rises and urine muddy brown, what is the most likely underlying
diagnosis:

a) Acute interstitial nephritis


b) Acute tubular necrosis
c) Glomerulonephritis
d) IgA nephropathy
e) Thin basement membrane disease.

1617. Which of the following hepatobiliary disorder are most classically associated with ulcerative
colitis:

a) Gallstones
b) Primary sclerosing cholangitis
c) Bile duct stones
d) Liver hamartomas
e) Hepatocellular carcinoma

1618. Which portion of the bladder is intraperitoneal?


a) Dome only
b) Dome and body
c) Dome, body, neck
d) None of the above

1619. Which of the following does not supply arterial blood to the testes?

a) Testicular artery
b) Deferential artery
c) Scrotal artery
d) Cremasteric artery

1620. Hydroceles form between?

a) The external spermatic and cremasteric fascia


b) The cremasteric and internal spermatic fascia
c) The internal spermatic fascia and the parietal layer of the tunica vaginalis
d) The visceral and parietal layers of the tunica vaginala

1621. Patient with bladder stones are at increased risk for which of the following bladder corners?

a) Adenocarcinoma
b) Transitional cell carcinoma
c) Squamous cell carcinoma
d) Chlorcarcinoma

1622. Elevated serum level of (a- fetoproetein) AFP in a man with firm testicular mass make which of
the following diagnosis most likely:

a) Seminomatous germ cell tumor


b) Nonseminomatous germ cell tumor
c) Leyding cell tumor
d) Sertoli cell tumor

1623. As a busy surgical trainee on the colorectal unit are given the enenviable task of reviewing the
units histopathology results for colonic polyps, which of the polyp types described below has the
greatest risk of malignant transformation?

a) Hyperplastic polyp
b) Tubular adenoma
c) Villous adenoma
d) Hamartomas polyp
e) Serrated polyp villous

1624. A 45 y/o man has widespread metastatic adenocarcinoma of the colon which of the following
tumor markers is most likely to be elevated:

a) CA-19-9
b) CEA (carcinoembryoonic antigen)
c) AFP
d) CA 125
e) BHCG
1625. What is the most common cause of mesenteric infarction?

a) Mesenteric vein thrombosis


b) Acute embolism affecting the superior mesenteric artery
c) Acute on chronic thrombus of the superior artery
d) Sub intimal dissection of the superior mesenteric artery
e) Proximal migration of the abdominal aortic aneurysm

1626. A 2 day old baby is noted to have voiding difficulties and on closer inspection is noted to have
hypospadias, which of the following abnormalities is most commonly associated with the condition:

a) Cryptorchidism
b) Diaphragmatic hernia
c) Ventricular septal defect
d) Bronchogenic cyst
e) Atrial septal defect

1627. Male 45 y/0, history of : ulcerative colitis and rectal dysplasia and DALM lesion in the rectum,
what is the most appropriate management:

a) Snare polypectomy
b) Repeat endoscopy in 2 years
c) Discharge
d) Anterior resection
e) Panproctocolectomy

1628. Male 39 y/o, he has suffered from terminal ileal crohons disease for the past 20 years ago, which
the least likely to develop:

a) Gallstone
b) Malabsorption
c) Pyoderma gangrenosum
d) Amyloidosis
e) Felty syndrome

1629. Which of the following is not absorbed after gastrectomy?

a) Vit C
b) Vit B12
c) Zinc
d) Copper

1630. 34 y/o male with peptic ulcer, which of the following is responsible of release of gastric acid?

a) Chief cell
b) Parietal cell
c) G cell
d) Brunners’ gland

1631. Female 34 y/0 diagnosis hyperparathyroidism, PTH surgeon performs parathyroidectomy, how
many time need to decrease PTH level if we removed the adenoma successfully:

a) 6 hours
b) 24 hours
c) 2 hours
d) 1 hour
e) 10 minutes

1632. Male 25 y/o who has been morbidly obese for the past 5 years , he reviewed bariatric clinic, in
this patient release of which of the following hormones would increase apetite?

a) Leptin
b) Thyroxine
c) Adiponectin
d) Ohrelin
e) Serotonin

1633. Male 42 y/o homeless, he had an emergency inguinal hernia repair 24hr previously, he has BMI
15, he has been put on a feeding regime of 35 Kal/kg/day, with no additional medication, the nursing
staff contact you as he has become confused and unsteady. On examination: patient is disoriented to
place , has diplopia and nystagmus, what is the most likely diagnosis:

a) Cerebellar stroke
b) Acute dystonic reaction
c) Cerebrovascular accident
d) Parkinsonism
e) Wernickes enecephalopathy

1634. Health research usually aims it:

a) Visiting ideas which are familiar to all


b) Reinventing the wheel
c) Being novel applicable and easy
d) Following other people’s footsteps

1635. A health research protocol is:

a) A detailed oral plan of the proposed study considered the scientific component of a proposal for
funding
b) A detailed written plan of the proposed study, considered the scientific component of a
proposal for funding.

1636. A statement of expected relationship between two or more variables known as :

a) Concept definition
b) Hypothesis
c) Problem statement
d) Research question

1637. The purpose of the literature review is to

a) Use the literature to identify present


b) Assist in defining the problem and operational definition
c) Identify strengths and weakness of previous studies
d) All of the above
1638. A community health survey may be best defines as

a) A study which attempts to assertion the frequency of a disease in a fixed geographic regon or in
a group defined by a common membership.
b) A study which attempts to assertion the long term effects of disease in a multiple geographic
region or in a grouped defined by a common membership.

1639. Community health surveys usually have the following design approach

a) Experimental
b) Case control
c) Cohort
d) Cross sectional

1640. The following frequency measure could be calculated from surveys

a) Odds ratio
b) Prevalence only
c) Prevalence and possibly incidence
d) Risk ratio

1641. The following effect measure (s) could be calculated from surveys:

a) Studying rare diseases


b) Setting priorities and health planning
c) Studying more exposures
d) Reducing budgets and planning timelines.

1643. Surveys are quite popular-study-designs because

a) They are cost and time effect

1644. The following is true about telephone surveys

a) Need well trained interviewers


1645. Most common organism found in the colon

a) Aerobes E-coli
b) Anaerobes Bacteroid

1646. Most common type of choledocal cyst

a) Intrahepatic diffuse

1647. Most reliable sign in acute appendicitis

a) Localized RLQ tenderness

1648. Pathophysiology of acute appendicitis

a) Ulceration of the mucosa

1649. Most common cause of spontenous pneumothorax


a) Rupture of bleb

1650. Fibroadenoma is commonest in

a) > 50 years
b) 15-35

1651. Female swelling 3cm with peau de orange with supra clavicular LN no evidence of distant
metastasis

a) T 3 N0 M 0
b) T3N1M0
c) T4N3M0
d) T4N1M1

1652. Medial border of triangle of doom

a) Iliac vessel
b) Genetal vessel
c) Inf. Epigastric vessel
d) Ductus VAS

1653. Case of obstructed hernia (with picture) of dark patch of small intestine antimesenteric border,
what to do first:

a) Hot saline
b) Resection
c) Wedge resection
d) Nothing

1654. Female 46 y/0 diabetic on oral, hypoglycalemia epigastric pain 6 hr amylase elevated what
pathophysiology?

a) Gallstone
b) Alcohol
c) Drugs
d) Old age

1655. 32 y/0 male alcoholic epigastric pain radiated to back amylase elevated

a) Alcohol
b) Gallstone
c) Drugs
d) Old age

1656. Male 42 y/o, BMI 40, wants to do surgery with the lowest morbidity and mortality.

a) Ventral sleeve
b) Lap. Ray-en-y by pass
c) Gastric mini bypass
d) Gastric band

1657. When to give blood on shock


a) Hb .7
b) Hb. 8
c) Hb.9
d) Hb.10

1658. Duration of antibiotics treatment for complicated of diserticulitis

a) 2weeks
b) 4 weeks
c) 6 weeks
d) 12 weeks

1659. Case of fistula what is the best investigation

a) CT
b) MRI
c) X ray
d) Fistulogram or fluoroscopy

1660. Case of female 22 years breast mass 2x3 cm mobile, diagnosis:

a) DCIS
b) LCIS
c) Intraductal papilloma
d) Fibroadenoma

1661. Case of female 32 years mass 3x3 cm retro areolar, not mobile serious discharge, diagnosis?

a) Duct ectasia
b) Duct papilloma
c) Malignant
d) Apocrine metaplasia

1662. Case of herni containing part of intestine, diagnosis?

a) Litter’s hernia
b) Richter hernia
c) Amyand hernia
d) Sliding hernia

1663. Patient with vomiting and diarhea what is the electrolytes disturbance

a) Hypokalemia
b) Hyponatremia
c) Hypomagnesemia
d) Hypocalcemia

1664. Patient with tingling sensitive around lip, what electrolyte disturbance

a) Hypokalemia
b) Hyponatremia
c) Hypomagnesemia
d) Hypocalcemia
1665. Safety margin of melanoma

a) 0.5 cm
b) 1 cm
c) 1.5 cm
d) 2 cm

1666. Origin of BCC pluripatent cells

1667. Patient with massive hemothorax, BP 100/60 pulse 110, what to do first

a) Blood
b) Emergency thoracotomy
c) Chest tube

1668. Patient do subtotal thyroidectomy for MNG, now has prevent goiter TSH., T3 and T4 normal
what cause of prevent

a) Hypothyroidism
b) Malignancy
c) T4 support
d) Autonomous nodule

1669. Case of thyrotoxicosis, female 18 years diagnosis

a) Hashimoto
b) Riedal
c) Graves

1670.

1671.

1672. 53 y/0 male with anal verge ulceration biopsy SCC. Infection with which viruses

a) Human papiloma v.7


b) H. Papiloma v. 16
c) H. T-lymphotopic v.1
d) H. immunodeficiency v.1
e) H. immunodeficiency v.2

1673. 72 y/o male with preforal vascular disease develops gangrenous toe, on examination: evidence
infection surrounding tissues , crepitus most likely infective organism:

a) Staphylococcus aureus
b) Streptococcus pyogenrres
c) Clostridium difficile
d) Clostridium perfringens
e) Clostridium botulinum

1674. Gas gangrene is caused mainly by

a) Exotoxin produced by clost perfrin


b) Endotoxin produced by clost perfringes
c) Exotoxin produced by clost tetani
d) Endotoxin produced by clost tetani

1675. Best investigation to visualize cystic breast mass is

a) MRI
b) CT
c) Mammogram
d) U/S

1676. Best investigation of ductal carcinoma

a) MRI
b) CT
c) Mammogram
d) U/S

1677. Patient female 55 y/o with BIRADS (3) what is the chance percentage of malignancy

a) 2%
b) 10%
c) 50%
d) 95%

1678. Patient male 40 y/o with BIRAD (2), in which category is she?

a) Suspicious malignancy
b) Probably benign
c) Benign
d) Normal

1679. Gallbladder stone asymptomatic and patient has history of enteric typhoid fever and pain in
right UQ treatment

a) Cholecystectomy
b) Conservative

1680. Patient 60 y/o gallbladder with stone by U/S and CBD dilated without stone, best investigation.

a) ERCP
b) MRCP
c) CT

1681. Gallbladder stone with fever and pain in right UQ, tenderness, she started on antibiotics after 3
days no improved, best investigation

a) ERCP
b) MRCP
c) CT

1682. Patient 70 kg female stab wound to abdominal he is hypotensive with tachycardic and confused,
what percent of blood lost.
a) 5%
b) 15%
c) 35%
d) 55%

1683 Patient fell down from 4 level of building after 3 hours he came in ER , the nurse observed his
urine is dark why

a) Ureter injury
b) Bladder injury
c) Kidney injury
d) Muscle distraction

1684. Most common salivary gland tumor is in

a) Subclavian
b) Parotid
c) Sublingual
d) Submandibular

1685. Most common site for mixed tumor

a) Subclavian
b) Parotid
c) Sublingual
d) Submandibular

1686. Important cell for wound healing

a) Basophils
b) Macrophage
c) Neutrophils
d) Esonophils

1687. what nerve is commonly injures in humerus fracture

a) Radial
b) Ulnar
c) Median
d) Musculocutanious

1688. Cholecystitis may arise from

a) TPN
b) Over hydration
c) Low fatty diet

1689. which method for nutrition

a) Jejunostomy tube
b) Gastronomy tube
c) NGT
d) Nasoduedenal tube

1690. Chronic liver disease, bilirubin 2,5, INR : 1.7 mild acites, albumin 3, patient 5 non
encephalophaty, what is the child pugh score?

a. 7
b. 9
c. 11
d. 13

1691 Case of umbilical hernia scenario of strangulation with audible peristaltism, ask about content?

a. Bowel
b. Omentum

1692. Patient after total gastrectomy, drain removed after 2 days, start oral at 4 th day then abdominal
pain and suspect leak?

a. Duodenal stump blow out


b. jejuna leak

1693. History of right hemicolectomy 2 years ago with recurrent abdominal pain?

a. Colonoscopy
b. Barium

1694. Patient 10 days post operative a febrile with large amount of fluidy pus?

a. Betidine gauze
b. irrigation with saline
c. Closure
d. Culture

1695. Case of hypovelemic shock, ask about?

a. low cardiac output


b. Low arterial resistance

1696. New born with cystic hygroma, what to do?

a. intubation
b. Aspiration
c. 100% O2 mask

1697. To decrease errors in future?

a. See one
b. Do one
c. Teach one

1698. Initial investigation i patient unstable with RTA?

a. C.T
b. Ultrasound
c. DPL

1699. Femoral hernia (trnsguinal)?

a. Loc kwood
b. Lothessen
c. Mc Evedy
d. Mc vay

1700. Femoral hernia (incarcenation)?

a. Lock wood
b. Lothessen
c. Mcevedy
d. Mc vay

1701. Fermoral hernia (high approach)

a. Lock wood
b. Lothessen
c. Mcevedy
d. Mc vay

1702. Initial investigation in patient unstable with RTA?

a. C.T
b. DPL
c. Ultrasound

1703. Paraumbical Hernia detect diameter 2cm?

a. Anatomical repair
b. Laparascopy with mesh
c. Ghrelin

1704. Hormone cause indirect hyperglycemia and increase insulin resistance?

a. Thyroxin
b. Glucagon
c. Ghrelin

1705. A case of insulinoma and C.T free, how to confirm?


a. Endoluminal Ultrasound
b. MRI
c. PET scan

1706. Flial chest with paradoxical movements?

a. The diaphragm low inspiration


b. The diaphragm high expiration
c. The diaphragm low expiration
d. The diaphragm high expic and inspiration

1707. Gastric foveolar metaplasia with absent parital cells, initial treatment – medical if failed medical
treatment? What to do?

a. Total gastrectomy
b. Endoscopic management
c. Vagatomy

1708. Gastric ulcer (malignant), 3 L.N on right gastric artery + mass 4x4 near the fundus?

a. Total gastrectomy
b. Palliative

1709. FAP with 1000 polyps, 25 years male with family history, ask about treatment?

a. Total colectomy
b. Endoscopic

1710. How to differentiate between local recurrence and scar tissue after breast conservative
surgery?

a. MRI
b. Mammography
c. Ultrasound

1711. Breast cyst not completely resolved after aspiration (clear fluid)

a. Core biopsy
b. Observation

1712. Indicator of second fluid challenge in patient BP=70/40

a. CVP < 15
b. Pulmonare wedge pressure 20

1713. A case of insulinoma and CT free, how to confirm?

a. Endoluminal
b. MRI
c. PET scan

1714. A case of primary hyper parathyroitism PTH high, Ca high?

a. Primary hyperparathyroidism adenoma


b. Secondary hyperparathyroidism carcinoma

1715. A sample of 7 patients with weights as follows, 84, 85, 78, 82, 87, 88, 90. What is the range?

a. 78
b. 84
c. 85
d. 90

1716. A sample of 7 patients with weights as follows 84, 82, 78, 87, 88, 90. What is the range?

a. 12
b. 8
c. 85
d. 90

1717. A study was done on foetuses who were exposed to DES with control group and they were
followed for 20 years looking for development of cancer, what is the type of study?

a. Cohort
b. Case-control study
c. Cross – sectional study

1718. 40 years old breast mammogram show 1cm mass on LUQ satalit appearance with minimal
calcification and skin affection, next step?

a. MRI
b. Corbiopsy
c. Surgical Excision
d. Ultrasound

1719. Patient with thyroid hot, nodul, palpitation, carotid pulsation, T3 high, TSH normal
management?

a. Lobectomy
b. Radioactive
c. Total thyroidectomy
d. Anti thyroid

1720. 60 years old represent in ER with fever, lower abdominal pain new moderate ascites, and
management?

a. Antibiotic
b. C.T abdominal
c. Exploration
d. Paracentisis

1721. 11 years old RTA, splenic tear 3cm BP= 90/60 management?

a. Conservative
b. Exploration splenectomy
c. Exploration splenorraphy

1722. Condition has high mortality rate in old age?

a. Intussusceptions
b. Radiation Proctitis
c. Aortoentric Fistula
d. Acute Appendicitis

1723. Hernia below and medial medial to pubic tubercle when ring test is negative be? And when
positive be?

a. Ring Negative = direct


b. Ring positive = Indirect

1724. Lap post RTA gives deep liver laceration best treatment?

a. Deep sutures
b. Resection of liver lobe
c. Paking

1725. Which of the following is normally not palpable?

a. Hyoid bore
b. Parotid gland
c. Sublingual gland
d. Thyroid gland

1726. Hernia inferior and medial to pubic tuberculosis and ring test still negative

a. Complete indirect
b Incomplete indirect
c. Direct hernia

1727. Groin hernia in relation pupic tubercle above and medial is?
a. inguinal hernia
below and lateral is?
a. Femoral hernia

1728. Most appropriate investigation help for management the case?

a. C.T
b. Ultrasound
c. MRI
d. X-ray

1729. Ulcer reach to involve muscle 3 or 4 stage?

1730. Lady with 3cm mass solid non tender, mobile persist during menses, slightly increased in size

a. Fibrocyst
b. Papiloma
c. Ductal Carcinoma
d. Fibro adenoma

1731. Acute pancreatitis, nutrition by?

a. TPN
b. Jeunal

1732. Male patient with epigastric pain that charged to generalized abdominal pain + tenderness, x-
ray abdominal air under diaphragm, Diagnosis is?

a. Gastric perforation
b. Append Rupture
c. Ileal Perforation

1733. Most common cause of extra hepatic biliary dilatation is?

a. Pancreatic Ca
b. Choledocholitaiasis

1734. The most important option in treatment of empyema is?

a. Antibiotic
b. Tube drainage
c. Closure of potential space
d. Repaeted wash with antibiotic

1735. Patient complain pain at right shoulder + right side of neck at morning and inability to abduct
right shoulder above head, diagnosis?

a. Compression at root C4, C5


b. Should do CT myelography
c. Should not do x-ray in cervical spine

1736. Patient diagnosis is non Hodgkin lymphoma by cytology, what is the next step?

a. Excisional biopsy
b. Start Chemotherapy
c. Start Radiotherapy

1737. Which of the following antibodies of patient with Grave disease?

a. Antithyroglobulin (Anti TGAb)


b. Antithyroid peroxidase (Anti TPOAb)
c. Anti – DNA antibodies (antinuclear)
d. TSH DNA antibodies (Anti TSab)
e. Anticardiolipin antibodies

1738. Patient with thyroid nodule 42 years old, T3 high, next step?

a. Radio active Iodine


b. Ultrasound
c. FNA

1739. Condition has high mortality rate in old age?

a. Intusseption
b. Radiation Proctitis
c. Aortoentric fistula
d. Acute appendicitis

1740. A 40 years old women complain hyperthyroidism O/E she has palpable nodule, but no evidence
of exophthalmos she does have pretibial myxedema Lab: TSH suppressed, free T3 high, what is the
next step in management?

a. Radioactive uptake scan


b. Neck Ultrasound
c. PTU
d. FNA
e. Cervical Ultrasound

1741. Which of the following is not an acceptable indication for surgical treatment of hyperthyroidism
in the patient in question 1740?

a. Nodule confermed or suspicious for malignancy


b. Multinodular Goiter
c. Non compliance

1742. With regard to the pharmacologic treatment for hyperthyroidism, which of the following is not
true?

a. PTU works by inhibiting organic binding of iodine coupling of iodotyrosine


b. PTU is associated with agranulocytosis
c. PTU is the preferred treatment in pregnant patient
d. MEthimazole can worsen exophthalmos in patient grave disease
e. Methimazole has longer half life and and require once dosing
1743. Hematocrit threshold for blood transfusion?

a. 30%
b. 25%
c. 21%
d. 18%

1744. Female 78 years old midline swelling not move with deglution but move upward with
swallowing patient refuse any surgery, what is the fate of this swelling?

a. Spontaneous disappear
b. Malignancy trasnformated
c. Laryngeal compression
d. Nothing

1745. Acute pancreatitis, CT revealed pancreatic tail cystic 3cm dilatation in pancreatic duct with
filling defect and papillary projection inside the cystic dilatation, what is the treatment?

a. Total pancreatectomy
b. Pancreaticocduodenectomy
c. Lateral pancreaticojejunostomy
d. ERCP with stenting

1746. Patient with abdominal distension, vomiting, tenesmus, RUQ, RLQ pain, best initial
investigation?

a. X-ray
b. C.T
c. DPL

1747. Solitary solid nodule move with swallowing diagnosis?

a. Ultrasound
b. TSH
c. C.T

1748. The most common cause of hyperkalcemia in hospitalized patient is?

a. Primary hyperparathyroidism
b. Metastatic carcinoma
c. Sarcoidosis
d. Immobility
e. Milk alkali syndrome

1749. 20 years athlete RTA, in ER he shouts that he cannot move his legs, O/e. There are no
abnormality of the chest abdomen, pelvis, the patient has no sensation in his legs, cannot move them
but his arms are moving, the patient respiratory rate is 23, heart rate 88, BP= 80/60, pale, Sweaty,
what the most likely cause of his condition?
a. Neurogenic shock
b. Cardiac contusion
c. Myocardial contusion
d. Hypotermia
e. Abdominal haemorrhage

1750. Most major bile duct injuries during laparoscopic cholecystectomy occur in patients under
which of the following circumstances?

a. Acute Cholecystitis
b. Gallstone Pancreatitis
c. Choledocholitiasis
d. Elective Cholecystectomy
e. Conversion of laparoscopic procedure to open procedure

1751. Laparoscopic cholecystectomy is most strongly contraindicated in which of the following


situation?

a. Pregnancy
b. Previous abdominal surgery
c. Known common bile duct stones
d. Chronic obstructive pulmonary disease
e. Gallbladder cancer

1752. On the second operative day following elective laparoscopic cholecystectomy, a 40 years old
female complain nausea with abdominal pain, on examination shows T0 37.8 degree. P=100, b/m, mild
abdominal distension, and moderate upper quadrant tenderness, (RUQ). Which of the following would
next be appropriate?

a. Administrative I.V antibiotics


b. MRCP
c. HIDA scan
d. ERCP
e. PTC

1753. Which of the following is most accurate in the diagnosis of acute cholecystitis?

a. Pain abdominal radiographs


b. Ultrasound
c. Oral cholecystegraphy
d. Thenetium – 99m per technetate and HIDA
e. Leukocytosis with elevated transminases

1754. Papillary thyroid cancer 3cm left lobe no palpable lymph node, what to do?

a. Left lobectomy with modified neck dissection


b. Total Thyrodectomy with radical
c. Left lobectomy with radical limph 0node dissection
1755. Big incisional hernia, preoperative what should to do?

a. C.T Abdominal to detected the size

1756. Onguinal hernia, sac at the apex of scrotum

a. Complete indirect
b. Incomplete indirect

1757. Case about RHQ, pain fever palpable mass at RHQ area?

a. Mucocele
b. Acute cholecystitis

1758. Case about transverse colon cancer the surgeon should legate which of the vessel

a. Midd colic artery + right colicartery

1759. Case of peri pancreatic fluid 5x6 cm without symptoms

a. Wait and watch


b. Drainage

1760. Case of mid femur fracture, management?

a. Retrograde medullary nail


b. Antegrade Medullary nail
c. Retraction

1761. Spherocytosis

a. Ultrasound for gallbladder

1762. Which system resistence for antibiotic

a. Blood
b. Intra abdominal
c. Ganitourinary
d. Injury

1763. SIRIS, peripheral temperature > 38.9

a. WBC < 4000

1764. Case of arterial thromb was removed then patient developed tense calf weak pulse

a. Compartment syndrome

1765. Case of arterial thromb was removed then patient developed tense calf weak pulse
a. Compartment syndrome

1766. Neonat pyloric stenosis

1767. Which of the following lead to respiratory distress in neonat

a. Torticollis
b. Cystic huaroma

1768. Cost of old male 70 year with mid oesophagus cancer with a metastasis management?

a. Stent
b. Resection

1769. Case of child with infection in the foot, red streak, and organism?

a. Strep – pyogenes

1770. Post splenectomy immunization?

a. immunoglobulin

1771. Case of cut wound receive tetanus vaccine, 6 years back, what to give?

a. Immunoglobin

1772. Iliostomy 3 month ago?

a. Re fashion
b. Urgent refashion

1773. Recurrent inguinal hernia with history of prostatectomy old male?

a. TEP
b. TAPP

1774. Dumbing

1775. Insulinoma

1776. Indcation of liver trasnolant

a. Chronic venous occlusive disease

1777. Head trauma with concave in C>T

a. Subdural haemorrhage
1778. Best fluid for gastric outlet obstruction

a. Normal saline
b. RL

1779. Case about invasive oesophageal cancer

a. Bronchoscopy

1780. Case about 4cm appendicular mass histopathology after operation showed characinoid tumour,
what to do?

a. C.T abdomen
b. Right hemicolectomy

1781. Condition has high mortality rate elderly

a. Intussusceptions
b. Acute appendicitis
c. Aorto enteric fistula
d. Radiation Proctitis

1782. Female 45 years old with breast mammogram show 1cm ULQ, satellite appearance minimal
calcification, skin affection, what is the next step?

a. MRI
b. Ultrasound
c. Core biopsy
d. Excision

1783. Male w50 years old male constipation, bleeding PR, cachexic, has circumference anal verg
adenocarcinoma, what is the management?

a. Dysfunction colostomy
b. Abdominoperineal resection with permanent colostomy
c. Chemo radiotherapy

1784. Esophagitis healed with scar

a. Esophag Barnett
b. Short esophagus
c. Esophageal ring

1785. The vast historological majority of anal cancer?

a. SCC
b. BSC
c. Adenocarcinoma
1786. Female 56 years old alcoholic chronic pancreatitis main pancreatic duct cyst 3 cm, small cystic
on small branced pancreatic duct management?

a. Pancreatectomy
b. ERCP stent
c. Side to side pancreaticojejunostictomy

1787. 25 years thyroid nodule 1.5cm with FNA show follicular cell, management?

a. Lobectomy
b. Isthmictomy
c. Total thyroidectomy
d. Radioactive iodine

1788. The most common condition during breast lactation (feeding)?

a. Breast abscess
b. Bacterial mastitis

1789. 2cm cystic breast lump 35 years male, diagnosis by ultrasound , local exam tender cystic
swelling?

a. Aspiration cytology
b. Reassurance follow up
c. Excision

1790. Patient show intra hepatic biliary dilatation by ultrasound with normal CBD, next step?

A ERCP
b. MRCP
c. PTC
d. C.T abdomen

1791. Case of MI, 2 weeks ago, now present with sudden severe pain calf muscles, pale foot, femoral
palpated, papliteal not palpated diagnosis?

a. Compartment syndrome
b. DVT
c. obstructed emboli
d. Throbus

1792. ilestomy stoma dusky, non functioning

a. Conservative
b. Hotpade
c. Refashioning

1793. The best elective repair of recurrent ainguina hernia in 50 years, male patient without previous
surgery on lower abdomen?
a. TAPP
b. TEP
c. open repair

1794. Blunt abdomen trauma, vital stable but has left hypochondrial pain radial to left shoulder?

a. Admit in ER observation
b. Hospital admission, investigation

1795. The best investigation for cerebral lesion?

a. C.T
b. MRI

1796. Case of hypovelemic shock, Oligurea hypokalemia?

a. Give I.V fluid + KCL at rate of 20mmlh


b. Give bolus IV fluid until up to increase, then give kcl iatrogenic

1797. During lap cholectostomy team in CBD 4mmverbal treatment?

a. Clipping on tear
b. ERCP + stent
c. T-tube

1798. T6 injury present with urine retention cause?

a. Detrusor muscle injury


b. Neurogenic shock

1799. Patient has right hemicolectomy 2 years ago, now come with abdomen pain, vomiting, what is
the best investigation?

a. C.t Abdomen
b. Colonoscopy
c. Barium enema

1800. Superficial pyloric adenocarcinoma is treatment, which types of radical?

a. Rl
b. R ll
c. R lll
d. R lV

1801. Patient with radical hemiglossectomy and mandebulectomy, other sid of neck dissection of L.N
thrombosis of internal jugular vein treatment?

a. Exploration thrombectomy
b. Warfarin therapy
1802. Breast cancer T2N3M1 in 70 years female

a. Local excision + chemotherapy


b. Radical mastectomy
c. Radiotherapy alon
d. Chemotherapy

1803. Best investigation to be done before splenectomy in case of heredity Spherocytosis?

a. ultrasound abdomen
b. osmotic fragility test

1804. Old patient alcoholic with melena 2 days ago, the cause is?

a. Bleeding GU
b. Bleeding DU
c. Oesophagus

1805. After splenectomy for trauma the optional timing for administration of the pnemococal vaccine
is on post operative day?

a. 1
b. 4
c. 7
d. 10
e. 14

1806. Most common cause abdominal pain in married female?

a. Acute appendicitis
b. Ovarian cyst rupture

1807. Common cause of liver pyogenic abscess

a. Ascending cholangitis
b. Pylephlepitis
c. C.B.D infection

1808. Imeedaite treatment hypercalcemia in vomiting dehydrated patient?

a. Biophosphonat
b. I.V fluids

1809. Chronic hypovolemia face old age bedridden, most common specific sign?

a. Bony prominence
b. isible veins
c. M.S wasting
d. Sunken eye

1810. most common strong anticoagulant secreted by medicainal leeches?

a. Hiparin
b. Warfarin
c. Hidrudin

1811. Safety margin needed for excision of malignant melanoma it thickness more than 4mm?

a. 10mm
b. 10-20mm
c. 20-30mm
d. 40mm

1812. Burn degree of electrical high voltage on armboy 15 years old ?

a. 1st
b. 2nd
c. 3rd
d. 4th

1813. RTA 35 years female on # trauma admitted to ICU and mechanical ventilation stable, after 15
days in ICU she developed on CT brain fresh sub arahnoid hemorrhage, what is most electrolytes
disturbance cause this?

a. Hyperkalemia
b. Hypercalcemia
c. Hypernatremia
d. Hyperglycemia

1814. On explosion 30 years male has nek injury as one splinter injury the internal carotid artery and
esophagus, what will you do?

a. repair of the artery and oesophagus


b. Ligation of the artery and esophagectomy
c. Repair artery and esophagectomy
d. Ligation of the artery and repair the oesophagus

1815. During normal appendectomy, surgeon find intra operative cecal diverticulitis, cecum
extensively affected and the frank pretonitis not find, what will you do?

a. Appendectomy
b. Right hemicolectomy
c. Ileoascinding colectomy with primary anastomosis
d. Fluid and antibiotic

1816. 30 year old with longitudinal tender tear on the 9óclock lithomy position with bright bleeding
per rectum, what finding merit to do biopsy?
a. Site
b. Size
c. Bleeding
d. Reccurency

1817. Epinephrine and norepinephrine secret from?

a. Adrenal
b. Thyroid
c. Pituitary

1818. Patient in ICU result for his laborator as PH=7.42 , Na= 133, K=3.3, BUN=17, what investigation
should do to help in determine serum osmolality for patient?

a. Ca+
b. Glucose
c. HCO3
d. Cl

1819. Patient complaining of bone pain depression, loss of memory, fatigality most likely cause?

a. High serum ca+ and intact PTH


b. Decrease ca+ urine level
c. Decrease serum phosphate level

1820. Patient gastric bypass operation since 2 years doing well. Loss of weight BMI 45 – BMI 35, but
has one episode after birthday dinner has confusion, uncomfortable relieved by drink?

a. Refractory hypervolemic
b. Refractory hyperglycaemic
c. Bile reflux
d. Food blocking

1821. 65 years old male complaining of abdominal cramping pain and tender 15-30 minutes from ear,
he loss weight and become afraid of eating, plain x-ray and ultrasound abdominal, free, diagnosis?

a. Colitis
b. D.U
c. Ch mesenteric ischemia

1822. 60 years old male heavy smoking abdomen pain diffuses, mild pain central, loght tenderness,
patient cyanosed, paller, cause?

a. Intestinal infraction
b. Intestinal obstruction

1823. 55 male patient with loss of weight, constipation and diarrhea, barium enema (apple core) most
appropriate next investigation?
a. CEA
b. C.T Abdominal pelvis
c. Colonoscopy
d. Ultrasound

1824. Right inguinal swelling reducible, cough pulsatile, swelling beneath pubic tubercle apex of
scrotum ring occlusive test negative, most likely diagnosis?

a. Complete indirect
b. Incomplete indirect
c. Direct inguinal
d. Femoral h.

1825. Most initial route for heparin administration


a. S.C
b. I.V
c. I.M
d. Oral

1826. RTA chest trauma patient left chest aire antry air sound, breathing diminished tracheal shift to
right, most initial management?

a. Needle thoracocentesis
b. Chest tube
c. Intubation
d. Ulcerative

1827. Most common cause of urinary tract infection with e-coli – colovesical fistula

a. Diverticulitis
b. Crohn
c. Ulcerative colitis

1828. Patent with history of diverticulitis come with high grade fever, lower abdominal pain,
ultrasound show 6x6 cm homogenous collection on pelvis, what will you do?

a. Percutaneous draining
b. Antibiotic
c. Follow up

1829. Patient take antibiotic for treatment of cellulites for more than 13 weeks develop oleic
abdominal pain, diarrhea, fever, most important to test to detect the cause?

a. Stool cytotoxin
b. ELISA
c. Blood culture
d. Folllow up
1830. Patient with diffuse painless thyroid gland enlargement with T3, T4, normal TSH high thyroid
antibodies positive?

a. Thyrodectomy
b. Radio iodine
c. Radiotherapy
d. Anti- thyroid drugs

1831. Patient complaining mass on the anal canal biopsy show Human papilloma virus

a. Paget’s disease
b. Bowen’s (skin care)
c. Adecarcinoma
d. Ohn

1832. Type of shock and vasopressin

a. Hypovolemic
b. Cardiogenic
c. Distributive

1833. What type of shock has the following character (Cardiac Output) COP low, (System vascular
Resistance) SVR high, BP low, o2 low

a. Cardiogenic
b. Hypovelemic
c. Distributive

1834. In ICU patient develop hypotension, hyponatremia, hypoglycaemia, hyperkalemia. What are the
most drugs given to save the patient?

a. Hydrocortisone
b. Fludrocortisone
c. Calcium
d. Sodium chloride
1835. Patient develop pyonatremic, hypokalemia, hypertension, CT show mass on kidney mostly high
diagnosis?
a. Adenocotocal carcinoma
b. Renal tumour
c. Pheochromocytoma

1836. Patient complaining of right side chest pain chest x-ray show right side lung opacity cyst,
breathing diminished on the right side diagnosis as hydrated cyst, most appropriate treatment?

a. Albendazol
b. Pneumonectomy
c. Excision Vd
d. PAIR
1837. Pregnant women 2nd trimester 13 weeks has tremors, palpitation ant neck swelling most need
treatment?

a. Thyrodectomy
b. Anti thyroid drug
c. Radio iodine
d. Radiation

1838. Papillary thyroid cancer 3cm left lobe no palpable lymph node, what do you do?

a. Left lobectomy with modified neck dissection


b. Total Thyrodectomy
c. Left lobectomy with radical lymph node dissection

1839. Big incisional hernia

a. CT scan abdomen to detect the size;

1840. Inguinal hernia sac at the apex of scrotum

a. Imcomplete indirect
b. Complete indirect

1841. Case about RUQ area

a. Mucocele
b. Acute Cholcystitis

1842. Case about transverse colon cancer the surgeon should ligate which vessels

a. Middle colic artery

1843. Case of peripancreatic fluid 5x6cm without symptoms

a. Wait and watch


b. Drainage

1844. Case of mid femur fracture management


a. Retrograd medullary nail
b. Anti grad medullary nail
c. Retraction

1845. Spherocytosis

a. Ultrasound

1846. Which system resistence for anti biotics


a. Blood
b. Intra abdominal
c. Gastrointestinal
d. Genitourinary

1847. SIRIS

a. Peripheral temperature
b. WBC <4000

1848. Case of hemosiderin deposits long time varicose

1849. Case of arterial thrombosis, was removed then patient developed tense calf, week pulse

a. Compartment syndrome

1850. Neonat pylonic stenosis

1851. Which of the following lead to respiratory distress in neonate

a. Torticollis
b. Cystic hygroma

1852. Case of old male70 years with mid esophagus cancer with with metastasis, what is
management?

a. Stent
b. Resection

1853. Case of child with infection intre foot, rd streak, organism?

a. Strep-pyogens

1854. Case of cut wound immunization?

a. Every 5 years

1855. Case of cut wound receive tetanus vaccine 6 years back, what to give?

a. immuglobulin

1856. Iliostomy?

a. Re-fashion
b. Urgent Re-fashion

1857. Recurrent inguinal hernia with history of prostectomy old male?

a. TEP
b. TAPP

1858. Dumbing syndrome

1589. Insulinoma

a. Alpha cell
b. Beta cells

1860. Indication of liver transplant

a. Hepatitis (c)

1861. Head trauma with concave in CT-scan

a. Subdural haemorrhage

1862. Best fluid for gastric outlet obstruction?

a. Normal Saline
b. R.L

1863. Case about invasive esophageal cancer

a. Bronchosope

1864. Case about 4cm appendicular mass histopatology after operation showed carcinoma tumour,
what to do?

a. CT abdomen
b. Right hemicolectomy

1865. Patient with and pain with no soaking cloth with endonoscopy free, pain of with DRE

a. Intersphenetric abscess
b. Pile
c. Fissure

1866. Female 31 years with one baby, 1 year lactining, on oral contraceptives, her mother died with
breast cancer, most indication for bad prognosis?

a. From OCP, age, lactation, ca

1867. What is coupling phenomenon?

a. Difference between conductive and capitance

1868. Most resistanct to infection?


a. GIT, urinary, pulmonary, blood

1869. Cancer with dyspepsia, weight loss associated with deposition and cystic enlargement and
ascites?

a. GB cancer
b. Stomach cancer
c. Pancreas cancer
d. Colon cancer

1870. Head trauma, CT showed hyperdense concave lesion in parital region?

a. Acute subdural age

1871. Child received with injured by wire 10 years ago

a. Tetanus toxoid only


b. Steroids

1872. Child received with injuired tetanus toxoid 10 years ago, now injured by wire, what is the
treatment?

a. Not give tetanus again


b. Give tetanus toxoid
c. Tetanus toxoid + antibiotics

1873. Most common benign tumour of breast?

a. Fibroadinoma
b. Fibrocyst
c. Papiloma

1874. Which compromise respiration during delivery to baby?

a. Cystic hygroma

1875. Best investigation in (CHTPS)?

a. Ultrasound
b. Endoscopy
c. G

1876. Patient with previous breast cancer 5 years back, ultra sound showed mass at upper of the
kidney, next step?

a. CT guided biopsy

1877. Most important determinant of surgery for hernia?


a. Size
b. Content
c. Complication

1878. Patient on TPN on blood transfusion comatosed, cause?

a. Electrolyte imbalance

1879. Cause of cold extremities with adequate O2?

a. Leriches sundrome

1880. Patient on TPN, doctor ask the nurses to give the patient 2u RBC, the nurses by fault stop TPN
and give the 2u on same line of TPN, after 2 hours patient become comatose, what is the caused?

a. Electrolyte imbalance

1881. Gunshot chest, stabbed patient with cold clammy skin from?

a. Hypoxia
b. Anorexia

1882. Optimal management of trauma duodenal hematoma?

a. Angiography mobilization
b. Surgical evacuations
c. Observation follows up

1883. Psychiatric patient with increased calcium signs and symptoms

a. IV fluid
b. Stop psychiatric drugs
c. Parathyroidectomy

1884. In wound healing which cell is essential?

a. Neutropils
b. Macrophage

1885. Patient child 4 weeks old presented with clear vomiting, what is the best investigation?

a. Ultrasound

1886. Patient old age voiting 3 times per day K=1.8, what is ECG changes will suspected

a. Prominent U-wave with flat T-wave

1887. Patient do cystectomy with uretro –sigmoid diversion ECG show prominent U-wave, what is the
problem in patient?
a. Hypokalamia

1888. Male patient diarrhea 1m, abdominal colicky anal fistula opening above dental line, what is the
best investigation?

a. CT
b. MRI
c. Colonoscopy
d. BR. Enema

1889. Female patient presented to ER, right upper abdominal pain, jaundice, laboratory show?

a. high direct bilirubin


b. High indirect bilirubin
c. High Alk. Phosphate
d. Temperature: 38.6

1890. What is the next step?

a. MRCP
b. ERCP
c. C.T abdominal

1891. What is the associated to coin indicated malignancy?

a. Calcification
b. Stallate like figure

1892. Female patient 65 years old with constipation, distension and abdominal cramps, show give
history that she is constipated 2 months ago, but she believed that the cause was changing diet, she
also do hysterectomy 10 years ago- on examination ++ intestinal sound with tenderness right iliac
fossa, what is the diagnosis?

a. Acute appendicitis
b. Adhesion I.O
c. Sigmoid vilvulus

1893. Female 78 years old medline swelling not moves with deglutition but move upward with
swallowing patient refuse any surgery, what is the fate of this swelling?

a. Spontaneous disappear
b. Malignancy transformation
c. Laryngeal compression
d. Nothing

1894. Origin of BCC is?

a. Pluripotent cells
1895. Patient of A.F after embolectomy it femoral artery start pain, numbness, swelling of the foot,
examination shows tenderness of calf ms., popliteal artery is felt, DX?

a. DVT
b. Compartment Syndrome
c. Re-embolism

1896. Patient sudden epigastric pain become generalized abdominal pain and tenderness, x-ray
abdominal show air under diaphragm, Dx?

a. Gastric perforation
b. Appendicular rupture
c. Ileal perforation

1897. Scenario patient acute right iliac fossa pain 2 days associated with vomiting, pain started
around umbilicus, PR tender, Dx?

a. Acute appendicitis

1898. Female patient no history of jaundice, during lap chole you find wide cystic duct and wide CBD,
what will you do?

a. Cholecystectomy then ultrasound


b. Pre-operative cholangiography
c. Open cholecystectomy and palpate CBD
d. Macroglossia
e. Intra operative cholangiography

1899. Pre term baby (32w weeks) with gastroschisis is more like to have?

a. Exstrophy of cloaca
b. Intestinal atresia
c. Microcephaly
d. Macroglossia

1900. A motorcyclist sustained a blunt abdominal trauma, 6 hours laters he admitted to ER with
abdominal pain and distension, his BP is 100/40 and won’t respond to resuscitation fluids given in ER,
what is the most suitable initial investigation to do in this case?

a. DPL
b. USG
c. Laparoscopic exploration
d. Plain x-ray

1901. Most frequent cause of liver transplantation in adult is?

a. Active HCV infection


b. Biliary cirrhoisis
c. hepalocelular carcinoma
d. Metastotic liver disease

1902. Most common cause of hemobilia

a. Liver trauma
b. Gallbladder stone
c. Pyocele
d. Muccocele

1903. Patient had right hemicolectomy for FAP 10 years ago, now presenting with a 14 cm parietal
abdominal mass at the operation site, what is the proper management?

a. Triple chemotheraphy
b. Conservative management
c. Wide local excision
d. Combined Chemo radiotherapy

1904. Small cell lung cancer associated with?

a. Horner
b. Cushing
c. Pulmonary osteodytrophy
d. Myastenia gravis like syndrome

1905. Undescended impalpable testises, best investigation

a. C.T
b. ultrasound
c. MRI
d. Abdominal exploration laparoscopy

1906. Exploration for parathyroid adenoma but not evidence of parathyroid tissue, what is the next
step?

a. Thyroidectomy total or hemi


b. Stemomastoid divion
c. Sestambi scan

1907. What thickness of melanoma at which sentinel LN biopsy is indicated?

a. < 0.76mm
b. > 1mm
c. >3mm
d. >5mm

1908. Bacteria which previously thought to be normal skin commensal with no pathogenic activity,
now believed to cause major systematic infections with high morbidity?
a. Pseudomonas
b. Fusobactreia
c. Bacteroids
d. E-coli

1909. During on incisional hernia repair in the upper abdomen, an iatrogenic transverse colon injury
happened with soiling of the wound area with colon contents, the injury was primary repaired, wound
is cleaned and good peritoneal toilet was performed, what is the best decision to consider about the
hernia repair?

a. Proceed with mesh repair


b. Primary repair of abdominal with no mesh
c. Delay the repair for 4 days then proceed with mesh repair
d. Close the abdmominal wall under tension with primary repair.

1910.Male patient with diarrhea 1 month abdominal colicky pain anal fistula opening above dental
line, what is the best investigation?

a. CT.
b. MRI
c. Colonoscopy
d. Br enema

1911. Best investigation for cerebral lesion?

a. CT
b. MRI

1912. Abdominal cramps 15 minutes after eating, weight loss from eating

a. Perforated DU
b. Ischemic
c. GU

1913. According to the most current recommendations at what size should operative intervention of
adrenal incindentalomas be considered?

a. 2cm
b. 3cm
c. 4cm
d. 5cm
e. 6cm

1914. Feale patient with history of breast cancer 4 years back operated, on MRI – 5cm mass adrenal
medulla, what is the treatment?

a. FNA
b. Excision
c. MIBG

1915. Tia (transient ischemic attact), investigation

a. Carotid duple
b. carotid angio

1916. Which is normal not palpable

a. Thyroid
b. Carotid angio

1917. Factors affecting recurrence of cancer colon is?

a. Staging
b. Lymph

1918. Scenario patient do upper endioscopy show grade ll varices, what is the best management?

a. Sclero therapy
b. Devascularizetion
c. Total shunt

1919. Old age male patient alcoholic present with melenra 2 days ago, what is the cause?

a. Bleeding duodenal ulcer


b. Bleeding gastric ulcer
c. Esophageal varices

1920. Scenario of jaunae new born ultrasound show no evidence of any extra hepatic biliary ducts,
what is the age beyond operation will have poor results?

a. 8 weeks
b. 12 weeks
c. 14 weeks
d. 18 weeks

1921. Scenario old age male patient suffering from upper abdominal pain, CT> show dialted distal 2/3
of pancreas MRCP show stone in juc=nction between distal 2/3 and 1/3 of pancreatic duct, what is the
best management?

a. Distal pancreatectomy
b. Duodenopancreatesctomy
c. ERCP and stent

1922. Scenario 2 years old baby with bilateral undscended testis. What is most diagnostic test?

a. Laparoscopy
b. Karyotyping
1923. Patient very malnourished, what is element should be normal if patient is for surgery?

a. Vitamin c
b. Folic acid
c. Copper
d. Iron

1924. Scenario patient papillary thyroidectomy yesterday on normal saline and dextrose present with
generalized seizure, what is the cause?

a. Cerebral metastasis
b. Calcium

1925. Patient did total thyroidectomy presented on 6 th day with tingling hand ca+ 6, what is
management?

a. Vitamin D
b. Calcium
c. Complete without treatment until 1m
d. Vitamin + calcium

1926. Scenario of patient vomiting many times ABG parameters written PH= 7.5, what is
management?

a. Ringer solution
b. Saline
c. -- formula
d. --formula

1927. Scenario female patient swelling 3cm with peu de orange with supra clavicular LN, without no
evidence of distant metastasis?

a. T3N0M0
b. T3N1M0
c. T4Nb1M0
d. T4Na1M1

1928. Scenario female patient trauma right hypochondria pain yesterday she missing herself until
next day presented with severe right hypocodrial pain, ultrasound normal, what is possible organ to be
injury?

a. Spleen
b. Liver
c. Stomach
d. Intestine
1929. Scenario patient with sudden lumbar back pain radiated to flanks, he give history of back
lumbar pain not radiated 2 days ago, vital sign, P=80, BP=110/70, Temperature=37, after 1 hour,
BP=80/40, P=106, temperature=37, what is management?

a. C.T
b. laparotomy
c. Repair of endoscopy for aneurysm

1930. Aspiration of pancreatic sac fluid rich with carbohydrates, diagnosis?

A C.T
b. Ultrasound
c. EUS
d. MRI

1931. GB with SCC not reach serosa, treatment by?

a. Extended cholecystectomy with regional lymphadenectomy

1932. Most pre-opertive risk factor stenosis

a. Aortic stenosis
b. Bronchitis

1933. Patient 110% burn, fluids should be?

a. Crystalloid
b. Colloids
c. Blood transfusion
d. Fresh frozen plasma

1934. Patient with blunt trauma after 6 hours is complaining of blood pressure 100/40 and
tachycardia, best initial investigation?

a. MRI
b. Ultrasonography
c. CT
d. DPL

1935. Case of pyogenic liver abscess surgery is done 1 week ago and evacuation is done, now patient
come with abdominal pain, distension, fever oliguria with urinary bladder pressure is 28 by u genital,
what will you do?

a. CT to detect collection
b. Lap and evacuation of collection
c. Re operate and colostomy

1936. In what basis of liver is divided anatomically according to?


a. Hepatic venous drainage
b. Hepatic arterial supply
c. Biliary drainage
d. Portal vein

1937. Scenario of patient trauma left hypochondrial pain radiated to shoulders, wital signs BP:
100/70, P;108, what is initial management?

a. Ultrasound
b. C.T
c. Laparotomy

1938. Scenario of patient hypovelemic shock, mild anxious BP: 120/80, P: 106, what is stage of shock?

a. l
b. ll
c. lll

1939. Patient suffer from anal discharge with cord like structure around anus?

a. Fistula
b. Fissure
c. Piles

1940. Scenario of the patients has anal swelling and bleeding swelling is protruded not reduced
except manually, management?

a. Hemorrhoidectomy
b. Band
c. Legation
d. laxative

1941. Scenario of patient suffering from bleeding per rectum-R show, ulcerative mass 4cm from anal
verge, management?

a. Colonoscopy with biopsy


b. CT
c. MRI

1942. Complication of laparoscopic bariatric surgery

a. Gallstones
b. Dumping syndrome

1943. On explosion30 years old male has splinter injury the proximial of Internal Carotid Artery
oesophagus, what will you do?

a. Repair of A + oesophagus
b. Ligate A + esophagectomy
c. Repair a + esophagectomy
d. Ligate A + oesophagus

1944. The most common intra abdominal solid tumour in children is?

a. Neuroblastoma
b. Nephroblastoma

1945. Which can compromise respiration during delivery?

a. Cystic hygroma
b. Troticollus

1946. Appendicitis complications?

a. Small bowel obstruction


b. Paralytic ileus

1947. Blanchable skin rash, this is prodomal or 1st degree ulcer

1948. Stones in pancreatic duct, management?

a. Modified peustoprocedure

1949. Scenario shows ulcer in great curative, what it he management?

a. Right hemicolectomy
b. Appendectomy

1950. Patient scenario ulcer in great curative, what is management?

a. Upper endoscopy and biopsy


b. Gastrectomy

1951. Scenario patient from swelling at mid 1/3 oesophagus, what is the best management?

a. Endoscopy and biopsy


b. Esophagectomy

1952. The commonest histological malignancy in GIT?

a. Adenocarcinoma
b. Squamous cell arcinoma

1953. The best prognosis of breast cancer?

a. Mucinous
b. Paget
c. Tubular
1954. Skin commensal that discovered cause pathogenesis?

a. Staph epidermic

1955. Scenario patient recurrent ingunal scrotal hernia irreducible only, not obstructed, not
strangulated history of prostatectomy, what is management?

a. Open repair with mesh


b. laparoscopic repair

1956. Boy with piece of wood in foot red streaking, what is the causative organism?

a. Treaptococcus pyogenes

1957. Patient develop systemic inflammation response syndrome (SIRS), what is paramote?

a. Temperature > 38.6


b. WBC < 4000

1958. Inguinal scrotal hernia, what is the sign?

a. Can’t palpate the vas

1959. One hour after laparoscopic repair of left inguinal hernia, the patient complains of severe
burning groin pain, which of the following is the most appropriate recommendation?

a. Immediate return to the operating room for laparoscopy


b. Nonsteriodal anti inflammatory drug
c. Neurocotin
d. Opiod analgesia
e. Inject groin region with local anaesthetic

1960. A 74 years old male presents to your clinic hoping to have his reducible umbilical hernia
repaired secondary to increasing but intermittent pain and discomfort, two days before his clinic visit
he had been discharged from the hospital for unstable angina for which he underwent balloon
angioplasty with placement of a bare metal coronary artery stent (BMS). When should he’s surgery be
scheduled?

a. 2 weeks
b. 1 month
c. 2 months

1961. The risk of regional node metastases in 0.70mm thick melanoma is?

a. < 5%
b. 10%
c. 20%
d. 30%
e. 50%

1962. What the organism which was suspect to be skin commensal but later discover to be th cause of
GIT infection?

a. Staph Epidermitis
b. Colstridi difficili
c. Staph aureus
d. Strepttoccoci

1963. The source of B growth factor?

a. Neutrophils
b. macrophage
c. Lymphocyte
d. Esinophil

1964. In cohort study which reduce risk factor of colon cancer?

a. Folic acid
b. Aspirin

1965. In case of septic shock patient need blood when Hb% is?

a. 6
b. 7
c. 8
d. 9

1966. Sigmoid polyp submucosal next step?

a. Surgery
b. Chemo
c. Radio

1967. GIST treatment?

a. Partial gastrectomy

1968. GB stones for 6 years, now high grade fever, RUQ tenderness, GB palpable diagnosis?

a. Acute cholecystitis
b. GB mucocele

1969. Epigastric pain with melena, UGI endoscopy showed leioyoma, treatment?

a. Wedge resection
b. Partial gastrectomy
1970. Most common complication after retal prolapsed repair?

a. Bowel obstruction
b. Organ or nerve damage
c. Recurrence
d. bowel habit changes

1971. Scenario of old patient with pain after eating afraid to eat?

a. Colitis
b. Íschemic colitis
c. Peptic ulcer

1972. H.pylori cause?

a. Type 1
b. Type 2
c. Erosive type

1973. Female 30 years brought to the emergency department after she stepped on a rusty nail and
sustained a puncture wound to the foot, the patient has been on the therapeutic dose of steroids for the
past 5 years for ulcerative colitis, her last tetanus toxoid booster was 8 years ago. What should the
patient received?

a. Tetanus toxoid booster


b. Human immunoglobin
c. Antibiotics with anerobic coverage
d. Tetanus toxoid plus human immunoglobulin
e. Tetanus toxoid plus human immunoglobin plus anti biotics with anerobic coverage

1974. Contraindication to irradiation in skin malignancy include the following?

a. Reccurent tumors (after irradiation)


b. Deeply invasive tumor
c. Lesion in upper half face
d. Old age group

1975. Under which group of conginetal hand anomalies syndactyly can be classified?

a. Over growth
b. Duplication
c. Undergrowth
d. Failure of differentiation of parts

1976. Patients burn in the entire back , scalp (50% of the head and the neck) and the posterioir thighs
has what percentage of his or her total body surface area burned?

a. 40%
b. 28%
c. 20%
d. 32%

1977. Ligation of testicular artery will result in testicular artrophy in?

a. 100% of cases
b. 75% of cases
c. 15% of cases
d. 0% of cases

1978. The common breast cancer is:

a. Intraductal
b. Mucinous

1979. Post cholecystectomy 4 weeks jaundice?

a. CBD stricture
b. Missed stone

1980. Bacteria previously thought normal skin commensal now believed highly pathogenic

a. pseudomonas
b. E-coli
c. Bacteroids
d. Fusobacteria

1981. Patient with stab wound left side chest between 9, 10, 11 tibs anti axillary line, patient is stable,
what is next?

a. Observation and follow up


b. Laparatomy
c. Thoracotomy
d. Chest tube

1982. Upper GI bleeding by endoscopy, mess after 35 cm of insertion of endoscopy, what is the best
for stagin?

a. EUS
b. C.T abdominal

1983. Under weight dripping saliva, abdominal distended, dyspnea, cianosis (NGT) ryles fill with air,
x-ray showed NGT in chest?

a. Continues cleaning and sunction of oesophagus


b. Repair of oesophagus fistula
c. Gastrostomy to decompress stomach
d. Intubation
1984. Abdominal pain 2 days ago after uncomplicated M.I from 2 weeks ultrasound and x-ray normal
no fever, no obstruction?

a. ischemic colitis
b. Mesenteric artery thrombosis
c. Mesenteric vein thrombosis

1985. Most common cause of react abdominal wall hernia?

a. Obese
b. Weak wall
c. missed defect

1986. During repair of incisional hernia color is injured, repair of the colon done, what you will do
about hernia?

a. Tension sutures
b. Primary tissue repair
c. Repair with mesh

1987. Child 2 years with undescuded testis, what is the best investigation, what is the best
investigation?

a. C.T
b. Ultrasound
c. Laparoscopy
d. Karytyping

1988. Gastroschiasis usually associated with?

a. Intestinal atresia

1989. Insect bite show colluitis with grayish discharge from it?

a. Antibiotic
b. Debridement + antibiotic
c. Debridement

1990. Wound infection with greenish discolorate gram positive RODS organ

a. Streptococci
b. Staphylococci
c. Debridement

1991. Myeoloprolifentive spleen, best investiagated by?

a. C.T with contrast


b. MRI
c. Ultrasound
d. DMSA scan

1992. Female with tumour base of the nose risk factor?

a. Sun exposure
b. Ultraviolet

1993. Endocopy – gastric mass, patient with melena mass in the body of stomach mucosal biopsy
negative treatment is?

a. Enucualtion
b. Wedge resection
c. Gatrotectomy

1994. Burn of arm the pathology of the hand

a. Demorcation
b. Coagulation
c. Stasis
d. Hyperemia

1995. Anal prolapsed 40 year old man soilinghis clothes treatment is?

a. Good salls
b. Thiersch procedure
c. Delorme
d. Altemeier

1996. Which of the following will be the main reason to consider research participants vulnerable?

a. Poorer than other participants


b. unavailable to protect their interests
c. Persons with emotional distress
d. Mentally disabled or handicapped

1997. Study for patients with M.I, among 100 controls, 50 died within the 3 days follow up period,
compared with 30 deaths among the 100 patients on the program. What is the relative risk of death for
the exercise group compared to control?

a. 0.30
b. 0.60
c. 1.67
d. 3.33

1998. Study to determine if there was difference in the prevalence of diabetes mellitus among health
care workers in Riyadh, the results was 30 out of 100 doctors with diabetic, compared to 50 out of 200
nurses, what is the test?

a. T-test
b. ANOVA
c. Chi-square
d. Correlation

1999. Pregnant lady came to your clinic to check HTN, the time of BP measure she extend her arm
regarding consent?

a. Pregnant lady can’t consent


b. Don’t take the BP measure she extend her arm regarding consent
c. No need consent, she extend her arm as a sign of agreement

2000. Someone from Pharmacy Company came to your clinic and explain to you about their drugs and
it is already use in another countries?

a. Take samples for trial


b. Prescribe the drug since it was used in another countries
c. The company should provide the efficiency of the drug

You might also like